Está en la página 1de 552

COMPENDIO

Dr. Alejandro Sandoval G. Travesi


CMN La Raza
ENARM
Consejos
1. Leer diariamente y en bloques

2. Adis Partys un tiempo

3. Has ejercicio y come bien durante el estudio

4. Toma algn curso bueno si tienes la posibilidad

5. Ten Fe.
HEMATOLOGIA
Cifras normales de hemoglobina y hematocrito
al nivel del mar (Wintrobe)

Hemoglobina Hematocrito

Recin nacido
19.5 5.0 54 10
Mujeres
14.0 2.0 42 5

Hombres 16.0 2.0 47 7


BANDAS
1-3%
QUIMICA Y ELECTROLITOS
GLUCOSA HDL
70- 105 40- 90

UREA LDL
10- 50 0 120

ACIDO URICO VLDL


2.5- 7.7 0- 50

NITROGENO UREICO ALBUMINA


6- 20 3.5- 5.5

CREATININA SERICA SODIO


0.5- 1.2 135- 145

COLESTEROL POTASIO
120- 200 4- 5.3

TRIGLICERIDOS CLORO
10- 160 98- 106
PFH Y ENZIMAS CARDIACAS
BILIRRUBINA TOTAL ALT/TGP
0 1.5 0 35

BILIRRUBINA INDIRECTA DHL


0 1.50 100 190

BILIRRUBINA DIRECTA GGT


0 0.5 7 50

AST/TGO CPK
6 38 0 226
GASES ARTERIALES
PH
7.35--7.45
PCO2
33-45mmHg
PO2
75-105mmHg
OSMOLARIDAD SERICA Y LCR
OSMOLALITY SERUM
275-295mOsmol/Kg
LCR
CELLCOUNT
0.5cells/mm3
CHLORIDE
118--132mEq/L
GAMMA GLOBULIN
3--12% total Proteins
GLUCOSE
40--70mg/dL
PRESSURE
70--180mm h2O
PROTEINS,TOTAL
< 40mg/dL
ANEMIA POR DEFICIT DE HIERRO
ANEMIA POR DEFICIT DE HIERRO
ANEMIA MEGALOBLSTICA
# de Eritrocitos Otros
Hemoglobina estudios
Hematocrito
V.G.M. Aspirado de
H.G. M. mdula sea:
C.M.H.G. N HIPERCELULAR
Reticulocitos N o lig.
Leucocitos bilirrubina
Diferencial L= indirecta = lig.
Frotis N=
Plaquetas DHL =
VITAMINA B12
A que se refiere el termino POLICITEMIA ESPURIA?
R = AUMENTO DEL HTO por aumento de la concentracin del VOLUMEN PLASMTICO NO POR EL > DE LA
MASA ERITROCITARIA

Cual es el cc de la TROMBOSIS ESENCIAL?


R = El primer signo puede ser trombosis mesentrica, heptica o portal. Eritromelalgia.

Cuales son los datos de laboratorio de la trombosis esencial?


1) PLAQUETAS >2, 000,000
2) FSP CON > DE PLAQUETAS GRANDES
3) MO CON > DE MEGACARIOCITOS
4) GEN FILADELFIA AUSENTE que descarta leucemia mieloide crnica

Que otras enfermedades pueden causar > plaquetario sin ser TE?
R = AR, CUCI e infeccin crnica

Cual es el manejo de la TE?


R = HIDROXIUREA y ASA hasta reducir a < 500, 000

Que es la MIELOFIBROSIS?
1) Trastorno mieloproliferativo caracterizado por FIBROSIS DE LA MO, esplenomegalia y cuadro leuco
eritroblastico en FSP con POIQUILOCITOS EN LAGRIMA.
2) HEMATOPOYESIS EXTRAMEDULAR
Cual es la etiologa de la mielofibrosis?
R = Se desarrolla en respuesta a un > EN LA SECRECIN DE FACTOR DE CRECIMIENTO DERIVADO DE LAS PLAQUETAS

Cual es el cc de la mielofibrosis?
1) Fatiga, distensin abdominal por esplenomegalia, DOLOR OSEO PRINCIPALMENTE RETROESTERNAL,
hepatomegalia.
2) Hemorragias por secuestro de plaquetas
3) HIPERTENSIN PORTAL Y VARICES ESOFGICAS POR HEMATOPOYESIS HEPTICA

Cuales son los datos de laboratorio y puncin obtenidos en la mielofibrosis?


1) Anemia
2) FSP con POIQUILOCITOS EN LAGRIMA, formas inmaduras mieloides y PLAQUETAS GIGANTES
desgranuladas
3) PUNCIN SECA por aumento de fibras reticulares

Cual es la triada clsica de la mielofibrosis?


1) POIQUILOCITOS EN LAGRIMA
2) SANGRE LEUCO-ERITROBLASTICA
3) PLAQUETAS GIGANTES
4) Supervivencia de 3-5

Cual es el manejo de la mielofibrosis?


R = Transfusiones continuas, TETOSTERONA. TMO + talidomida

Cuando se considera LEUCEMIA segn la OMS?


R = Cuando hay al menos 20% DE BLASTOS EN SANGRE O MO.
LEUCEMIA AGUDA LINFOBLASTICA
LEUCEMIA AGUDA LINFOBLASTICA
Cuales son las bases para el dx de LLC?
R = Es una malignidad ORIGINADA EN LOS LINFOCITOS B, en la cual cursan asintomticos, CON
INFILTRACIN DE LB EN LOS RGANOS

Cual es el cc de LLC?
R = Fatiga, LINFADENOPATIA Y HEPATO-ESPLENOMEGALIA

Cual es el hallazgo de laboratorio de la LLC?


1) LINFOCITOSIS AISLADA > 5000
2) MARCADOR CD 19 EN LB Y CD 5 EN LT

Cual es el manejo de la LLC?


R = Clorambucil

Cual es la estirpe histolgica de la que proviene la LEUCEMIA DE CLULAS VELLOSAS?


R = LINFOCITOS B

Cual es el cc de LCV?
R = Fatiga, ESPLENOMEGALIA o asintomtico
PUNCION SECA
I. MIELOFIBROSIS
Cual es la etiologa de la mielofibrosis?
R = Se desarrolla en respuesta a un > EN LA SECRECIN DE FACTOR DE CRECIMIENTO DERIVADO DE LAS
PLAQUETAS

Cual es el cc de la mielofibrosis?
1) Fatiga, distensin abdominal por esplenomegalia, DOLOR OSEO PRINCIPALMENTE RETROESTERNAL,
hepatomegalia.
2) Hemorragias por secuestro de plaquetas
3) HIPERTENSIN PORTAL Y VARICES ESOFGICAS POR HEMATOPOYESIS HEPTICA

II. LEUCEMIA DE CELULAS VELLOSAS


Cual es la estirpe histolgica de la que proviene la leucemia de clulas vellosas?
R = LINFOCITOS B

Cual es el cc de LCV?
R = Fatiga, ESPLENOMEGALIA o asintomtico
Que es el MIELOMA MLTIPLE?
1) Es una neoplasia de clulas plasmticas que sustituye la MO.
2) Hay DESTRUCCIN SEA con formacin de PARAPROTENAS plasmticas IgG o IgA
3) Hay HIPERVISCOCIDAD debido a paraprotenas
4) Puede provocar IRA por las paraprotenas

Cual es el cc del mieloma mltiple?


1) DOLOR SEO 70% que aumenta con los movimientos, principalmente en la espalda CON EVIDENCIAS DE FRACTURAS
VERTEBRALES
2) Edad > 65 , ANEMIA, dolor seo, infeccin
3) Amiloidosis manifiesta: macroglosia, neutropatia, ICC o hepatomegalia

Cuales son los datos de laboratorio del MM?


1) HIPERCALCEMIA, Sx anmico con ANEMIA NORMO-NORMO por efecto supresor de eritropoyetina por IL6
2) NEFRITIS intersticial con prdida DE CADENAS LIGERAS
3) Biopsia de MO con infiltracin normal o difusa

Cual es el manejo de mieloma mltiple?


1) Vincristina, doxorrubicina y dexametazona
2) TMO autologo
3) TALIDOMIDA + DEXAMETAZONA PARA RECADAS
4) Dx dif. Con plasmocitoma solitario que responde bien a la radioterapia que no presenta la clnica tpica del mieloma

Como estadificas la carga tumoral en el MM?


1) BAJA: con espiga IgG <5 gr/dl sin hipercalcemia ni IR
2) ALTA: con espiga IgG > 7 gr, Hto <25% y calcio > 12

Que es la MACROGLOBULINEMIA DE WALDESTROM?


R = Enfermedad MALIGNA DE LB, con hibrido de LB clula plasmtica con PARAPROTEINA IgM
Cuales son las causas secundarias ms comunes de PTI?
1) Frmacos como la heparina
2) LES y LLC

Cual es el mecanismo por el cual la heparina causa trombocitopenia?


R = Auto anticuerpos IgG vs factor 4 plaquetario

Cual es el manejo de la PTI?


1) PREDNISONA 1-2 mg/kg/dia con respuesta en una semana
2) INMUNOGLOBULINA iv para aumento de plaquetas
3) ESPLENECTOMA

Que es la PURPURA TROMBOCITOPENICA TROMBOTICA?


R = Es una enfermedad caracterizada por ANEMIA HEMOLTICA MICROANGIOHEPATICA,
TROMBOCITPENIA y AUMENTO NOTABLE DE DHL SRICA (a diferencia de la PTI donde no hay
elevacin de la DHL)

Cual es la etiologa de la PTT?


R = Deficiencia de PROTEASA que divide al factor de VON WILLEBRAND (VIII), por lo tanto se acumula
dicho factor originando aglutinacin de plaquetas en endotelio vascular. TAMBIN LO PUEDE
DESENCADENAR EL EMBARAZO, LOS ESTRGENOS ELEVADOS Y LAS INFECCIONES.

Cual es el cc de la PTT?
1) PENTADA CLASICA: Anemia hemoltica microangiopatica, Trombocitopenia, Afeccin neurolgica, Falla
Renal y Fiebre. ATARF
2) Fiebre, palidez, PETEQUIAS
3) Anemia, hemorragia.
4) Sntomas neurales como HEMIPARESIA QUE DURA MINUTOS
DEFICIT DE ADHESION PLAQUETARIO

I. Que es la tromboastenia de GLANZMAN?


A. Es un trastorno AR que produce hemorragias ( trastorno
plaquetario). LAS PLAQUETAS NO PUEDEN AGREGARSE por que no
hay RECEPTORES (GLUCOPROTEINAS IIB Y IIIA) para el fibringeno.
B. Se maneja con desmopresina

I. Que es el sndrome de BERNARD-SOULTIER?


A. Es un trastorno AR que OCASIONA HEMORRAGIAS. Las PLAQUETAS
NO SE ADHIEREN por FALTA DEL RECEPTOR IIB para el factor de vWF
B. Se maneja con transfusion de plaquetas
A que se debe la HEMOFILIA TIPO C y cual es su principal cc?
R = Dficit del FACTOR XI y se manifiesta con HEMORRAGIAS LEVES.

Que es la HEMOFILIA TIPO A?


R = Es un trastorno hereditario AR LIGADO AL X con dficit del FACTOR VIII y SOLO
AFECTA A VARONES

Cual es el cc de la hemofilia tipo A?


R = HEMORRAGIA EN ARTICULACIONES de rodilla, tobillo, codo, msculos y aparato GI

Cuales son los datos de lab en la hemofilia tipo A?


1) TPT SE PROLONGA nicamente
2) FACTOR VIII DISMINUIDO Y VWF NORMAL

Cual es el manejo de la hemofilia tipo A?


R = Concentrados de FACTOR VIII 4000 UI para 70 kg

Como puede transmitirse la enfermedad de anticuerpos vs el factor VIII?


R = Puede desarrollarse POST PARTO o sin una causa, hemofilia A que han recibido
concentrados plasmticos.
Cual es el cc de la enfermedad de anticuerpos vs el factor VIII?
R = Hemorragia grave

Cual es el manejo de eleccin en la enfermedad de anticuerpos vs el factor VIII?


R = CICLOFOSFAMIDA

Que es la HEMOFILIA TIPO B o enfermedad de Christmas?

R = Trastorno AR LIGADO AL X con dficit de FACTOR IX en el cual se eleva el TPT


Cual es el manejo de la hemofilia tipo B o enfermedad de Christmas?
R = Concentrados de FACTOR IX 6000 para 70 kg

Que factores de coagulacin se ven afectados en COAGULOPATIA POR ENFERMEDAD


HEPTICA?
1) Se fabrican todos los factores de coagulacin en el hgado EXCEPTO EL FACTOR VIII
2) Primero se ven afectados los vitamina K dependientes II, VII, IX y X y el factor V
3) Aumenta la fibrinlisis por la disminucin de plasmina

Cual es el manejo de la coagulopatia por enfermedad heptica?


R = PLASMA FRESCO CONGELADO
Cual es el cc de la coagulacin intravascular diseminada?
1) HEMORRAGIAS Y TROMBOSIS
2) Hemorragia +++ por puncin, heridas o espontanea
3) Trombosis + como isquemia digital o gangrena

Que es el SNDROME DE TROSEAU en la coagulacin intravascular diseminada?


R = Coagulacin intravascular diseminada manifiesta como TROMBOSIS VENOSA SUPERFICIAL Y PROFUNDA POR CNCER
principalmente de manera recurrente.

Cuales son los datos de lab para la coagulacin intravascular diseminada?


1) HIPOFIBRINOLEMIA
2) Aumento de productos de degradacin de la fibrina, TROMBOCITOPENIA y AUMENTO DEL TP Y TPT
3) DMERO D como producto de degradacin de la fibrina que aumenta si hay fallo heptico.

Cual es el manejo para la coagulacin intravascular diseminada?


1) TRATAR EL TRASTORNO SUBYACENTE
2) Heparina, S- AMINOCAPROICO (AUMENTA EL FIBRINGENO)
3) El XITO del tx consiste en que AUMENTE EL FIBRINGENO
4) CRIOPRECIPITADOS PARA FIBRINGENO

Como se diagnostica la disfibrinolemia?


R = Aumento del TIEMPO DE REPTILASA

Que medicamento se utiliza de eleccin en los estados de hipercoagubilidad?


R = WARFARINA
PATOLOGIA CELULA PATOGNOMONICA
ANEMIA DE CELULAS FALCIFORMES Cuerpos de Howell-Jolly:Estructuras nicas o
dobles, pequeas y redondas. Se observan
como grnulos densos y de color azul rojizo o
violeta. Localizados excntricamente

DEFICIT DE 6 GPDH Cuerpos de Heinz: Citoplasma de los glbulos


rojos , aparecen como pequeos puntos
oscuros bajo el microscopio

LINFOMA DE HODKING Reed Stemberg cells: Celula B modificada por


lo general es muy grande dando la pariencia de
ojos de Buho con nuclolos prominentes en
forma de inclusiones, la variedad mononuclear
tiene un solo nucleo y un nuclolo prominente.

LEUCEMIA AGUDA Bastones de Auer: Son estructuras filiformes


dentro de los blastos
ANEMIA SIDEROBLASTICA Cuerpos de Papenheimer: Son acmulos
de hemosiderina unida a protenas. Consisten
en grnulos basfilos, con las tinciones
habituales, que adems, se tien tambin de
azul con el colorante de Perls (azul de Prusia).
PATOLOGIA CELULA PATOGNOMONICA
INTOXICACION POR PLOMO, LEUCEMIA Y Punteado basofilo: Pueden ser agregados
TALASEMIA. ribosmicos originados por una degeneracin
vacuolar del citoplasma o precipitados de
cadenas globnicas libres. Consiste en puntitos
basfilos, con las tinciones habituales, de
tamao variable y dispersos por toda la
superficie del hemate.

ANEMIA MEGALOBLASTICA Anillos de cabot: Estn formados por restos de


la membrana nuclear o de microtbulos.
Consisten en una especie de hilos basfilos,
con las tinciones habituales, que adoptan una
forma de anillo o de ocho y que pueden
ocupar toda la periferia celular.

CLULA WARTHIN-FINKELDEY Sarampin: Clula gigante multinucletica con


citoplasma eosinoflico e inclusiones nucleares
PATOLOGIA
De que patologa son caractersticos los CUERPOS DE HEINZ?
R = Dficit de glucosa 6PDH

De que patologa son patognomnicos los cuerpos de HOWELL-JOLLY?


R = Anemia de clulas falciformes

De que patologa son patognomnicos los BASTONES DE AUER?


R = Leucemia mieloide aguda en la que adems la distingue la presencia de MIELOPEROXIDASA

Que es la LEUCEMIA MIELOGENA CRNICA?


R = Se caracteriza por SOBREPRODUCCIN DE CLULAS MIELOIDES (principalmente LEUCOCITOS), presenta EL COMOSOMA FILADELFIA
(translocacion de los brazos 9 y 22) BCR/ABL que produce TIROCINA CINASA.

Que diferencia la LEUCEMIA MIELOIDE AGUDA de la leucemia linfoblastica aguda?


R = Mieloperoxidasa que no se produce en la LLA

Cual es el hallazgo patognomnico de linfoma de Hodking?


R = RED STEMBERG CELLS (CLULA B MODIFICADA por lo general es muy grande dando la apariencia de ojos de Bho con nuclolos prominentes
en forma de inclusiones, la variedad mononuclear tiene un solo ncleo y un nuclolo prominente).

Que patologa te da trombos hialinos en la biopsia?


R = PTT y SUH

Citogenetica reconocible con T (18:14) de que enfermedad es patognomnico?


R = Linfoma no Hodking
ENF. DE HODKING
Caracterstica de la Enfermedad de Hodgkin?
Linfadenopatia indolora, Sntomas constitucionales, no se conoce su origen, diseminacin
ordenada

Cuadro clnico?
MASA INDOLORA (CUELLO), mas frecuente en varones, diseminacin ordenada, fiebre de larga
evolucin, DISMINUCIN DE PESO, PRURITO INTENSO, sudacin nocturna, DOLOR GANGLIONAR
(ALCOHOL)

Diagnostico definitivo?
Biopsia del ganglio linftico afectado, CLULAS DE REED-STENBERG, no hay anormalidades
cariotipicas

El virus Epstein Barr con que enfermedad tumoral se ha relacionado?


Enfermedad de Hodgking

En la Enfermedad de Hodgking cual es la variedad histolgica mas frecuente?


ESCLEROSIS NODULAR
En la Enfermedad de Hodgking cual es la variedad de mejor pronostico?
VARIEDAD LINFOCITICA

En la Enfermedad de Hodgking cual es la variedad menos frecuente?


Deplecin linfocitaria

En la Enfermedad de Hodgking que se relaciona con VIH?


DEPLECIN LINFOCITARIA

Tratamiento de la de la Enfermedad de Hodgking?


1) MOPP (mecloretamina, Vincristina, procarbacina, prednisona)
2) ABVD (adriamicina, bleomicina, Vincristina, Dacarbacina)

Clasificacin de ann-arbor?
Etapa 1 una sola cadena ganglionar
Etapa 2 2 regiones ganglionares (mismo lado del diafragma)
Etapa 3 regiones ganglionares en ambos lados del diafragma
Etapa 4 metstasis
(A asintomtico, B sntomas constitucionales)
ENF. NO HODKING
Caracterstica de la Enfermedad no Hodgking?
No tiene cl de Reed Stemberg, si hay citogentica reconocible T (8:14), correlacin con proto-
oncogen C-myc

Cuadro clnico de la E no H?
Linfadenopatia desordenada, a menudo diseminada al momento del diagnostico, sntomas
constitucionales

En que consiste el termino de ganglio de Richter?


1) Es un ganglio generalmente SUPRACLAVICULAR .
2) GANGLIO que comparte una LLC q posteriormente se convertir (EVOLUCIONARA) en LINFOMA NO
HODGKIN

Tratamiento de los linfomas no Hodgkin?


1) 1 solo ganglio afectado: radioterapia local si esta asintomtico valorar conducta expectante
2) Tx: clorambucilo o CVP+fludarabina, CHOP

Tratamiento para un MALTOMA?


Terapia para HELICOBACTER PYLORI
LEUCEMIA AGUDA
Cncer mas frecuente en nios?
Leucemia siendo la aguda la mas frecuente (80%)

El cncer de los ganglios linfticos se llama?


Linfoma

El cncer de medula sea?


Leucemia

Caracterstica de la leucemia Linfoblastica Aguda?


Tienen blastos (20% en MO y 90% en sangre perifrica) y su caract.
Es la capacidad de infiltrar rganos (encas, pericardio, testculos, MO,
SNC etc), 2da. Capacidad de producir Citopenias o Pancitopenias
Cuadro Clnico de LLA?
Por procesos de infiltracin hay: Hemorragias (petequias, equimosis) Adenopatas,
Hepato-esplenomegalias, puede llegar a una CID

Diferencia entre LLA y la hipoplasia medular y purpura trombocitopenica


idioptica

En que puede haber trombocitopenia pero TANTO EN LA HIPOPLASIA COMO EN


LA PURPURA NUNCA habr ganglios linfticos palpables o hepatoesplenomegalias

Caractersticas de la CID?
Tiempo de protrombina alargado, dmero D presente, trombocitopenia.

Que tipo de leucemia mielocitica aguda tiene la caracterstica de presentarse


como una CID
La promielocitica y la monocitica aguda (habitualmente son de buen pronostico
mientras que no se presente con CID)
Citogeneticas desfavorables?
Monosomia 5 y 7, Cromosoma Ph t(4;11) (Encontrar Cromosoma ph en LMA(2%) es
de muy mal pronostico habitualmente se observa en LMC (98%)

Con respecto a marcadores de superficie:


Determinacin de una Enzima desoxinucleotidil transferasa terminal.. es una LLA (95%)

Si es MIELOPEROXIDASA POSITIVA. LMA


Marcador CD19, CD10. Es una LLA de clulas B (mas frecuente)
Marcador CD2, CD5, CD7. Es una LLA de clulas T (menos frecuente)

Tratamiento de Leucemias Mielocitica Agudas?


1ra etapa: Induccin de la Remisin Daunorrubicina o Idarrubicina mas citarabina
2da etapa: Quimioterapia intensa de repeticin mas transplante de MO autologa

Tratamiento de la Leucemia Linfoblastica aguda?


1ra etapa: Induccin de la Remisin Vincristina, Prednisona, Daunorrubicina, L
Asparaginasa (4 medicamentos) Metrotexate (profilaxis del SNC)
2da etapa: Quimioterapia a dosis altas mas transplante de MO
Leucocitosis con Desviacin a la DERECHA:
Cuando los LINFOCITOS son los que estn elevados (arriba del 20%)

Leucocitosis con Desviacin a la IZQUIERDA:


Cuando los NEUTRFILOS estn elevados

Caracterstica Histolgicas de LMA?


Mieloperoxidasas positivas, cuerpos de AUER en citoplasma

Caractersticas Histolgicas de LLA?


Antgeno CALLA (Antgeno comn de LLA) y PAS Acido peridico de Shift
positivo

Citogentica Favorables?
En px con LMA t(8:21), t(15:17),inv (cr16), Inv (p13; q22) buen
pronostico
LEUCEMIA CRONICA
Caracterstica de una LLC?
1) Linfocitos mayores de 5000, linfocitos de apariencia normal (madura), con expresin de CD19, y CD5
2) Origen: malignidad de linfocitos B (95%) inmunoincopetentes

Cuadro clnico de una LLC?


Se presenta en px MAYORES DE 50 AOS (90%), linfocitosis impresionante, linfadenopatia y
Hepatoesplenomegalia

Clasificacin de la LLC (CLASIFICACION RAI)


ETAPA O Linfocitosis mas de 5000
ETAPA 1 Linfocitosis MAS LINFADENOPATIA (revisar al px)
ETAPA 2 Linfocitosis MAS ORGANOMEGALIA HEPATOESPLENOMEGALIA
ETAPA 3 Linfocitosis mas anemia
Etapa 4 Linfocitosis , lo anterior mas trombocitopenia (mal pronostico)

Laboratorios de una LLC?


LINFOCITOSIS AISLADA (mas de 5000), Leucos mayor de 20000, inmunogenotipo: CD19 (LLC de Celulas
B), CD5 (LLC de Celulas T)
Tratamiento de LLC con sintomatologa?
CLORAMBUCILO alternativa fludarabina en px jvenes

Caracterstica de LMC?
1) Leucocitosis, Cromosoma Ph t(9:22) presentes en un 95%,
2) Leucocitosis con desviacin a la izquierda (aumento delos
neutrfilos)

Tx de LMC?
EL TX no es urgente Mesilato de Imatinib, alternativa Hidroxiurea
o interferon alfa

Cuadro clnico de LMC?


Crecimiento impresionante de bazo, mas leucocitosis con desviacin
a la izq con serie mieloide, sensibilidad esternal, cromosoma Ph (95%)
SINDROME PARANEOPLASICO/ AC
Frecuentemente presentan pleocitosis linfocitaria en LCR
con aumento de protenas y de IgG. Algunos pacientes
presentan anticuerpos caractersticos en suero y LCR que
proveen una fuerte evidencia de que el cuadro
neurolgico es paraneoplsico y que adems sugieren el
tipo de cncer asociado, por ejemplo el anticuerpo anti-
Yo se asocia con degeneracin cerebelosa y cncer
ginecolgico, el anticuerpo anti-Hu con encfalomielitis
paraneoplsica y cncer pulmonar de clulas pequeas y
el anticuerpo anti-Ki con opsocionus y cncer de mama
MICELANEAS
En un paciente con anemia ferropnica que
esperara encontrar en su estudio de
laboratorio?
Ferritina, ferremia y saturacin de transferrina
bajas y transferrina alta.
Por cuanto tiempo se realiza el tratamiento
de la anemia ferropenica?
R = 6 meses
CARDIOLOGIA

ENARM
EKG
EKG
INTERVALOS
PR:
Se denomina as, al espacio que va del inicio de la Onda P al comienzo de la Onda R.
Valor normal: 120 a 200 milisegundos ( 0.12 0.20 segundos).

COMPLEJO QRS:
Corresponde a la activacin del miocardio ventricular.
Valor Normal: 60 a 100 milisegundos ( 0.06 0.10 segundos).
QT:
Se mide desde el inicio del Complejo QRS hasta el final de la Onda T y corresponde a la duracin total de la Sstole
Ventricular.
Valor normal: 240 a 480 milisegundos ( 0.24 0.48 segundos ).
( Varia de acuerdo a la frecuencia cardiaca )

SEGMENTO
ST:
Es el intervalo normalmente iso-electrico entre el final del Complejo QRS y el inicio de la Onda T.
Valor normal: 60 a 160 milisegundos ( 0.06 a 0.16 segundos )
EKG
INTERVALO QT
P r o l o n g a d o:
C a r d i o p a t a I s q u m i c a.
I. C. C.
M i o c a r d i t i s.
Drogas: Quinidina, Amiodarona, Antidepresivos triciclicos...
H i p o m a g n e s e m i a.
H i p o c a l c e m i a.
H i p o k a l e m i a ?.

A c o r t a d o:
R e p o l a r i z a c i n p r e c o z.
D r o g a s: Digital...
H i p e r c a l c e m i a.
H i p e r k a l e m i a.
HIPOKALEMIA

< 3 mEq/L: onda T


plana, depresin ST,
ondas U

< 2,5 mEq/L: onda U


prominente,
inversin onda T, PR
y QT prolongado,
QRS ensanchado
HIPERKALEMIA : POTASIO SRICO > 5 mEq/L

LEVE 5.0 - 5.5 mEq/L

MODERADA 5.5 - 6.0 mEq/L

SEVERA > 6.0 mEq/L


HIPERPOTASEMIA Y EKG

K+ 5.0 - 6.5 mEq/L Cambios mnimos

K+ 6.5 - 8.0 mEq/L Onda T picuda


Aplanamiento y desaparicin de la onda P
Depresin del segmento ST

K+ 8.0 a ms Ensanchamiento del QRS


BAV
Arritmias cardacas
Ritmo idioventricular
Paro cardaco en asistolia.
EJES ELECTRICOS EKG
PULSO VENOSO YUGULAR
AXVAYC
Se distinguen fundamentalmente dos ondas, la "a" y la "v". La primera, la onda
"a", ocurre justo antes del sstole, y se debe a la CONTRACCIN DE LA AURCULA
DERECHA (al final del distole, cuando se termina de vaciar al ventrculo derecho).
El colapso de la vena despus de la onda "a", es el descenso "x" y se debe a
la RELAJACIN DE LA AURCULA. AX

v
La onda " " se debe al LLENE PASIVO DE LA AURCULA DERECHA debido al
retorno venoso normal, mientras la VLVULA TRICSPIDE PERMANECE CERRADA
durante el sstole. Por lo tanto, es una onda que ocurre al mismo tiempo del sstole
y que se vera sobre el vena yugular. El colapso que se observa despus de la onda
y
"v", se denomina el descenso " ", que corresponde al PASO DE LA SANGRE
DE LA AURCULA AL VENTRCULO DURANTE EL DISTOLE, despus que se abre la
vlvula tricspide. VAY
Con registros muy finos, se describe una pequea muesca ubicada en el descenso
c
de la onda "a", que se ha llamado la onda " ", ATRIBUIDA AL CIERRE DE LA
VLVULA TRICSPIDE, despus que se ha terminado de contraer la aurcula
derecha y est comenzando el sstole, pero no es posible de ver a simple vista.
FARMACOLOGIA
ANGINA DE PECHO
ANGINA DE PECHO
ANGINA ESTABLE
Es la ms frecuente, aparece con el esfuerzo y
remite espontneamente con el reposo y/o la
medicacin. Posee una duracin de pocos
minutos y presenta un patrn regular, por lo que
el paciente puede ser capaz de identificarla e
incluso predecir su aparicin. Su origen se halla
primordialmente en una arteriopata
aterosclertica que causa la progresiva reduccin
de la luz vascular, de uno o varios vasos
coronarios, en porcentajes del orden del 70% o
superior.
ANGINA INESTABLE
La angina inestable no se relaciona con un mayor trabajo cardaco,
es decir no deriva de un mayor consumo miocrdico de oxgeno. Su
causa debe buscarse en una disminucin aguda del flujo cardaco
coronario, que puede deberse a la complicacin de una placa
aterosclertica coronaria por erosin, fisura o rotura y trombosis
sobreaadida que cause una interrupcin sbita del flujo coronario
o por causas extrnsecas al rbol coronario que produzcan
inestabilizacin. Su sintomatologa clnica es muy similar a la que
registra el infarto agudo de miocardio, sin embargo, en la angina
inestable no se produce necrosis miocrdica.

La angina inestable incluye diversos tipos de anginas caracterizadas


por su evolucin imprevisible, aunque no siempre fatal y que se
apartan claramente del patrn tpico de angina estable: angina de
reciente comienzo, angina progresiva, angina de reposo, angina
prolongada, angina vasoespstica o de Prinzmetal y angina
postinfarto, todas ellas consideradas urgencias mdicas.
IAM
IAM
IAM
IAM
SICA
CORONARIAS INVOLUCRADAS EN IAM
ARTERIA
DERIVACION CON
TIPO DE INFARTO CORONARIA COMPLICACIONES
ELEVACION DEL ST
RESPONSIBLE
Descendente
V1-V2 Septal anterior (ramos Bloqueos de rama
septales)
Descendente
Disfuncion VI, IC,
V3-V4 Pared anterior anterior (ramos
Bloqueo de rama
diagonales)
Hipotension (evitar
V5-V6 Lateral alto Circunfleja nitroglicerina o
morfina)
Posteroinferior con
Derecha (ramos
DII, DIII y aVF extension al Hipotension
proximales)
ventriculo derecho
Circunfleja o
V1-V4 Posterior descendente Disfuncion VI
posterior
PREGUNTAS
Cual es la triada clsica del infarto del ventrculo
derecho?
Hipotensin, campos pulmonares limpios y
elevacin de la PVY

Cual es el tratamiento de la taquicardia auricular


multifocal?
Suele asociarse a enfermedad pulmonar grave,
mejora con la ventilacin mecnica y la
oxigenacin
TIPOS DE SHOCK
TIPOS DE SHOCK
VALVULOPATIAS
VALVULOPATIAS
VALVULOPATIAS
RESUMEN SOPLOS
ESTENOSIS MITRAL; SINTOMAS Y SOPLO
DISNEA de esfuerzo, ORTOPNEA, EAP, disfona por compresin del NLR.
Imagen de 4 arcos en Rx por congestion venocapilar.
Soplo diastlico
INSUFICIENCIA MITRAL; SINTOMAS Y SOPLO
DISNEA de esfuerzo y fatiga, ORTOPNEA, DISNEA PAROXSTICA
NOCTURNA. Por prolapso valvular dolor torcico (signo de Barlow) dx dif con
SICA.
Soplo sistlico
ESTENOSIS AORTICA; SINTOMAS Y SOPLO
Triada clsica, ANGINA DE PECHO - IC SINCOPE. Palpitaciones, visin
borrosa. HV.
Soplo sistlico
INSUFICIENCIA AORTICA; SINTOMAS Y SOPLO
Disnea que va de esfuerzo a DISNEA PAROXSTICA NOCTURNA, estertores.
Soplo diastlico.
BRI
BRD
B AV I
Prolongacion del intervalo PR en forma
continua
>20 adultos
>16 en infantes
B AV II/ M1
B AV II/M2
B AV C
B AV C
B AV C
RESUMEN BLOQUEO AV I-III
RESUMEN BLOQUEO AV I-III
BLOQUEOS AV MANEJO
BLOQUEOS AV MANEJO
FA
QT LARGO
TRAZOS EKG PATOLOGICOS
SX WPW
SX BRUGADA
INSUFICIENCIA CARDIACA
INSUFICIENCIA CARDIACA
ENDOCARDITIS INFECCIOSA
Endocarditis infecciosa
Profilaxis antibitica en procedimientos dentales,
cavidad oral, respiratorio y esfago

A. Amoxicilina 2gr 1 hora antes


B. Ampicilina 2gr IM 30 minutos antes
(en intolerancia oral)
Alergia a penicilina
1) Claritromicina 500 mg 1 hora antes
2) Clindamicina 600 mg 1 hora antes
3) Cefalexina 2gr 1 hora antes
Endocarditis infecciosa
Profilaxis antibitica en procedimientos genitourinarios
y gastrointestinales

Ampicilina 2 gr IM o IV + Gentamicina
1,5mg/Kg/ 30 minutos antes, 6 h despus
Ampicilina 1gr IM/IV o Amoxicilina oral 1gr

Alrgicos a penicilina

Vancomicina 1gr IV en 1-2 h + Gentamicina


1,5 mg/Kg IV/IM terminando la perfusin 30
minutos despus del procedimiento
LESIONES DE JANEWAY
MANCHAS DE ROTH
NODULOS DE OSLER
TAMPONADE
JOVEN + DOLOR PRECORDIAL, FIEBRE Y ST (LESIN)
EXTENSA = PERICARDITIS AGUDA. LO EXACERBA LA RESPIRACIN.
ES FRECUENTE ANTECEDENTE DE IRAS VIRALES
DE NO CONOCERSE LA ETIOLOGA SE DAN AINES (AAS)
PERICARDITIS
El supradesnivel del ST en la pericarditis se diferencia del infarto
agudo del miocardio porque en ste el supradesnivel es convexo y
ms localizado, pueden coexistir ondas T negativas al mismo tiempo
del supradesnivel, presencia de ondas Q cuando es un infarto Q y el
EKG no normaliza antes de hacerse negativa la onda T.

De la angina de Prinzmetal se distingue porque el supradesnivel es


transitorio y slo durante el dolor; adems, compromete
derivaciones ms localizadas.

La imagen de la repolarizacin precoz es parecida pero nunca hay


depresin del PR, no evoluciona y no hay sntomas.
MIOCARDIOPATIAS
Adulto + anasarca + complejo bajos =
pericarditis constrictiva
Rx de mujer , imagen de hiperflujo pulmonar, ECG con
BRD y eje a la derecha= Comunicacin Interauricular
(OS)
CARDIOPATIAS CIANOGENAS Y
ACIANOGENAS
CIV
CIA
COARTACION AORTICA
ANEURISMAS
CARDIOPATIAS CONGENITAS
DISECCION AORTICA
EMBOLIA VS TROMBOSIS
OCLUSION ARTERIAL CRONICA
HAS
HAS
HAS
HAS
Cual es el signo electrocardiografico mas comn en tromboembolismo pulmonar?
R = Taquicardia sinusal.

-A 42-year-old man develops shortness of breath (SOB) and chest pain 7 days after an open cholecystectomy.
His blood pressure is 145/86 mm Hg, pulse is 120/min, respirations 24/min, and oxygen saturation of 97%.
Pulmonary embolism is clinically suspected. Which of the following is the most common ECG finding of
pulmonary embolism?

(A) a deep S wave in lead I


(B) depressed ST segments in leads I and II
(C) prominent Q wave in lead I, and inversion of T wave in lead III
(D) sinus tachycardia
(E) clockwise rotation in the precordial leads
Correcta D

Cuales son los sntomas de estenosis mitral?


R = DISNEA, ortopnea y disnea paroxstica nocturna

Como es el soplo en insuficiencia mitral?


R = PanSISTOLICO en vrtice e irradiado a axila.

-A 25-year-old woman is found to have a midsystolic murmur on routine evaluation. The murmur does not
radiate but it does increase with standing. She otherwise feels well and the rest of the examination is normal.
R = Mitral valve prolapsed.

-A 65-year-old man with a previous history of an anterior MI comes for follow-up. On examination, he has a
systolic murmur heard best at the apex and radiating to the axilla. Transient external compression of both arms
with blood pressure cuffs 20 mm Hg over peak systolic pressure increases the murmur.
R = Regurgitacin mitral
Que caracteriza al signo de MUSSET y en que patologa se presenta?
R = Hay movimientos de la cabeza en cada pulsacin debido a INSUFICIENCIA DE LA VLVULA AORTICA.

-A 75-year-old man is bought to the hospital because of a syncopal episode. There was no incontinence or post-event confusion. On examination, his blood
pressure is 140/80 mm Hg, pulse 72/min with no postural changes. His second heart sound is diminished and there is a systolic ejection murmur that radiates
to the carotids. With the Valsalva maneuver, the murmur decreases in length and intensity.
R = Aortic stenosis.

En que patologa se escucha el soplo de AUSTIN FLINT?


Insuficiencia aortica

Que sucede cuando hay regurgitacin aortica en forma aguda?


Insuficiencia ventricular izquierda manifestada como EDEMA AGUDO PULMONAR.

-A 32-year-old asymptomatic woman has a rapidly rising, forceful pulse that collapses quickly. Which of the following is the most likely diagnosis?
(A) mitral stenosis
(B) mitral regurgitation
(C) aortic stenosis
(D) aortic regurgitation
Respuesta correcta D

Que medicamento es de eleccin en el sndrome de Marfan con regurgitacin aortica?


R = BETABLOQUEADORES

Que medicamentos pueden disminuir la dosis de homocisteina?


R = Acido flico, B6 y B12.

Cul es el fenmeno de hibernacin y aturdimiento miocardico?


R = Son las reas del miocardio que se encuentran subperfundidas que se adaptan para ser viables con disfuncin contrctil sostenida.

Que es el sndrome de TIETZE?


R = Inflamacin de uniones condrocostales tumefactas, rojas que causa dolor retroesternal

Que antianginosos han demostrado prolongar la vida?


R = Los BETABLOQUEADORES a excepcin de el pindolol que exacerba la angina
RESUMEN SOPLOS
ESTENOSIS MITRAL; SINTOMAS Y SOPLO
Disnea de esfuerzo, ortopnea, EAP, disfona por compresin del NLR.
Imagen de 4 arcos en Rx por congestin venocapilar.
Soplo diastlico
INSUFICIENCIA MITRAL; SINTOMAS Y SOPLO
Disnea de esfuerzo y fatiga, ortopnea, disnea paroxstica nocturna. Por
prolapso valvular dolor torcico (signo de Barlow) dx dif con SICA.
Soplo sistlico
ESTENOSIS AORTICA; SINTOMAS Y SOPLO
Triada clsica, angina de pecho - IC Sincope. Palpitaciones, visin borrosa.
HV. Signo de Musset.
Soplo sistlico
INSUFICIENCIA AORTICA; SINTOMAS Y SOPLO
Disnea que va de esfuerzo a disnea paroxstica nocturna, estertores. PULSO
DE CORRIGAN , QUINQUE Y DUROZEIZ . Soplo de Austin Flint.
Soplo diastlico.
En cuanto tiempo tiene que estar el TTP cuando se da tratamiento anticoagulante?
R = 50-70 segundos.

En que pacientes se utiliza la profilaxis antiarrtmica?


R = LATIDOS ECTPICOS FRECUENTES y taquicardia ventricular.

Como se trata una fibrilacin ventricular que no responde a una cardioversin?


R = Amiodarona y RCP

Como se dx infarto del ventrculo derecho?


R = Con la ELEVACIN DEL ST DEL HEMITORAX DERECHO

-A 69-year-old woman complains of some atypical chest pain 2 days prior to presentation. On examination, the
JVP is at 8 cm, positive Kussmauls sign, and normal heart sounds. The lungs are clear. The ECG is abnormal, and
the CXR shows a normal cardiac silhouette.

Que tan frecuente se presenta un AUNERISMA VENTRICULAR IZQUIERDO POST IAM?


R = 20% y se da el dx por medio de la ELEVACIN PERSISTENTE DEL ST DURANTE 4-8 SEMANAS.

-Three months after an anterior MI, a 73-yearold man has a follow-up ECG. He is clinically feeling well with no
further angina symptoms. His ECG shows Q waves in the anterior leads with persistant ST-segment elevation.
The current ECG is most compatible with which of the following diagnosis?

(A) ventricular aneurysm


(B) hibernating myocardium
(C) acute infarction
(D) silent infarction
(E) early repolarization
Que caracteriza al SNDROME DE DRESSLER?
R = SE PRESENTA POST IAM, debido a un FENMENO AUTO INMUNITARIO CON PERICARDITIS, FIEBRE,
LEUCOCITOSIS y ocasionalmente derrame pericardico o pleural.

-A 67-year-old man presents with an anterior myocardial infarction (MI) and receives thrombolytic therapy.
Three days later, he develops chest pain that is exacerbated by lying down, and his physical findings are normal
except for a friction rub. His ECG shows evolving changes from the anterior infarction but new PR-segment
depression and 1-mm ST-segment elevation in all the limb leads. Which of the following is the most likely
diagnosis?

(A) reinfarction
(B) pulmonary embolus
(C) viral infection
(D) post-MI pericariditis
(E) dissecting aneurysm
Respuesta correcta D

Cual es el manejo del infarto agudo del VD?


R = CARGA LIQUIDA para mejorar el llenado ventricular derecho e INOTRPICOS.

Cual es el mecanismo de accin de los antiarrtmicos clase Ia y cuales son?


R = BLOQUEADORES DE LOS CANALES DE SODIO. Procainamida, quinidina y disopiramida.

Cual es el mecanismo de accin de los antiarrtmicos clase Ib y cuales son?


R = BLOQUEADORES DE LOS CANALES DE SODIO. Lidocaina y difenilhidantoina.

Cual es el mecanismo de accin de los antiarrtmicos clase II y cuales son?


R = BETABLOQUEADORES, retardando la conduccin AV. Propanolol, metoprolol y esmolol.
Cual es el mecanismo de accin de los antiarrtmicos clase III y cuales son?
R = PROLONGAN EL POTENCIAL DE ACCIN. Amiodarona, sotalol, dofelitida.

Cual es el mecanismo de accin de los antiarrtmicos clase IV y cuales son?


R = BLOQUEADORES DE LOS CANALES LENTOS DE CALCIO. Verapamil, diltiazem, digoxina y adenosina.

Con que fenmenos se relaciona la taquicardia paroxstica supraventricular, la cual suele cursar asintomtica.
R = Intoxicacin por digitalicos, bloqueos AV y fenmeno de reentrada.

Que tratamiento se utiliza en taquicardia paroxstica supraventricular en caso de estar contraindicados los
antiarrtmicos clase IV y que ha demostrado 100% de xito?
R = Cardioversion.

Cual es el frmaco de eleccin en caso de prevencin de ataques de taquicardia paroxstica supraventricular?


R = DIGOXINA

Que caracteriza al sndrome de LOWN-GANONG-LEVINE?


R = INTERVALO PR CORTO Y MORFOLOGA NORMAL DE QRS

En que casos no necesita tratamiento las taquicardias supraventriculares causadas por vas accesorias o sndrome de
preexitacion?
R = En casos de no presentar palpitaciones, mareos o sincope.

Cual es el procedimiento de eleccin en pacientes con sndrome de preexitacion?


R = ABLACIN POR RADIOFRECUENCIA

Que agentes farmacolgicos deben evitarse en el sndrome de preexitacion?


R = Digoxina, betabloqueador y bloqueador de los canales de calcio.

Cual es el tratamiento a largo plazo de el sndrome de preexitacion?


R = Procainamida, verapamil y digoxina.
Cuales son los frmacos de eleccin en extrasstoles ventriculares sintomticas?
R = BETABLOQUEADORES.

Que medida se utiliza cuando la taquicardia ventricular es recurrente?


R = Marcapasos

- A22-year-old woman complains of palpitations and has a regular heartbeat at a rate of 170/min, with a blood pressure of 110/70 mm Hg. The rate
abruptly changes to 75/min after applying carotid sinus pressure. Which of the following is the most likely diagnosis?
(A) sinus tachycardia
(B) paroxysmal atrial fibrillation
(C) paroxysmal atrial flutter
(D) paroxysmal supraventricular tachycardia (PSVT)
Respuesta correcta D

Cual es el mecanismo de eleccin para la prevencin de muerte sbita en un paciente con factores de riesgo?
R = Desfibrilador implantado

Que caracteriza al sndrome QT PROLONGADO?


R = SINCOPE RECURRENTE, arritmias ventriculares y muerte sbita. Es una ANOMALA CONGNITA hereditaria que afecta los CANALES DE Na y K.

Cual es la teraputica mas eficaz en el sndrome de QT prolongado?


R = BETABLOQUEADORES Y DESFIBRILADOR IMPLANTADO.

Que hallazgo EKG caracteriza al bloqueo AV tipo I?


R = PROLONGACIN DEL PR > .20 EN ADULTOS Y > .16. LA ALTERACIN ES NODAL.

Que hallazgo EKG caracteriza al bloqueo AV tipo II?


1) - 1er grado o fenmeno de WENCKEBACH, MOBITZ I: prolongacin progresiva del PR hasta que 1 onda P no conduce.
2) - 2do grado o MOBITZ II: intervalo PR, sin embrago hay onda P que no conduce con un patrn 2:1 o 3:1

En que casos se diagnostica el sndrome de SENO ENFERMO?


R = PARO SINUSAL, bloqueo sinoauricular, BRADICARDIA SINUSAL PERSISTENTE o bradiarritmias.
Cual es el manejo del sndrome de seno enfermo?
R = TEOFILINA ORAL Y MARCAPASOS.

Cual es la indicacin teraputica del Mobitz III?


R = MARCAPASOS

Cuales son los sntomas inadvertidos de ICC?


R = Tos crnica, nicturia

Que medicamento se utiliza en caso de intoxicacin por digoxina?


R = Anticuerpos FAP

Cual es el cuadro clnico caracterstico de alguien con ICC con edema agudo pulmonar?
R = DISNEA, tos, ESPUTO ROSADO, diaforesis, cianosis.

En que consiste el fenmeno de reentrada?


R = Involucra un circuito que conduce de forma anterograda al ventrculo y de forma retrograda a la aurcula
utilizando vas accesorias.

Que tratamiento se utiliza en ICC despus de que los diurticos y nitratos no mejoran la sintomatologa?
R = NISERITIDA, forma recombinante de pptido atrial natriuretico del cerebro humano.

Que agente antihipertensivo se debe evitar en ICC?


R = CALCIOANTAGONISTAS

Como es tpicamente la Rx de torax en edema agudo pulmonar con ICC?


R = PATRN EN MARIPOSA en la distribucin del edema alveolar.
En el EAP la presin capilar en cuna se encuentra siempre elevada, a que cantidad aproximadamente?
R = > 25 mm Hg

La morfina es eficaz en el manejo de derrame pleural cardiogenico, debido a que mecanismo?


R = AUMENTA LA CAPACITANCIA VENOSA, disminuye la presin auricular izquierda y disminuye la ansiedad.

Cul es la diferencia entre cardioversin y desfibrilacin?


R = La primera se encuentra sincronizada con complejo QRS y la 2da no hay sincronizacin.

Cules son los agentes ms comunes para miocarditis aguda?


R = VIRAL (COXACKIE) bacteriano, riketsias, espiroquetas, micoticos y parasitarios.

-A 23-year-old man develops sharp left-sided chest pain, fever, and a friction rub heard at the lower left sternal
border, unaffected by respiration. The pain is also aggevated by lying down and relieved by sitting up. He is
otherwise well with no other symptoms and the remaining physical examination is normal. Which of the
following is the most likely cause for his symptoms?

(A) rheumatic fever


(B) tuberculosis (TB)
(C) herpes simplex virus
(D) MI
(E) coxsackievirus
Respuesta correcta E

La miocarditis Riketsial , por que enfermedad es originada?


R = TIFO, fiebre de las montanas rocallosas y fiebre Q.
Cual es el agente causal y cual es el periodo de incubacin de la ENFERMEDAD DE CHAGAS?
R = Es causado por el TROFOSZOITO TRIPANOSOMA CRUZI y su principal manifestacin clnica aparece
en 10 aos.

Que afeccin sistmica es la regla para un individuo que padece Chagas?


R = MEGAESOFAGO O MEGACOLON.

Entre las afecciones parasitarias cual es la mas frecuente de afeccin cardiaca?


R = La triquinosis.

Cual es la fisiologa de la maniobra de valsalva?


R = Se produce un aumento del tono parasimptico por liberacin de acetilcolina, lo que genera un
retardo en la conduccin AV causando un bloqueo AV transitorio.

Que dato puede ser nico o inicial en el EKG en MIOCARDITIS INFECCIOSA?


R = Ectopia ventricular

Que medicamento esta indicado en el espasmo coronario inducido por la COCANA?


R = BETABLOQUEADORES Y CALCIOANTAGONISTAS

Cuales son los sntomas de MIOCARDIOPATIA DILATADA?


R = Disnea, dolor torcico .

Cual es el manejo del paciente con CARDIOPATA HIPERTRFICA?


R = BETABLOQUEADORES, CALCIOANTAGONISTAS Y MARCAPASOS.
-A 45-year-old woman has developed increasing SOB on exertion and fatigue. She has a loud systolic ejection murmur heard best at the left
sternal border, and the murmur increases with standing. A double apical impulse is also felt.
R = Cardiomiopatia hipertrofica.

66. Cuales son las causas principales de MIOCARDIOPATIA RESTRICTIVA?


R = AMILOIDOSIS, RADIACIN, fibrosis por ciruga, sarcoidiosis, hemocromatosis y sndrome carcinoide.

-57 A 63-year-old man develops edema, and dyspnea on exertion. He has no prior cardiac or renal conditions, and his examination is
significant for macroglossia, elevated jugular venous pressure (JVP), hepatomegaly, and 3+ pedal edema. His investigations reveal 3.5 g/d of
protein in the urine, anemia, normal fasting glucose, and serum immunoelectrophoresis is positive for a monoclonal immunoglobulin.
Which of the following is the most characteristic neurologic finding associated with this condition?

-Peripheral motor and sensory neuropathy: In addition to peripheral motor and sensory neuropathy, cardiac involvement, tongue
enlargement, gastrointestinal (GI) manifestations, and carpal tunnel syndrome are also seen in amyloidosis. The specific diagnosis requires
tissue biopsy with presence of amyloid with specific stains. In primary amyloidosis and myeloma, the amyloid protein is of the ALtype. In
reactive amyloidosis, the protein is of the amyloid Aprotein (AA) type.

Cuales son los criterios mayores de Jones de CARDIOPATA REUMTICA?


1) CARDITIS: pericarditis, ICC, cardiomegalia y soplos.
2) ERITEMA MARGINADO
3) NDULOS SUBCUTNEOS: los ndulos subcutneos SON INDOLOROS y menores de 2 cm
4) COREA DE SHYDENHAM: Movimientos coreoatetoides
5) ARTRITIS: dura de 1-5 semanas y responde a AINES

Cuales son los criterios menores de cardiopata reumtica?


R = Fiebre, VSG elevada, prolongacin del PR.

Cual es el manejo de cardiopata reumtica?


R = PENICILINA BENZATINICA 1.2 millones DU o PEN-PRO 600, 000 x 10 dias.

Como se previene la fiebre reumtica?


R = PEN-BEN 1.2 cada 4 semanas

En los pacientes con Wolf-Parkinson-Withe. Que medicamentos estn contraindicados?


R = ADENOSINA, DIGOXINA, CALCIOANTAGONISTAS Y BETABLOQUEADORES YA QUE BLOQUEAN EL NODO AV.
En los pacientes con Wolf-Parkinson-Withe. Cual es el manejo indicado?
R = Cardioversin elctrica y ablacin por radiofrecuencia de vas accesorias.

Cuales son los sndromes de preexitacion?


R = Sndrome de WPW y Lown-Ganong-Levine

Cual es el microorganismo causal de la ENFERMEDAD DE LYME?


R = Borrelia Burgdorjeri

Que caractersticas tiene la pericarditis secundaria a sndrome urmico?


R = Pericardio afelpado, derrame hemorrgico y exudativo.

Que caracteriza al sndrome de Dressler?


R = Pericarditis post IAM o ciruga del corazn abierto y de etiologa autoinmune.

Como se define la hipertensin pulmonar primaria?


R = Como aumento de la resistencia vascular pulmonar sin enfermedad subyacente.

-Auscultation of the heart of a 17-year-old boy reveals an increased intensity of the pulmonary component of the
second heart sound. He complains of dyspnea on exertion but no other cardiac or pulmonary symptoms. Which of the
explanations is the most likely cause of his dyspnea?
(A) pulmonary stenosis
(B) aortic stenosis
(C) MI
(D) pulmonary hypertension
(E) systemic hypertension
Respuesta correcta D

Cual es el manejo de hipertensin pulmonar primaria?


R = CALCIOANTAGONISTAS, anticoagulantes, prostaciclina (potente vasodilatador pulmonar)
Cual es el tratamiento de eleccin de la hipertensin primaria?
R = Endotelina

A que hace alusin el termino Cor-pulmonale?


R = Hipertrofia ventricular derecha e insuficiencia por enfermedad pulmonar, mas frecuente en EPOC

Cual es el dato de laboratorio mas frecuente en el Cor-pulmonale?


R = Policitemia

Cuales son los tumores que afectan al corazn?


R = Carcinoma broncogenico, carcinoma mamario, melanoma maligno, linfomas, carcinoma de clulas renales y sarcoma de kaposi.

Cuales son los tumores primarios del corazn mas frecuentes?


R = MIXOMA AURICULAR el cual es un tumor benigno que puede embolizar sistmicamente y se ubica comnmente en la aurcula derecha.

-A 47-year-old woman has new-onset transient right arm weakness and word finding difficulty symptoms lasting 3 hours. She is also
experiencing exertional dyspnea, and had a syncopal event 1 month ago. Her echocardiogram reveals a cardiac tumor in the left atrium, it
is pendunculated and attached to the endocardium. Which of the following is the most likely cause of this lesion?
(A) myxoma
(B) sarcoma
(C) rhabdomyoma
(D) fibroma
(E) lipoma
Respuesta correcta A

En que consiste la ENFERMEDAD DE BURGUER?


R = Es un PROCESO INFLAMATORIO Y TROMBOTICO de las arterias y venas perifricas PRODUCIENDO CLAUDICACIN, DOLOR Y NECROSIS.

Cuales son los datos clnicos mas frecuentes de la enfermedad de Takayasu?


R = Soplos vasculares, PULSOS PERIFRICOS DISMINUIDOS y ASIMETRA DE LA PA DE LAS EXTREMIDADES.

Cual es el tratamiento de la enfermedad de Takayasu?


R = ESTEROIDES para la inflamacin y despus COLOCAR STENT O PUENTES.
Cual es el tratamiento para pacientes con displasia fibromuscular de la intima?
R = ASA

Que fenmenos deben descartarse en una persona que haya cursado con enfermedad de Raynaud?
R = 80% ESCLERODERMIA, MIOSITIS, LES, AR.

Cual es el sndrome doloroso regional o complejo tipo 1 distrofia simptica refleja?


R = DOLOR ARDOROSO O QUEMANTE de mas duracin de lo esperado POR TRAUMATISMO EN EXTREMIDAD
secundario a aplastamiento o quemadura.

Cuales son los datos clnicos de la distrofia simptica refleja?


R = DOLOR LOCAL, hiperestesia, calor, RIGIDEZ MUSCULAR, RIGIDEZ ARTICULAR, con consiguiente DESUSO Y
OSTEOPENIA RADIOLGICA

Que tratamientos se utilizan en distrofia simptica refleja?


R = OPIOIDES Y GABAPENTINA

Cual es el tipo mas comn de sndrome de salida torcica?


R = NEUROGENICO 90%, venoso y arterial.

En el sndrome de salida torcica , que es el sndrome de PAGET-SCHROETTER O TROMBOSIS DE ESFUERZO?


R = Se presenta como EDEMA UNILATERAL AGUDO DEL BRAZO, pesantez de axila, CIANOSIS DE LA MANO E
INGURGITACIN DE LAS VENAS DEL HOMBRO Y TRAX.

Qu medidas se toman en un sujeto con choque que no responde a estmulos?


R = DEXTROSA AL 50%, naloxona 2 ml iv. Manteniendo la diuresis horaria mayor de .5

Que te indica la PVC?


R = MENOR DE 5 mm hg indica HIPOVOLEMIA, MAYOR DE 18 SOBRECARGA VOLUMEN.
Cules son los datos clnicos de taponamiento cardiaco?
R = Taquicardia, HIPOTENSIN, pulso paradjico, AUMENTO DE LA PVY. Excepto en pacientes urmicos o con
hipotiroidismo.

-A 25-year-old man complains of left precordial chest pain that radiates to the left shoulder but not down the
left arm. The pain is accentuated by inspiration and relieved by sitting up. The pain is accompanied by fever and
chills. His blood pressure is 105/75 mm Hg, pulse 110/min and regular, and temperature 37.5C. Aside from the
tachycardia, there are no abnormal physical findings in the heart or lungs. The ECG shows STsegment elevation
in all leads except aVR and VI. On the third hospital day, the patients blood pressure falls, JVP rises, and he
goes into CHF. Which of the following is the most likely diagnosis?
(A) a second pulmonary embolus
(B) extension of a myocardial infarct
(C) cardiac tamponade
(D) secondary bacterial infection
(E) rupture of a chordae tendineae
Respuesta correcta C

-A 56-year-old man presents with SOB, fatigue, and edema. He has also noticed weight gain, abdominal
discomfort, and distension. He has a prior history of lung cancer treated with radiotherapy to the chest. There
is no history of liver or cardiac disease in the past. On examination, he has an elvated JVP, prominent y descent
of neck veins, and positive Kussmauls sign. The heart sounds are normal. The CXR shows a normal cardiac
silhouette and the ECG has low voltages.

-A 55-year-old woman with metastatic lung cancer presents with dyspnea and pedal edema. On examination,
the JVP is at 10 cm, with a negative Kussmauls sign. The heart sounds are diminished and the lungs have
bibasilar crackles. The ECG shows QRS complexes of variable height

Como afecta el cliostazol en la enfermedad arterial perifrica?


R = Es un INHIBIDOR DE LA FOSFODIESTERASA 3 que disminuye la agregacin plaquetaria, activa la lipoprotena
lipasa y causa vasodilatacin.
Cul es el objetivo del tratamiento en un paciente con choque?
R = PVC 8-12, PA 65-90, ndice cardiaco 2-4 lts/min, y O2 venoso > 70%.

Cules son los efectos de la dopamina?


R = 2-3 ug/kg/min estimula RECEPTORES DOPA y B AGONISTAS con AUMENTO DEL FG, FC Y IONOTROPISMO. >5 ug/kg/min
estimula receptores ALFA adrenergicos produciendo VASOCONSTRICCIN PERIFRICA.

Cual es el medicamento de eleccin para choque cardiogenico?


R = DOBUTAMINA con aumento del ionotropismo y disminucin de la poscarga

Que efecto tiene la desmopresina utilizada en choque distributivo?


1) Vasoconstriccin perifrica, disminucin de la FC, vasodilatacin coronaria, pulmonar y cerebral.
2) La vasoconstriccin es mediada por oxido ntrico.

Cual es la causa de la miocardiopata hipertrfica?


R = DEFECTO CONGNITO en los sarcomeros.

Cual es la causa de miocardiopata dilatada?


R = Isquemia, cardiotoxicos. EMBARAZO.

Cual es la causa de la miocardiopata restrictiva?


R = Primaria: fiebre endomiocardica, endocarditis de coffler o ideopatica, Secundaria. RADIACION.

-A 55-year-old woman is recently diagnosed with amyloidosis. She is now noticing increasing SOB, fatigue, and edema. On
examination, the JVP is at 10 cm with a negative Kussmauls sign but prominent x and y descent. The blood pressure is 90/70
mm Hg, no pulsus paradoxus, pulse 100/min with low volume, and normal heart sounds.

-A 60-year-old man presents with SOB, increasing abdominal distention, and lower leg edema. He has no prior history of
cardiac, renal, or liver disease. On examination, the JVP is at 8 cm with a negative Kussmauls sign but prominent x and y
descent. The blood pressure is 95/75 mm Hg, no pulsus paradoxus, pulse 100/min with low volume, and normal heart sounds.
There is shifting dullness of the abdomen and pedal edema. His blood glucose and hemoglobin A1C are elevated.
Que medicamentos estn contraindicados en miocardiopata restrictiva?
Digitalicos y agonistas B adrenrgicos.

Cuales son los criterios para ENDOCARDITIS BACTERIANA DE DUKE.

CRITERIOS MAYORES:
1. Hemocultivos positivos para EI
1.1. Microorganismos tpicos de EI en dos hemocultivos separados
1.1.1 ESTREPTOCOCO VIRIDANS
S. BOVIS
HACEK
1.1.2. S. Aureus o Enterococus adquiridos en la comunidad en ausencia de foco primario
1.2. Hemocultivos persistentes positivos
1.2.1. Hemocultivos extraidos con ms de 12 horas de separacin
1.2.2. La totalidad de tres, o la mayora de cuatro o ms hemocultivos separados
siempre que entre el primero y el ltimo haya al menos una hora

2. Evidencia de afectacin miocrdica


2.1. Ecocardiograma positivo
2.1.1. VEGETACIN EN VLVULA O ESTRUCTURAS ADYACENTES o en el choque del jet, o
sobre dispositivos protsicos en ausencia de otra explicacin anatmica
2.1.2. Absceso
2.1.3. Nueva dehiscencia parcial de una vlvula protsica
2.2. Nueva regurgitacin valvular (incremento o cambio en un soplo preexistente no
es suficiente)
CRITERIOS MENORES
1. Predisposicin. Una cardiopata predisponente o ser ADVP.
2. FIEBRE > 38C
3. Fenmenos vasculares: mbolos en arterias mayores, infartos pulmonares, spticos, aneurismas micticos,
hemorragia intracraneal, hemorragia conjuntival y LESIONES DE JANEWAY
4. Fenmenos inmunolgicos (glomerulonefritis, NDULOS DE OSLER, MANCHAS DE ROTH Y factor
reumatoide).
5. Ecocardiograma: sugestivos de eI sin alcanzar los criterios mayores antes mencionados.
6. Evidencia microbiolgica (hemocultivos positivos que no cumplen los criterios mayores) o evidencia
serolgica de infeccin activa con un microorganismo que produce EI.
-A 28-year-old man develops viridans group streptococci septicemia. Which of the following cardiac lesions has
the highest risk of developing endocarditis?
(A) ventricular septal defect
(B) atrial septal defect, secundum type
(C) mitral valve prolapse with regurgitation
(D) pure mitral stenosis
(E) asymmetric septal hypertrophy
Respuesta correcta A

En que consiste la PERICARDITIS CONTSTRICTIVA?


R = Constrictive pericarditis is characterized by a prominent y descent of the neck veins and LOW VOLTAGE ON
ECG. THE PRESENCE OF A POSITIVE KUSSMAULS SIGN HELPS DIFFERENTIATE THE SYNDROME FROM COR
PULMONALE AND RESTRICTIVE CARDIOMYOPATHIES. Apericardial knock is characteristic of constrictive
pericarditis. It is in fact an early S3, occurring 0.060.12 seconds after aortic closure. S1 and S2 are frequently
distant.

-A 56-year-old man presents with SOB, fatigue, and edema. He has also noticed weight gain, abdominal
discomfort, and distension. He has a prior history of lung cancer treated with radiotherapy to the chest. There
is no history of liver or cardiac disease in the past. On examination, he has an elvated JVP, prominent y descent
of neck veins, and positive Kussmauls sign. The heart sounds are normal. The CXR shows a normal cardiac
silhouette and the ECG has low voltages.
-A 64-year-old presents with dyspnea and edema. He had previous coronary
bypass surgery 5 years ago, which was uncomplicated. Since then he has
had no further chest pain. On examination, his JVP is at 8 cm, with
prominent Kussmauls sign. The heart sounds are easily heard but there is
an early diastolic filling sound (pericardial knock).

Cul es el diagnostico diferencial entre miocardiopatia constrictiva y


pericarditis restrictiva?
1) PERICARDITIS RESTRICTIVA: Puede ser causada por tuberculosis, Rx del
trax y ciruga cardiaca.
2) PERICARDITIS CONSTRICTIVA: Causada por un tamponade.
3) MIOCARDIOPATIA CONSTRICTIVA: Lo causa la amiloidosis,
hemocromatosis, sarcoidiosis, esclerodermia, carcinoide.
REUMATOLOGIA

ENARM
ARTRITIS CRONICA JUVENIL
ESPONDILITIS ANQUILOSANTE
Que espondiloartropatias se asocian al HLA-B27?
R = ESPONDILITIS ALQUILOSANTE, ARTRITIS REACTIVA, ESPONDILOARTROSIS
PSORIASICA, uvetis anterior aguda.

Cual es el sntoma inicial de la espondilitis alquilosante (EA) y que hallazgos son frecuentes en la ex fis?
R = DOLOR en regiones SACROILIACAS Y LUMBARES. En la ex fis HAY DISMINUCIN DE LA
MOVILIDAD DE LA COLUMNA LUMBAR, dolor al presionar las regiones sacroiliacas y disminucin de la
expansin del trax.

Cual es la manifestacin extrarticular mas frecuente de la EA?


R = UVEITIS ANTERIOR, 2/3 de px tiene CAMBIOS EN LA MUCOSA Y SUBMUCOSA DEL COLON E LEON
TERMINAL

Cuales son los hallazgos ms comunes en imagen obtenidas en la espondilitis anquilosante?


R = Erosiones y esclerosis sacroiliacas, IMAGEN EN COLUMNA DE BAMBU

Cual es el manejo de la EA?


R = AINES, COX-2, Infliximab, Etanercept mas terapia fsica. Indometacina, INFLIXIMAB que disminuye el FNTa.
ENFERMEDAD DE REITER
Cuales son los datos clinicos ms comunes de ARTRITIS REACTIVA
o enfermedad de Reiter y con que HLA se relaciona?
R = Ligado al HLA B27, oligoartritis, CONJUNTIVITIS, URETRITIS,
CERVICITIS, ULCERAS ORALES x un CUADRO INFECCIOSO PREVIO.

Que situacin precede a una manifestacin de ARTRITIS


REACTIVA o sndrome de Reiter y cual es el patgeno > comn?
R = Alguna INFECCIN por SALMONELLA, SHIGELLA, YERSINIA,
CLAMYDIA TRACHOMATIS +++ o
CAMPYLOBACTER que PRECEDE a la MANIFESTACIN ARTICULAR
3-4 semanas despus.

Cual es el estudio de laboratorio o gabinete de eleccin del


sndrome de Reiter?
R = CULTIVO CON ANTIBIOGRAMA, se puede encontrar tambin Ac
anti bacteriano en el suero o liquido sinovial o DNA bacteriano.
ENFERMEDAD DE STILL
Que caracteriza clnicamente a la enfermedad de Still y que la precede?
R = VARIANTE DE LA ARTRITIS REUMATOIDE en la cual hay FIEBRE DE
MAS DE 40 y con precipitacin posterior varios grados debajo,
ODINOFAGIA Y POLIADENOPATIAS. LO PRECEDE
FARINGITIS.
Cual es una manifestacin clnica caracterstica de la enfermedad de Still?
R = EXANTEMA NO PRURIGINOSO, maculopapular de COLOR SALMN
en TRAX Y ABDOMEN. (EL CUAL NO HAY EN REITER)

Cuales son los laboratorios distintivos de la enfermedad de Still, que


inmunoglobulina se eleva y como afecta al FR?
R = LEUCOCITOSIS > 40, VSG AUMENTADA, hipergamaglobulinemia IgG y anemia.
NO se detectan AUTOANTICUERPOS NI FR.

Cual es el manejo de la enfermedad de Still?


R = AINES
ARTRITIS SORIASICA
Que es la artritis soriasica?
R = Ligado al HLA B27. Se define como una ARTRITIS usualmente
SERONEGATIVA que se ASOCIA A SORIASIS.

Cual es el cuadro clnico de la artritis soriasica?


R = PRECEDE PSORIASIS A ARTRITIS, ARTRITIS ASIMTRICA con
DEDOS EN APARIENCIA DE SALCHICHA (DACTILITIS) de los dedos de manos
y pies.

Como diagnosticas artritis soriasica?


R = HC, y RX con evidencia de SACROILEITIS ASIMTRICA, alteraciones de la
conformacin de hueso nuevo con IMAGEN DE PUNTA DE LPIZ EN UNA
COPA.

Cual es el manejo de la artritis soriasica?


R = AINES E INFLIXIMAB. Tratamiento de soriasis.
RESUMEN ESPONDILOARTROPATIAS
SERONEGATIVAS
RESUMEN ESPONDILOARTROPATIAS
SERONEGATIVAS
LUPUS ERITEMATOSO SISTEMICO
Que HLA tienen relacin con el LES?
1) HLA B8, HLA DR2 y DR3.
2) Los alelos de HLA estn en el brazo corto del CROMOSOMA 6.

Que frmacos inducen la produccin de anticuerpos antinucleares y LES?


R = HIDRALAZINA, procainamida, clorpromazina, ISONIAZIDA y diversos anticonvulsivantes.

Cual es el cc del LES en cada aparato sistema y con que padecimientos se relaciona?
1. Se relaciona con: PTI, ANEMIA HEMOLITICA Y TIROIDITIS.
2. Constitucionales: la FATIGA es lo mas comn
3. Mucocutaneas: 80%, LESIONES DISCOIDES observndose en cara, pabellones auriculares y el
AREA DE V DEL ESCOTE. Eritema malar o en ALAS DE MARIPOSA QUE ES FOTOSENSIBLE. EL
neonatal QUE SE PRESENTA EN LOS HIJOS DE PX CON LES POR EL PASO DE IgG
CARACTERIZNDOSE POR LESIONES CUTNEAS Y BLOQUEO CARDIACO CONGNITO EN
LOS PRIMEROS 6 MESES DE EDAD. ALOPECIA.
4. Musculoesqueletico: ARTRALGIAS con artritis. Osteonecrosis en la cabeza femoral, humeral,
cndilos femorales, carpo, tarso manifestndose clnicamente con dolor que mejora con el reposo.
5. Renal: nefritis lupica siendo la mas comn la TIPO IV GLOMERULONEFRITIS PROLIFERATIVA
DIFUSA.
6. SNC: Meningitis asptica, EVC, sndromes desmielinizantes y cefaleas.
7. Cardiopulmonar: En corazn con VEGETACIONES ENDOCARDICAS de LIBMAN-SACKS, en
pleura lo mas comn es pleuritis con o sin derrame pleural con niveles altos de protenas DHL,
complejos inmunes, ANA y clulas LE. LA VALVULA MAS AFECTADA ES LA MITRAL.
8. Gastrointestinal: Se puede presentar con SEROSITIS, disfagia y dispepsia muy a menudo.
9. Hematolgico: PANCITOPENIA Y >TTP. Anemia por deficiencia de hierro MICROCITICA-
HIPOCROMICA.
ESCLERODERMIA
SX SJOGREN
POLIARTERITIS NODOSA
Cuales son los signos y sntomas MAS COMUNES de poliarteritis nodosa?
R = DOLOR EN EXTREMIDAD POR ARTRALGIA, MIALGIA QUE AFECTA
PANTORRILLAS, puede afectar rin causando HAS, IRA O HEMORRAGIA
POR MICROANEURISMAS. MONONEURITIS MLTIPLE
(PIE CADO), corazn con IAM. Livides reticular, ndulos subcutneos, y ulceras en
piel. Son necesarios 3 de 10 criterios para su diagnostico.

Cuales son las pruebas de laboratorio en la poliarteritis nodosa y con que otro
virus se relaciona habitualmente?
R = DESCARTAR VHB QUE SE ASOCIA EN 20-30%. ANCA (-) Y ANA (-) .Anemia,
leucocitosis y trombocitosis. Se confirma diagnostico con toma de BIOPSIA DE LA
LESIN.

Que datos se obtienen al tomar la biopsia en la poliarteritis nodosa?


R = BIOPSIA DE ARTERIA de mediano o pequeo calibre con PRESENCIA DE
INFILTRADO GRANULOCITICO, PMN Y CLULAS MONONUCLEARES EN LA PARED DEL
VASO.

Cual es el manejo de la poliarteritis nodosa?


R = ESTEROIDES, pero SI SE ASOCIA CON VHB SE AGREGAN RETROVIRALES.
ARTERITIS DE LA TEMPORAL
VASCULITIS
ARTRITIS SEPTICA PIOGENA
Cuales son los factores de riesgo para la artritis sptica pigena (gonoccica o no)?
R = Abuso de drogas IV, jvenes sexualmente activos, enfermedades concomitante como DM, AR o lupus,
antecedentes de ciruga o prtesis.

Cual es el cc de la artritis sptica pigena gonoccica o no?


R = Es una entidad de inicio agudo y progresivo caracterizado por dolor e inflamacin local (salvo en el primer
ao de vida donde es similar a la sepsis). El dolor es moderado a intenso con limitacin funcional, aumento de
la temperatura local y rubor es monoarticular, cuando es poliarticular es en paciente inmunocomprometido
siendo la rodilla la mas afectada.

Cuales son los hallazgos radiolgicos en la artritis sptica pigena?


R = Edema de tejidos blandos y derrame articular. EN AFECCIN POR ANAEROBIOS SE VE GAS EN TEJIDOS
BLANDOS. El gamagrama seo con tecnecio (valora flujo) o con galio (valora actividad inflamatoria) pueden
utilizarse de manera temprana.

Cual es el manejo de la artritis sptica pigena?


1) Consiste en la administracin de antibiticos y drenaje de la articulacin afectada. En caso de ARTRITIS
PIGENA debe administrarse de 3 A 4 SEMANAS y en caso de ARTRITIS GONOCCICA DE 7 A 14 DAS.
2) En menores de 2M: DICLOXACILINA mas CEFOTAXIMA
3) En LACTANTES: DICLOXACILINA mas CEFOTAXIMA
4) En nios MAYORES DE 3: DICLOXACILINA
5) Artritis sptica GONOCCICA: 1g por via IM o IV cada 24 hrs.
6) Artritis sptica en DROGADICTOS por via IV: DICLOXACILINA mas GENTAMICINA
7) Artritis sptica en INMUNOCOMPROMETIDOS: CEFTAZIDIMA mas AMIKACINA
8) Artritis sptica POSTOPERATORIA: VANCOMICINA mas AMIKACINA
9) Artitis sptica en FRACTURA ABIERTA: AMOXICILINA con ACIDO CLAVULANICA
ARTRITIS SEPTICA PIOGENA
RESUMEN DE LAS ENFERMEDADES
METABOLICAS OSEAS
CPK
Las siguientes patologas cursan con aumento
de la CPK
1) HIPOTIROIDISMO
2) RAZA NEGRA
3) POLIMIOSITIS Y
4) DERMATOMIOSITIS
GASTROENTEROLOGIA

ENARM
ACALASIA
Que es la acalasia?
R = Es un trastorno por motilidad inadecuada que se caracteriza por una RELAJACIN INCOMPLETA y
AUMENTO DE LA PRESIN BASAL DEL EEI, adems de presentar APERISTALSIS DEL CUERPO.

Que causa la acalasia?


R = Es IDIOPTICO, aunque una de las hiptesis indica que es de origen central con DESTRUCCIN
PARCIAL DEL NCLEO DORSAL DEL VAGO o sus fibras nerviosas a nivel del esfago con la siguiente
alteracin de la funcin.

Cuales son los sntomas ms comunes de la acalasia?


R = DISFAGIA EN EL 90% DE LOS CASOS, INICIALMENTE A SLIDOS Y DESPUS A LQUIDOS. El segundo
sntoma mas frecuente es la REGURGITACIN EN 75% mas acentuada en decbito, EL DOLOR 20-
30% ES MENOS COMN, DE LOCALIZACIN EPIGSTRICA, RETROESTERNAL, E IRRADIADO HACIA
CUELLO, DORSO Y EXTREMIDADES SUPERIORES.

Cuales son los estudios de gabinete necesarios para el diagnostico de acalasia?


R = La MANOMETRIA ES DE ELECCIN la cual nos muestra una PRESIN DEL EEI AUMENTADA (>69
MM/HG), SIN PRESENCIA DE ONDAS PERISTLTICAS A LA DEGLUCIN. La ESOFAGOGRAFIA CON BARIO
MUESTRA ALTERACIONES RELACIONADAS CON LAS ONDAS PERISTLTICAS Y LA AUSENCIA DE
RELAJACIN DEL EEI DANDO UNA IMAGEN EN PICO DE PJARO O EN PUNTA DE LPIZ caracterstica.

Cual es el manejo de la acalasia?


1) MEDICO: A base de DILATACIONES HIDROSTTICAS O NEUMTICA, adems de LA INYECCIN CON
TOXINA BOTULINICA.
2) Quirrgico: MIOTOMIA DE HELLER con una tasa de curacin del 95%.
ESPASMO ESOFAGICO DIFUSO
Que es el espasmo esofgico difuso?
R = Se trata de un trastorno motor esofgico que afecta fundamentalmente a la musculatura lisa y que se caracteriza
por LA APARICIN DE ACTIVIDAD CONTRCTIL NO PROPULSIVA que alterna con episodios de peristalsis normal.

Cual es el cuadro clnico del espasmo esofgico difuso?


R = DOLOR TORAXICO CON LOCALIZACIN fundamentalmente RETROESTERNAL, aunque a veces SE IRRADIA A
CUELLO Y A LOS HOMBROS SIMULANDO UNA ENFERMEDAD CARDIACA aumentando cuando estn bajo estrs
sicolgico o emocional. La DISFAGIA es tan frecuente o mas que el dolor torcico. Tambin es intermitente SE
DESENCADENA CON FRECUENCIA CON LA TOMA DE SLIDOS O BEBIDAS FRAS.
Cuales son las alteraciones histolgicas que esperas encontrar en el espasmo esofgico difuso?
R = HIPERPLASIA DE LAS CLULAS musculares y ocasionalmente mnimas ALTERACIONES a nivel de las fibras nerviosas
terminales del PLEXO DE AUERBACH, no existiendo alteraciones significativas en las clulas ganglionares y nervios o en
todo caso estas son mnimas.

Cuales son los hallazgos al gabinete en el espasmo esofgico difuso?


R = ESOFAGOGRAMA: Que nos muestra hallazgos caractersticos con APARICIN DE FRECUENTES ONDAS NO
PROPULSIVAS en el musculo liso esofgico QUE INDENTAN LA COLUMNA DE BARIO y retrasan su evacuacin, lo
cual se describe como ESFAGO EN SACACORCHOS. MANOMETRIA: Alteraciones segmentarias, apareciendo
generalmente en los DOS TERCIOS INFERIORES DEL CUERPO ESOFGICO. PHmetria obligatoria para descartar ERGE
como causa del espasmo.

Cual es el manejo MEDICO Y QUIRURGICO del espasmo esofgico difuso?


MEDICO: NIFEDIPINO, ANTIDEPRESIVOS Y ANSIOLTICOS.
QUIRURGICO: La ESOFAGOMIOTOMIA AMPLIADA hasta el cayado artico puede ser una alternativa muy acertada EN
CASOS GRAVES REFRACTARIOS a cualquier otro tratamiento.
PERISTALSIS ESOFGICA SINTOMTICA O ESFAGO EN CASCANUECES
Que es la peristalsis esofgica sintomtica o esfago en cascanueces y con que se relaciona?
R = Trastorno de la motilidad esofgica que se caracteriza por la PRESENCIA DE CONTRACCIONES DE
GRAN AMPLITUD, SIN ALTERACIONES EN EL PERISTALTISMO del cuerpo esofgico y relajacin del
esfnter esofgico inferior. Se RELACIONA CON ERGE y a la histologa no presenta alteraciones.

Cual es el cuadro clnico de la peristalsis esofgica sintomtica o esfago en cascanueces?


R = DOLOR TORCICO RECURRENTE y con menor frecuencia disfagia.

Cuales son los hallazgos de gabinete en la peristalsis esofgica sintomtica o esfago en cascanueces
en la manometria o en el esofagograma?
R = El diagnostico se establece por la MANOMETRIA y se caracteriza por la aparicin de ONDAS DE
GRAN AMPLITUD (180 MM/HG), QUE OCASIONALMENTE TAMBIN SON DE LARGA DURACIN
(6SEG), siendo condicin indispensable que EL PERISTALTISMO ESOFGICO SEA NORMAL.
Siempre se debe descartar ERGE realizando una PHmetria. NO HALLAZGOS CON ESOFAGOGRAMA.

Cual es el manejo de la peristalsis esofgica sintomtica o esfago en cascanueces?


1) MEDICO: Si existe ERGE se recomiendo uso de OMEPRAZOL y evitar MIORRELAJANTES (nifedipino,
diltiazem o nitratos) POR QUE EMPEORARAN EL ERGE. BENZODIAZEPINAS Y SOBRE TODO LOS
ANTIDEPRESIVOS TRICICLICOS parecen tiles.
2) QUIRURGICO: ESOFAGOMIOTOMIA AMPLIADA HASTA EL CAYADO ARTICO puede ser una alternativa
en pacientes intratables. La inyeccin de toxina botulinica tiene resultados muy pobres.
EEI HIPERTENSO
Como se define el EEI hipertenso?
R = Como la existencia DE UN TONO O PRESIN media mxima basal DEL
EEI IGUAL O SUPERIOR A DOS DEVIACIONES ESTNDAR, mas de 40-45
mm/Hg, siendo adems condicionantes indispensables que existan
RELAJACIONES COMPLETAS DEL EEI y que EL PERISTALTISMO ESOFGICO
SEA NORMAL.

Cual es el cuadro clnico del EEI hipertenso?


R = DOLOR TORCICO O DISFAGIA. Mejorando generalmente el dolor tras el
tratamiento, siendo la disfagia la mas difcil de tratar.

Cual es el manejo del EEI hipertenso?


R = RELAJANTES DE LA MUSCULATURA LISA
SX DUMPING
CNCER GSTRICO
Cual es la estirpe etiolgica mas comn del cncer gstrico?
R = ADENOCARCINOMA.

Cual es el cuadro clnico de un paciente con cncer gstrico?


R = PERDIDA DE PESO, dolor abdominal, SACIEDAD TEMPRANA. Generalmente se presentan
con palidez. Entre las manifestaciones dermatolgicas mas frecuentes se encuentran la
ACANTOSIS NIGRICANS y SIGNO DE LESER-TRELAT (queratosis seborreica de aparicin
sbita)

Como se manifiesta la metastasis comnmente el cncer gstrico?


R = GANGLIOS EN SUPRACLAVICULAR (Virchow), GANGLIOS PERIUMBILICALES (Hermana
Maria Jose), estructuras rectales (signo del anaquel de blmer), OVARIOS (tumor de
Krukenberg).

Cual es el estndar de oro para el diagnostico del cncer gstrico?


R = PANENDOSCOPIA con toma de 7 BIOPSIAS de la lesin y despus TAC PARA
ESTADIFICACION en caso de metstasis.

Cual es el manejo del cncer gstrico?


R = Quirrgico. La gastrectoma total se recomienda en caso de lesiones en el estomago
proximal. La QUIMIOTERAPIA no mejora la sobrevida sobre la ciruga, sin embargo
COMBINADA CON LA RADIOTERAPIA si la mejora.
LINFOMA GASTRICO
60. Cual es la estirpe etiolgica mas comn en el linfoma gstrico?
R = No HODKING 95%

61. Cual es el cuadro clnico del linfoma gstrico?


R = Sintomatologa no especifica como DOLOR ABDOMINAL, nausea,
vomito, PERDIDA DE PESO, SUDORES NOCTURNOS y diarrea.

62. Cual es la apariencia del linfoma gstrico a la endoscopia?


R = Aparecen como LESIONES FUNGOIDES, PLIPOS, ENGROSAMIENTO DE
PLIEGUES.

63. Cual es el manejo del linfoma gstrico?


R = Quirrgico
MALABSORCION
Cuales son los datos clnicos de malabsorcin?
R = Diarrea, DISMINUCIN DE PESO y desnutricin, acompaados de
DISTENCIN ABDOMINAL, sntomas y signos secundaros a los dficit
especficos, ESTEATORREA.

Que datos de laboratorio se encuentran en un paciente con malabsorcin?


R = Hipocolesterolemia, HIPOALBUMINEMIA, alteraciones electrolticas y PH.

Cual es la posologa de la prueba con D-Xilosa en malabsorcin?


R = Se da EN AYUNAS midiendo su nivel en suero 1hr despus y la excrecin en
orina en 6hr, este azcar SE ABSORBE EN FORMA PASIVA SI LA MUCOSA
INTESTINAL ESTA INTEGRA.

Cuales son las principales causas de malabsorcin?


R = DEFICIENCIA DE LACTASA, ESPRUE celiaco, ESPRUE TROPICAL,
ENFERMEDAD DE WIPPLE, sobre crecimiento bacteriano y sndrome de
intestino corto.
ESPRUE CELIACO
Que es el esprue celiaco?
R = Causa malabsorcin debido a lesin ocasionada por el GLUTEN a nivel intestinal, el cual
es contenido en el trigo, CEBADA, CENTENO Y AVENA.

Cual es el cuadro clnico del esprue celiaco?


R = Hay malabsorcin de CHBTS, PROTENAS Y GRASAS. Perdida progresiva de peso,
distensin abdominal, flatulencia y diarrea, evacuaciones voluminosas y de mal olor.

Cuales estudios de diagnostico hay para el esprue celiaco y cuales son los anticuerpos
especificos?
1) La biopsia en LA UNIN DUODENOYUYENAL que suele REPORTAR ATROFIA O
ACORTAMIENTO DE LAS VELLOSIDADES, HIPERPLASIA DE LAS CRIPTAS y aumento en la
mitosis CON INCREMENTO DE LINFOCITOS, NEUTROFILOS, MASTOCITOS Y EOSINOFILOS EN
LAMINA PROPIA.
2) Cuantificacin de grasa en heces de 24 hrs mayor a 7g/dia , la ABSORCIN DE D-XILOSA
SUELE SER ANORMAL, los ANTICUERPOS SRICOS ANTIGLIADINA SON POSITIVOS EN 90%,
el anticuerpo IgA es el mas especifico.
3) Los anticuerpos IgA ANTIENDOMISIO Y ANTIRRETICULINA son los
MAS ESPECFICOS.

Cual es el manejo del esprue celiaco?


R = Dieta libre de gluten
ESPRUE TROPICAL
Cual es la etiologa del esprue tropical?
R = Es producido por la COLONIZACIN PERSISTENTE del intestino delgado por KLEBSIELLA
PNEUMONIAE, ENTEROBACTER CLOACAE o E. COLI.

Cual es el cuadro clnico del esprue tropical?


1) CRISIS DE DIARREA ACUOSA, acompaada de DOLOR ABDOMINAL y FLATULENCIA, los
cuales disminuyen de intensidad a la semana de evolucin y posteriormente se hacen
crnicos.
2) ANEMIA MEGALOBLASTICA.

Cuales son los estudios diagnsticos del esprue tropical?


1) La biopsia de intestino delgado reporta ALARGAMIENTO DE LAS CRIPTAS, ensanchamiento
y acortamiento de las vellosidades e INFILTRACIN DE LA LAMINA PROPIA POR CLULAS
INFLAMATORIAS.
2) Se encuentra esteatorrea, ABSORCIN DE D-XILOSA ANORMAL.

Cual es el manejo del esprue tropical?


R = TETRACICLINA y ACIDO FLICO para mejorar ANEMIA MEGALOBLASTICA.
ENFERMEDAD DE WHIPPLE
Cual es el agente causal de la enfermedad de Whipple y con que HLA se relaciona?
R = Es secundaria a la infeccin por TROPHERYMA WHIPELLII y se relaciona con HLA B27 + en 25%.

Cual es el cuadro clnico de la enfermedad de Whipple?


R = Perdida de peso, diarrea o esteatorrea, artralgias, FIEBRE, dolor abdominal y HEMORRAGIA
INTESTINAL, LAS ARTRALGIAS SON MIGRATORIAS, no destructivas y afectan las grandes
articulaciones, se presenta SACROILEITIS EN 25%. Un signo caracterstico es la MIORRITMIA
OCULOMASTICATORIA, la cual consiste en la convergencia rtmica de los ojos asociada a
contracciones sincrnicas de los msculos de la masticacin. A nivel ocular se manifiesta como
UVETIS, vitreitis, retinitis y papiledema. A la ex fis se encuentra LINFADENOPATIA
perifrica.

Cuales son los hallazgos de laboratorio en la enfermedad de Whipple?


R = La biopsia de intestino delgado muestra MACRFAGOS POSITIVOS CON LA TINCIN PAS. Sin
embargo se debe demostrar el AGENTE CAUSAL EN EL LCR, tejido cerebral o GANGLIOS LINFTICOS.
Anemia secundaria a enfermedad crnica, deficiencia de hierro, folato o VB12, asociada a neutrofilia.
Prueba de D-Xilosa anormal, LINFADENOPATIA ABDOMINAL detectada por ultrasonido.
Radiografa de TORAX CON INFILTRADOS, FIBROSIS O DERRAME PLEURAL.

Cual es el manejo de la enfermedad de Whipple y por cuanto tiempo se realiza?


R = TMP/SMZ durante 1, si hay ALERGIA A SULFAMIDAS esta indicada la PENICILINA por va oral
durante el mismo periodo y se deben tratar los trastornos nutricios.
SX COLON IRRITABLE
Cuales son los criterios de ROMA II para integrar el sndrome de colon irritable?
R = Que los sntomas hallan durado POR LO MENOS 12 SEMANAS (no necesariamente
continuas) durante los 12m previos de malestar o dolor abdominal que tiene 2 de las
siguientes caractersticas. Se dice que el sndrome de intestino irritable se diagnostica
cuando cumple al menos con 1 o mas de los criterios 2, 4 y 6 o bien 1, 3 y 5 DEPENDIENDO
DE SI ES DIARREA LA QUE PREDOMINA O CONSTIPACIN. Se exacerba con el estrs y es
MAS FRECUENTE EN MUJERES.

1. Menos de 3 evacuaciones por semana


2. Mas de 3 evacuaciones al da
3. Evacuaciones de consistencia dura
4. Evacuaciones sueltas o duras
5. Pujo al evacuar
6. Urgencia al evacuar
7. Sensacin de evacuacin completa
8. Moco durante la evacuacin
9. Distencin abdominal

Cual es el manejo del sndrome de colon irritable?


R = Debe ser INDIVIDUALIZADO a los sntomas. PInaverio con dimeticona.
POLIPOS DEL COLON
Cual es la clasificacin de los plipos neoplsicos de colon?
R = A)TUBULARES 75% que corresponde a ADENOMAS. B) VELLOSOS 10% de los adenomas, mas
relacionado con cncer colonico y C) TUBULOVELLOSOS 15% de los adenomas. Requirindose un periodo
mnimo de 10 para que un adenoma se convierta a cncer.

Cuales son las alteraciones genticas implicadas en los adenomas y el cncer de colon?
R = Mutacin del GEN RAS y lesiones de los cromosomas 5, 17 y 18.

Cual es el cuadro clnico de un paciente con SNDROME DE PEUTZ-JEGHERS?


R = Es un TRASTORNO AD caracterizado POR MANCHAS HIPERPIGMENTADAS en los labios, mucosa bucal, cara
y dedos, PRESENTANDO HAMARTROMAS EN TODO EL TUBO DIGESTIVO e INTUSUCEPCION CON STDB.

Cual es el manejo del Sndrome de Peutz-Jeghers?


R = Reseccin de plipo sintomtico por endoscopia

Cual es el cuadro clnico de la poliposis juvenil difusa?


R = Es un TRASTORNO AD caracterizado por una poblacin homognea de plipos, tanto hamartromas como
adenomas, los plipos pueden ocasionar hemorragia e intususcepcin en este sndrome. 10% RIESGO DE
DESARROLLO DE CA DE COLON.

Cual es el manejo de la poliposis juvenil difusa?


R = RESECCIN del colon con y anastomosis ileorectal. SEGUIMIENTO CON PROCTOSCOPIA CADA 6M y
extirpacin de cualquier plipo nuevo.
Cual es el cuadro clnico del SNDROME DE COWDEN?
1) Enfermedad AD caracterizada por HAMARTROMAS en todo el tubo digestivo, PAPULAS FACIALES Y
BUCALES, CRECIMIENTOS QUERATOSICOS EN MANOS Y PIES.
2) Estos pacientes pueden presentar cncer de mama, tiroides o de tero.

Cual es el cuadro clnico del SNDROME DE CRONKHITE-CANADA?


R = NO ES HEREDITARIO y se caracteriza por HAMARTROMAS INTESTINALES generalizados, ALOPECIA,
PIGMENTACIN CUTNEA y ATROFIA DE NAILS. Los SNTOMAS INCLUYEN VOMITO, DIARREA,
MALABSORCIN y enteropata con perdida de protenas, LA MAYORA DE LOS PACIENTES MUERE POCO
DESPUS DEL DIAGNOSTICO.

Cual es el cuadro clnico de la poliposis adenomatosa familiar?


R = TRASTORNO AD y su defecto se localiza en el CROMOSOMA 5. Se caracteriza por la presencia de mas de
100 POLIPOS en todo el colon y recto, 100% DESARROLLA CNCER.

Cual es el manejo de la poliposis adenomatosa familiar?


R = Reseccin con RECTOCOLECTOMIA total con ileostoma.

Cual es el cuadro clnico de la ENFERMEDAD DE GARDNER?


R = POLIPOSIS ADENOMATOSA familiar que se asocia a OSTEOMIELITIS, QUISTES EPIDERMOIDES Y
FIBROMAS DE LA PIEL.

Que es el SNDROME DE TURCOT?


R = POLIPOSIS ADENOMATOSA FAMILIAR que se asocia con NEOPLASIAS DEL SNC.

Como se manifiesta un Adenocarcinoma de colon ubicado en el lado derecho?


R = Perdida de peso, HEMATOQUZIA, anemia por dficit de hierro y UNA MASA ABDOMINAL EN EL CID.

Como se manifiesta un Adenocarcinoma de colon ubicado en el lado izquierdo?


R = Perdida de peso, cambios en hbitos intestinales, RECTORRAGIA y dolor abdominal tipo clico. Se debe
realizar una colonoscopia con biopsia.
TROMBOSIS MESENTERICA
Que pacientes tienen el riesgo de desarrollo de trombosis venosa
mesentrica?
R = Pacientes traumatizados, hipertensin portal, peritonitis o con
hipercoagubilidad

Cual es el cuadro clnico de la trombosis venosa mesentrica?


R = DOLOR ABDOMINAL DIFUSO, DISTENCIN ABDOMINAL, nauseas,
vmitos y signos de deshidratacin

Cual es el estndar de oro para el dx de la trombosis venosa mesentrica?


R = TAC EN ESPIRAL con medio de contraste oral e IV

Cual es el manejo de la TVM?


R = ANTI COAGULACIN E HIDRATACIN.
STDB
TIPS STDA Y STDB
o Los pacientes que requieran mas de 10u de paquetes globulares debern recibir
tambin PLASMA FRESCO CONGELADO, PLAQUETAS O AMBOS.

o La colocacin de BANDAS ELSTICAS es el tratamiento de eleccin en la rotura de


VARICES ESOFGICAS.

o El LAVADO NASO GSTRICO se utiliza para DIFERENCIAR entre el STDA Y STDB.

o La POSITIVIDAD EN EL ASPIRADO del tubo naso gstrico, RUIDOS INTESTINALES


HIPERACTIVOS y elevacin de los niveles del BUN apoyan el diagnostico de STDA.

o En la hematemesis secundaria a la ingesta de AINES se debe realizar una


esofagogastroscopia y administrar un inhibidor de la bomba.

o El hematoma retroperitoneal produce una IMAGEN DE VIDRIO DESPULIDO en la


radiografa de abdomen.
ABSCESO ANAL
Cuales son las causas de absceso anorrectal especificas y no especificas?
R = ESPECIFICAS: Chron, CUCI, Tb, actinomicosis, ca, leucemias. NO
ESPECIFICAS: Infeccin bacteriana de glndulas anales

Cual es la complicacin principal que sucede al drenar un absceso anal?


R = Posterior formacin de FISTULAS

Cual es el cuadro clnico en un px que presenta un absceso anal?


R = DOLOR ANAL INTENSO o una TUMORACIN ANAL MUY DOLOROSA que
apareci en forma reciente.

Cual es el manejo de un absceso perianal?


R = DRENAJE Y METRONIDAZOL O CIPROFLOXACINO.
TUMORES DEL ANO
FISTULA ANAL
Cual es la etiologa de una fistula anal?
R = Abscesos crnicos, CHRON Y CUCI.

Cual es el cuadro clnico de las fistulas anales?


R = Es caracterstico la historia de un ABSCESO PREVIAMENTE DRENADO en la
misma localizacin, DOLOR PERIANAL, la principal molestia es la PERMANENTE
SALIDA DE MATERIAL PURULENTO a travs de un orificio que MANCHA LA ROPA
INTERIOR.

Cual es el estudio de imagen utilizado para el diagnostico de las fistulas anales?


R = ULTRASONIDO RECTAL

Cual es el manejo de las fistulas anales?


R = La forma para predecir por donde drena la fistula es la regla de GOODSALL-
SALMON y una vez identificada se realiza FISTULOTOMIA.
FISURA ANAL
Cual es la etiologa de las fisuras anales?
R = ISQUEMIA EN MUCOSA POR ESFNTER HIPERTNICO.

Cual es el cuadro clnico de las fisuras anales?


R = DOLOR intenso TIPO ARDOROSO que se relaciona al inicio
de la evacuacin con una SENSACIN DE DESGARRO
al pasar el bolo fecal. SANGRADO FRECUENTE, suele ser en
POCA CANTIDAD.

Cual es el manejo de las fisuras anales?


R = Baos de asiento, laxante, RELAJANTE DE MUSCULATURA
PLVICA COMO CREMA DE ISOSORBIDE, DILTIAZEM O
NIFEDIPINO.
HEMORROIDES
HEMORROIDES
HERNIAS ABDOMINALES
De donde emerge la hernia inguinal directa?
R = Emerge DENTRO DE LOS VASOS EPIGSTRICOS originndose en el piso del
conducto inguinal

De donde emerge la hernia inguinal indirecta?


R = Por FUERA DE LOS VASOS EPIGSTRICOS y son DE ORIGEN CONGNITO siendo
mas FRECUENTES DEL LADO DERECHO.

De que manera se distingue a la ex fis una hernia inguinal directa de indirecta?


R = Una HERNIA INDIRECTA EJERCE CIERTA FUERZA CONTRA LA PUNTA DEL DEDO,
en tanto que la DIRECTA PRESIONA LA PULPA DEL DEDO. Adems cuando es
colocado el dedo en el orificio inguinal y pedirle al paciente que tosa es posible
parar la hernia indirecta y no as la directa.

Cual es la localizacin de la hernia crural?


R = DEBAJO DEL LIGAMENTO INGUINAL, medial a los vasos femorales
manifestndose CON DOLOR EN LA REGIN INGUINAL, progresivo e incapacitante.
CAUSAS DE ELEVACION DEL AST O ALT
HEPATITIS VIRAL
VHA-PICORNAVIRUS
Cual es el dato de laboratorio que orienta al VHA?
R= IgM VS VHA AGUDO E IgG CUANDO YA HABA SIDO INFECTADO EL
PACIENTE

Cual es el manejo del VHA?


R = Sintomtico, Inmunizacin pasiva como profilaxis postexposicion.

Cuales son las indicaciones de la VACUNA PASIVA del VHA?


R = PERSONAS QUE PLANEAN UN VIAJE, contactos ntimos, PERSONAL DE
GUARDERAS, ASILOS E INSTITUCIONES. NO esta INDICADA en px que tienen
DATOS CLNICOS DEL VHA

Cuales son las indicaciones de la VACUNA ACTIVA del VHA?


R = ENFERMEDAD HEPTICA CRNICA, VARONES HOMOSEXUALES,
DROGADICTOS.
VHB- HEPADNAVIRIDAE
Cual es el dato de laboratorio que te orienta a VHB?
1) HBs Ag, la persistencia de >6m sugiere infeccin crnica.
2) Anti HBs confiere inmunidad protectora ante el virus.
3) Hbe Ag es el antgeno que refiere la REPLICACIN ACTIVA del virus y la
CONTAGIOSIDAD, en px con infeccin crnica permanece constante.
4) IgM antiHBc infeccin aguda.
5) - 10% EVOLUCIONA A CRONICIDAD, BUEN PRONOSTICO. TX EN CRONICO

Cual es el manejo del VHB?


1) INTERFERON ALFA
2) LAMIVUDINA (anlogo de nucleosidos que inhibe la TRANSCRIPTASA REVERSA),
3) ADEFOVIR (anlogo de nucletidos sinttico que inhibe a la DNA polimerasa del VHB),
4) ENTECAVIR ( anlogo nucleosido deoxiguanina que inhibe la actividad del DNA
polimerasa del VHB).
VHD
Que caracteriza al VHD?
R = Es EL VIRUS MAS PEQUEO con 36 nm, NECESITA DE LA ENVOLTURA DEL VHB para
sobrevivir y su medio de CONTAGIO ES EL MISMO QUE EL DEL VHB.

Cual es el curso del VHD en coinfeccin aguda con VHB?


R = Tiende a ser LIMITADO Y ELIMINAR EL HBsAg POR LA RESPUESTA INMUNE, al mismo
tiempo que desaparece el VHD.

Cual es el curso del VHD en sperinfeccin con VHB?


R = Exposicin del VHD EN INDIVIDUO PREVIAMENTE INFECTADO POR VHB. LA
REPLICACIN DEL VHD ES MAYOR Y MS RPIDA al encontrar previamente expresado al
HBs Ag. En 70% de pacientes de los casos la infeccin se vuelve crnica y se encuentran
mayores tasas de mortalidad

Cual es el estudio de laboratorio que orienta al diagnostico del VHD?


R = IgG anti VHD o por deteccin de RNA DE VHD EN SUERO.
Cual es el manejo del VHD?
R = No hay tratamiento especifico, se ha encontrado mejora con INTERFERON ALFA.
VHE- CALICIVIRUS
FAMILIAS DE HEPATITIS VIRICA
VHA
PICORNAVIRUS
VHB
HEPADNAVIRUS
VHC
FLAVIVIRUS
VHE
CALICIVIRUS
HEPATITIS AUTOINMUNE
Que predisposicin gentica tienen las personas que padecen hepatitis autoinmune?
R = HLA-B8, HLA-DR3 Y DR-52.

Que clasificaciones tiene la hepatitis autoinmune?


TIPO I: Autoinmune clsica o Lupoide, se presenta en mujeres jvenes donde 30% presenta trastornos como
TIROIDITIS AUTOINMUNE O CUCI. A diferencia de otros grupos se presenta HIPERGAMAGLOBULINEMIA EN
SUERO 5-6 GR/DL.
TIPO II: Mas comn en Europa y los anticuerpos que distinguen a este grupo son los ANTIMICROMOSOMA DE
HGADO Y RINON TIPO 1 (ANTI LKM 1), asociada a enfermedades inmunolgicas como el VITLIGO, DM 1,
TIROIDITIS AUTOINMUNE.
TIPO III: Se encuentran anticuerpos anti antgeno heptico soluble/hgado- pncreas (anti-SLA/LP) encontrando
mas frecuentemente la TIROIDITIS AUTOINMUNE COMO ASOCIADA.

Cual es el cuadro clnico de las hepatitis autoinmunes?


R = MLTIPLES TELANGIECTASIAS, astenia, ARTRALGIAS, ESTRIAS CUTNEAS, acn, hirsutismo, AMENORREA Y
HEPATOMEGALIA.

Cuales son los CRITERIOS diagnsticos para hepatitis autoinmune?


1) MAYORES: Elevacin persistente de transaminasas, Hipergamaglobulinemia 2.5 o mayor, Ac circulantes y/o
clulas LE positivas, marcadores serolgicos para virus de hepatitis negativos.
2) MENORES: Manifestaciones sistmicas como fiebre-artralgias-erupciones cutneas, Complicaciones con
otras enfermedades autoinmunes , incluyendo las del colgeno, eritrosedimentacion elevada.
3) HISTOLOGIA: Hepatitis crnica o cirrosis con marcada infiltracin de clulas y necrosis de hepatocitos,
ausencia de hallazgos especficos para el diagnstico de otras entidades.

Cual es el manejo de la hepatitis autoinmune?


R = De eleccin son los GLUCOCORTICOIDES SOLOS O COMBINADOS CON AZATRIOPINA. La remisin se
considera cuando los px demuestran mejora clnica, niveles de transaminasas y gammaglobulinas normales en
caso de recidivas se usa tratamiento inmunosupresor de por vida.
HEPATITIS ALCOHOLICA
Cual es el cuadro clnico de un paciente con hepatitis alcohlica?
R = Perdida de peso, anorexia, ICTERICIA, FIEBRE Y
HEPATOMEGALIA DOLOROSA.

Cuales son los hallazgos de laboratorio en la hepatitis alcohlica?


1) AUMENTO DE LAS ENZIMAS HEPTICAS
2) ANEMIA
3) TROMBOCITOPENIA
4) LEUCOCITOSIS CON PREDOMINIO DE NEUTROFILOS Y
5) SE CONFIRMA EL DIAGNOSTICO POR BIOPSIA.

Cual es el manejo de la hepatitis alcohlica?


R = Abstinencia, CORTICOIDES y plan nutricio.
CIRROSIS HEPATICA
Que parmetros definen la presin portal?
R = Flujo venoso portal y la resistencia interior del hgado

Cuales son las causas mas frecuentes de cirrosis heptica?


R = Alcohol, HEPATITIS B Y C, METOTREXATO, alfametildopa e
hidralazina.

Cual es la clasificacin morfolgica de la cirrosis?


1) CIRROSIS MICRONODULAR: Ndulos menores de 3 mm.
2) CIRROSIS MACRONODULAR: Ndulos de mas de 3 mm.
3) CIRROSIS MIXTA: Mircro y macronodulares que ocurre comnmente por
alcohol.

Cual es el marcador mas til para el diagnostico de cirrosis biliar primaria?


R = ANTICUERPOS ANTIMITOCONDRIALES.
Cual es el cuadro clnico de la cirrosis?
R = Asintomatico o los siguientes:
A. SINTOMAS CONSTITUCIONALES:
B. Astenia, anorexia, perdida de peso muscular.
C. ASCITIS:
D. Por retencin de sodio y agua.
E. SIGNOS CUTANEOS:
F. Ictericia, palidez por la anemia. Telangiectacias cutneas y eritema palmar (que predomina sobre las eminencias tenar e
hipotenar y en las llemas de los dedos) son datos bastantes caracteristicos.
G. CIRCULACION COLATERAL:
H. Los sitios mas comunes de aparicin con significado patolgico son el esfago y el fondo gstrico.
I. ALIENTO:
J. Olor dulzn debido a la exhalacin de mercaptanos, productos derivados de la metionina por defecto de su metilacin.
K. ALTERACIONES ENDOCRINAS:
L. Varones con signos de feminizacin como ginecomastia y cambios en la distribucin del vello coroporal. El hipogonadismo en
varones produce atrofia testicular, perdida de la libido, impotencia. En ambos sexos el vello axilar es escaso. En las mujeres es
frecuente la oligomenorrea, amenorrea y esterilidad.
M. ALTERACIONES HEMATOLOGICAS:
N. Alteraciones en la coagulacin dado que todos los factores se sintetizan en hgado menos el factor de von Willebrand.
O. ALTERACIONES PULMONARES:
P. Ascitis torcica o derrame pleural
Q. ALTERACIONES DIGESTIVAS:
R. Hemorragia digestiva, colelitiasis.
S. ALTERACIONES RENALES:
T. Retencin de sodio con ascitis
U. SX HEPATORRENAL:
V. Se trata de una insuficiencia renal funcional como respuesta a mecanismos de descompensacin tales como el uso exagerado
de diurticos, AINES, paracentesis. La histologa es normal. Este sndrome se caracteriza por una insuficiencia renal funcional
con funcin tubular normal en un px con insuficiencia heptica.
W. INFECCIONES:
X. IVU mas comunes.
Y. ALTERACIONES NEUROLOGICAS:
Z. Encefalopatia heptica donde la primer manifestacin en aparecer son los cambios en el comportamiento. El GABA es el
principal neurotransmisor inhibitorio del cerebro, se ha postulado que el GABA producido a nivel intestinal escapa del
metabolismo heptico, atraviesa la barrera hematoencefalica y penetra al cerebro desencadenando la encefalopata heptica.
184. Cual es el manejo de la cirrosis heptica?
R = Dieta con contenido de protenas 1 a 1.2 g/gr. Prohibicion del consumo de
alcohol. Hay tratamientos utiles especficos de cirrosis, como las flebotomas en la
hemocromatosis, la d-penicilamina en la enfermedad de Wilson, los corticoides en
las cirrosis autoinmunes, los antivricos para cirrosis por VHB y VHC y el acido
urodesoxicolico para la cirrosis biliar primaria. La ciruga se asocia a una mortalidad
operatoria del 30% pero las indicaciones para la ciruga son rotura de varices o
sangrados. A las varices tipo 2 y 3 se administra betabloqueador en caso que no
exista contraindicacin. Seguimiento ecogrfico cada 6m para detectar
hepatocarcinoma.

185. Cual es el manejo de emergencia en la cirrosis heptica?


R = En caso de sangrado alto una endoscopia debe realizarse para dar
escleroterapia. Uso de vasopresina por via IV teniendo cuidado de la necrosis de la
piel. Anastomosis portosistemica intraheptica por via transyugular TIPS para
controlar la hemorragia digestica por varices esofgicas irreductible por otros
mtodos.
HEPATOCARCINOMA
Cuales son los factores de riesgo para desarrollar un hepatocarcinoma?
R = Aparece en personas que ya tienen alguna enfermedad heptica como
HEPATITIS POR VIRUS B O C, CIRROSIS.

Cual es el cuadro clnico de un px con hepatocarcinoma?


R = Puede cursar asintomtico. Cuando existen manifestaciones lo mas habitual es
que se presenten sntomas inespecficos como baja de peso, dolor abdominal,
saciedad precoz o masa palpable.
Cuales son los estudios de gabinete utilizados para diagnosticar hepatocarcinoma?
R = TAC DINMICA HELICOIDAL MULTICORTE DE ELECCIN. Us de seguimiento. La
ALFAFETOPROTEINA es el marcador tumoral mas utilizado para el dx y seguimiento
(puede elevarse en embarazo y tumores testiculares). Confirmacin por biopsia.

Cual es el manejo del hepatocarcinoma?


R = RESECCIN del tumor. INYECCIN DE ETANOL en tumor lo reseca y es fcil de
realizar. QUIMIOEMBOLIZACION que frecuentemente se complica con el llamado
sndrome postembolizacion caracterizado por fiebre, elevacin enzimtica, dolor
abdominal y nauseas. TRANSPLANTE HEPTICO.
ABSCESO HEPATICO AMEBIANO
VESICULA BILIAR
Que frmacos pueden obliterar el esfnter de Oddi?
R = Morfina y meperidina

Que elementos componen el sndrome de Mirizzi?


R = COMPRESIN de los conductos biliares debida a INFLAMACIN intensa por colecistitis
secundaria a la IMPACTACIN de un lito en el infundbulo de la vescula.

Cual es el estndar de oro para el diagnostico de coledocolitiasis?


R = CPRE

Cual es el cuadro clnico de la COLANGITIS?


R = TRIADA DE CHARCOT que se compone de FIEBRE, DOLOR EN CUADRANTE DERECHO E
ICTERICIA. Cuando se le agregan cambios en el estado mental y choque se conoce como
PENTADA DE REYNOLDS.

Cual es el estndar de oro para diagnostico de colangitis?


R = CPRE. La presencia de leucocitosis, elevacin de las bilirrubinas, FA y transaminasas
apoyan el diagnostico.
HEMOCROMATOSIS
Que tipo de trastorno es la hemocromatosis y que intercambio de aa se involucran?
1) AR. Se produce por sustitucin de CISTINA POR TIROSINA en la posicin
282 (C282Y).
2) Trastorno de almacenamiento de hierro en el que el incremento inapropiado de la
absorcin intestinal. Rara vez se presenta antes de los 20.

Cual es el cuadro clnico de la hemocromatosis?


R = Los sntomas iniciales consisten en debilidad, cansancio, perdida de peso, cambio de
coloracin de la piel, dolor abdominal, perdida de la libido y sntomas de DM. La
HEPATOMEGALIA 95% , HIPERPIGMENTACION 90 %(color gris pizarra o
metlico/ bronceado), POLIARTRITIS 20-50%, ICC 10%, angiomas en araa,
esplenomegalia, artropata, ascitis, arritmias cardiacas, HIPOGONADISMO con perdida
de vello, atrofia testicular y la ictericia son signos de enfermedad avanzada.

Como diagnosticas hemocromatosis?


R = 1) Medicin de hierro srico y porcentaje de TRANSFERRINA > 50% EN AYUNO 2)
Medicin de concentracin de ferritina 3) Biopsia heptica

Cual es el manejo de la hemocromatosis?


R = FLEBOTOMIAS de 500 ml una o 2 veces a la semana. DEFEROXAMINA cuando la anemia
o hipoproteinemia son muy graves.
ENFERMEDAD DE WILSON
Que causa la enfermedad de Wilson?
R = AR. Causado por mutaciones en el gen ATP 7B, cuyo producto es una ATPasa transportadora
de cobre ligada a membrana.

Cual es el cuadro clnico de la enfermedad de Wilson?


R = MANIFESTACIONES NEUROLGICAS manifestadas principalmente con trastornos del
movimiento como DISTONIA, INCOORDINACIN Y TEMBLORES. La disfagia y la disartria son
comunes. Anillos de KAYSER-FLEISCHER.

Cuales son las pruebas diagnosticas para la enfermedad de Wilson?


R = Niveles sricos de CERULOPLASMINA BAJOS. Anillos de Kayser-Fleischer 99 %. Biopsia
heptica con anlisis cuantitativos de cobre con valores >3. 1mol/g.

Cual es el manejo de la enfermedad de Wilson?


1) PENICILAMINA + PIRIDOXINA.
2) La TRIENTINA ha sustituido a la penicilamina.
3) ZINC es menos toxico.

Cual es el pronostico de la enfermedad de Wilson?


R = BUENO, los sntomas neurolgicos desaparecen una vez iniciado el tratamiento.
PANCREATITIS AGUDA
Cual es el cuadro clnico de la pancreatitis aguda?
R = Aparicin sbita de un dolor en la regin abdominal superior acompaado de nausea y vomito. El paciente
reporta el dolor en el epigastrio y en la regin periumbilical, con irradiacin hacia espalda, trax, flancos y
abdomen inferior. Tomando posicin antialgica. A la EX FIS: Fiebre, hipotensin, rigidez abdominal, taquipnea y
distencin abdominal. Es posible encontrar los signos de CULLEN (hematoma periumbilical) o de GREY-
TURNER (hematoma en los flancos) relacionados con hemorragias retroperitoneales.
Cuales son los hallazgos de laboratorio en la pancreatitis aguda?
1) Elevacin de la amilasa pancretica.
2) LIPASA PANCRETICA QUE SE ELEVA LAS PRIMERAS 4-8 HRS.
3) Niveles de AMILASA, TAC Y US TE ORIENTAN AL DIAGNOSTICO. En la pancreatitis aguda grave disminuye el
calcio.

Cual es el hallazgo radiolgico sugestivo de pancreatitis aguda?


R = ASA EN CENTINELA.

Cuales marcadores de gravedad de pancreatitis aguda hay?


R = Balthazar, Ranson y APACHE II.

Cuales son los manejos de la pancreatitis aguda?


1) TRATAMIENTO MEDICO: La pancreatitis leve se debe tratar con HIDRATACIN AGRESIVA hasta que el
paciente muestre balances hdricos positivos. El control del dolor se puede realizar con MORFINA. Nutricin
enteral. En los casos graves, se recomienda ELIMINAR LA INGESTIN POR VIA ORAL, sin descuidar la nutricin
y estado catablico del paciente.
2) TX QUIRURGICO: En necrosis infectada
Los pacientes que presentan hasta dos de las caracteristicas previamente
mencionadas tienen una mortalidad minima. Aquellos con tres a cinco de
estos factores, presentaran una mortalidad 10 y 20% y los que tengan mas
de 5 presentan 50% de mortalidad.
PANCREATITIS CRONICA
Cuales son las causas de pancreatitis crnica?
R = CONSUMO EXCESIVO DE ALCOHOL, enfermedades
autoinmunes, hipertrigliceridemia, hiperparatiroidismo,
pncreas divisum y FIBROSIS QUSTICA.

Cual es el cuadro clnico de la pancreatitis crnica?


R = ATAQUES INTERMITENTES de dolor intenso, de
localizacin a nivel abdominal superior con irradiacin en
forma de banda hacia la espalda.

Cuales son los estudios de imagen utilizados para el


diagnostico de pancreatitis crnica?
R = TAC, US y CPRE.
CANCER DE PANCREAS
Cuales son los factores de riesgo para desarrollar cncer de pncreas?
R = Tabaquismo, diabetes de larga evolucin, pancreatitis crnica y NEM 1.

Cual es el cuadro clnico del cncer de pncreas?


R = Perdida de peso, fatiga, dolor abdominal epigstrico transfictivo,
ictericia, tromboflebitis migratoria, datos de hipertensin portal
(hepatomegalia y ascitis) y VESCULA PALPABLE (signo de Courvoisier-
Terrier).

Como diagnosticas cncer de pncreas?


R = TAC. Elevacin FA y bilirrubina en sangre. Marcador tumoral CA 19-9.

Cual es el manejo del cncer de pncreas?


R = Quirrgico. Quimioterapia.
MANEJO DEL PX POLITRAUMATIZADO
TRAUMATISMO TORAXICO
NEUMOTORAX Y HEMOTORAX
TRAUMATICO
TRAUMATISMOS ABDOMINALES
TRAUMATISMOS DEL
APARATO
GENITOURINARIO
SHOCK HIPOVOLMICO
Cundo un paciente est
en shock hipovolmico?
Prdida estimada de sangre en un hombre de 70 Kg
CLASE I CLASE II CLASE III CLASE IV

Prdida de
< 750 750 - 1500 1500 - 2000 > 2000
sangre (ml)
Prdida de
< 15 % 15 30 % 30 40 % > 40 %
sangre (%)
Pulso < 100 > 100 > 120 > 140
Presin arterial Normal. Normal. Disminuida. Disminuida

Presin pulso Normal Disminuida Disminuida Disminuida

Frecuencia
14 - 20 20 - 30 30 - 40 > 35
respiratoria
Diuresis (ml/h) > 30 20 - 30 5 - 15 Mnima

Conciencia Levemente Moderada Ansioso, Confuso,


ansioso ansioso. confuso. letrgico.
NEFROLOGIA

ENARM
ANION GAP
El Anin GAP es una ECUACIN que sirve PARA CALCULAR ANIONES ORGNICOS que estn presentes a
una CONCENTRACIN MUY PEQUEA como para ser medidas en un ionograma...
Partiendo desde el concepto de electro neutralidad, pareciera que los cationes "superan" a los aniones,
pero justamente la "brecha" entre ellos esta representados por estos Aniones...

La utilidad fisiolgica/clnica del Anin Gap, es para orientarte acerca de la POSIBLE ETIOLOGA ( causa)
DE LA ACIDOSIS METABOLICA ( y SOLO SIRVE PARA ESTO) que el paciente esta cursando...

Las AC METAB. pueden ser DE 2 "TIPOS"... por FALTA DE BASE ( el tipo tiene una DIARREA tremenda, esta
largando hasta la 1 papilla... ), o por EXCESO DE ACIDO ( esta TOMANDO ASPIRINA como si fueran tic
tacs... o CETOACIDOSIS etc.) ( esos son algunos ejemplos... )...

Si calculas el AG... SIENDO AG= [NA+]P - ([CL-]P+[HCO3-]P)

Los valores NORMALES: AG= 12 +- 2

En caso de que el Anion Gap de AUMENTADO (+ de 14) Entonces estamos viendo una ACIDOSIS
METABOLICA POR EXCESO DE ACIDO...

Si el AG DA NORMAL, entonces, el paciente tiene un AC METAB. POR FALTA DE BASE...


Este tipo de ac metab, tambin es conocida por ACIDOSIS HIPERCLOREMICA
Cuales son las causas de la hipokalemia?
POTASIO
R = Incremento en el recambio celular, disminucin de la ingesta o un aumento de su excrecin

Que medida de laboratorio es til para el diagnostico de hipokalemia de origen renal o extrarenal?
R = EL GRADIENTE TRANSTUBULAR DE POTASIO (TTKG), el cual si es < 2 TRADUCE HIPOKALEMIA NO RENAL y UNO >
10 ES DEBIDO A PERDIDAS RENALES. EL K URINARIO <25 MEQ/DIA es NO RENAL y K URINARIO > 30
MEQ/DIA traduce perdida de ORIGEN RENAL.

Cuales son lo sntomas de hipokalemia?


R = DEBILIDAD MUSCULAR, fatiga, CALAMBRES. <2.5 Meq parlisis flcida, HIPORREFLEXIA, hipercapnia y rabdomiolisis.

Que elementos participan en la captacin de potasio?


R = La insulina en presencia de glucosa, estimulacin B adrenrgica

Cual es el regulador de potasio mas importante?


R = ALDOSTERONA facilita la excrecin urinaria de K en los TCD.

Cuales son las causas de la hiperkalemia?


R = > 5.5 mEq. DISMINUCION DE LA EXCRESION: HIPOALDOSTERONISMO, IRA o IRC, ESPIRONOLACTONA,
IECA, B bloqueador. Desplazamiento de K al interior: Rabdomiolisis, hemolisis y ACIDOSIS METABOLICA

Cual es el cuadro clnico de hiperkalemia y EKG?


R = DEBILIDAD MUSCULAR, distencin abdominal, DIARREA. EKG con onda T picuda, QT corto y depresin de onda T.

Cual es el manejo de la hiperkalemia?


1) INSULINA + GLUCOSA AL 10%, SALBUTAMOL inhalado, GLUCONATO DE CALCIO IV, HCO3 que produce ingreso a la
celula de K.
2) EL TRATAMIENTO AGUDO CON GLUCONATO DE CALCIO DEBE RESERVARSE cuando se presentan manifestaciones
cardiacas de toxicidad celular por exceso de potasio (calcitosis).
NTA
A que padecimiento se le conoce como NTA?
R = IRA por lesiones tubulares siendo las principales causas la isqumica y por
toxinas

Que nefrotoxinas endgenas ocasionan NTA?


R = Productos contenedores de HEM, ACIDO RICO, MIOGLOBINURIA por
RABDOMIOLISIS

Cuales son los datos de lab de NTA?


R = ORINA COLOR CAF LODOSO por Hb y cilindros epiteliales de tbulos renales

Cual es el manejo de la NTA?


R = Se debe prevenir hipercalcemia, FUROSEMIDE, RESTRICCIN DE PROTENAS
DIETTICAS Y DILISIS EN PACIENTE CRITICO

Cual es la complicacin mas comn y mas temida de la NTA?


Falla cardiaca
NEFRITIS INSTERSTICIAL AGUDA
Que es la nefritis intersticial aguda?
R = Respuesta inflamatoria intersticial con edema y posible dao celular

Cuales son las causas de nefritis intersticial?


R = FRMACOS 70%. Infecciosas e inmunitarias

Cuales son los datos clnicos de nefritis intersticial aguda?

DETERIORO ABRUPTO DE LA FUNCIN RENAL


R = Se caracteriza por un
acompaado de poliuria, Nicturia, ACIDOSIS METABOLICA y glucosuria. FIEBRE 90%,
ARTRALGIAS 80%, EXANTEMA 20%. Bh: EOSINOFILIA EN FASE AGUDA. EGO con
EOSINOFILURIA, hematuria, leucocituria y cilindros leucocitarios.
Cuales son los hallazgos en la biopsia con nefritis intersticial aguda?
R = Infiltracin de clulas inflamatorias en intersticio renal con edema, predominan mononucleares y LT,
GRANULOMAS NO CASEIFICANTES.

Cual es el manejo de la nefritis intersticial aguda?


R = RETIRAR EL FACTOR predisponente. Puede llegar a REQUERIR DILISIS, MEDIDAS DE SOSTEN,
METILPREDNISOLONA en NI POR FRMACOS
Que caracteriza a la necrosis intersticial crnica?
1. Ausencia de proteinuria e hipoalbuminemia
2. Piuria estril y leucocitosis mas que hematuria
3. Poliuria y Nicturia
4. Otros defectos tubulares como ATR y osteomalacia.
IRC
Cual es la clasificacin de la IRC?
ESTADIO FG
I 90
II 60 89
III 30 59
IV < 15 O DIALISIS

Cual es el manejo restrictivo de IRC?


R = Restriccin de protenas, restriccin de Na y H2 O, restriccin de Mg
entre otros.

Que pasa con la capacidad de dilucin y de concentracin en la IRC?


La capacidad de DILUCIN se deteriora antes que la de CONCENTRACIN.
CRIOGLOBULINEMIA
Que caracteriza a la crioglobulinemia y con que enfermedades se relaciona?
R = Es una vasculitis de pequeos vasos DEBIDO A INMUNOGLOBULINAS que se
PRECIPITAN CON EL FRIO relacionndose mucho con el VHC +++ mas que con el
VHB, endocarditis o LES.

Cual es el cuadro clnico de la crioglobulinemia?


R = PURPURA 90 %, ARTRITIS 80% , Neutropenia 70 %, DANO RENAL 50% (de este
el 80% se manifiesta como GLOMERULONEFRITIS
MEMBRANOPROLIFERATIVA Y TROMBOS EN EL INTERIOR DE LOS
CAPILARES GLOMERULARES.

Cual es el manejo de la crioglobulinemia?


1) Proteinuria y dao renal leve dndose TRATAMIENTO SINTOMTICO Y ESPECIFICO
2) PARA VHC SI ES QUE ESTA ASOCIADO con origen se da INTERFERON Y RIVABIRINA,
3) Si la PROTEINURIA se encuentra en RANGO NEFRTICO se da
METILPREDNISOLONA, PLASMAFERESIS O RITUXIMAB.
NEFROPATA POR IgA O SX DE
BERGUER
Que caracteriza a la nefropata por IgA o sx de Berguer?

R = Se deposita IgA en el mesangio glomerular, observndose la MISMA LESIN


EN PURPURA DE HENOCH- SCHONLEIN
Cuales son los signos de nefropata por IgA?
R = HEMATURIA SINFARINGITICA con orina color COCA-COLA 100%. IVRS 50%, GI 10%

Que datos se encuentran en la biopsia en la nefropata por IgA ?


R = DEPOSITOS DE IgA acompaados de C3 E IgG.

Cual es el manejo de la nefropata por IgA?


1) IECAS, ESTEROIDES.
2) En caso de proteinuria se combina CICLOFOSFAMIDA CON AZATRIOPINA.

Cual es el pronstico de nefropata por IgA?


R = BUENO, pero es MALO para los que desarrollan SEMILUNAS EN LA BIOPSIA
GN PAUCINMUNITARIA
En que enfermedad se presenta GN paucinmunitaria?
1) GRANULOMATOSIS DE WEGENER,
2) ENFERMEDAD DE CHURG- STRAUSS,
3) POLIANGEITIS MICROCITICA.
Que patognesis se encuentra hubicada en 80% de las glomerulonefritis
paucinmunitarias?
R = Asociada a ANCA

Cuales son los datos de laboratorio de la GN paucinmunitaria?


R = Patron citoplasmtico c-ANCA, patrn perinuclear p-ANCA.

Cual es el manejo de la GN paucinmunitaria?


R = ESTEROIDES, CICLOFOSFAMIDA.
AFECCION RENAL X ENF SISTEMICA
NEFROPATIA LUPICA
NEFROPATIA LUPICA
NEFROPATA MEMBRANOSA
Que es la nefropata membranosa?
R = Enfermedad glomerular por DEPOSITO DE IgG
y complemento constituyendo la forma mas COMN DE
SNDROME NEFRTICO EN ADULTOS, es idioptico 70% y el resto se debe a virus, neoplasias, drogas o
enfermedades autoinmunes

Cual es el cuadro clnico de la nefropata membranosa?


R = SNDROME NEFRTICO.

Que etiologa tiene la NM secundaria?


R = LES, VHC Y ENDOCARDITIS son las mas comunes. Hepatitis B, sfilis, cncer, penicilamina y captopril.

Cuales son los signos y sntomas de NM secundaria?


R = Relacionados a SX NEFRTICO

Que revela la biopsia e inmunohistoquimica en la GMN?


R = HISTOLOGIA: PROLIFERACIN EN FORMA DE PARCHES, infiltrado leucocitario intraglomerular y necrosis
intracapilar. INMUNOHISTOQUIMICA: IgG siempre presente, IgM e IgA.

Cual es el manejo de la nefropata membranosa?


R = PROTEINURIA < 4 GR DAS IECAS CON META DE TA 125/75. PROTEINURIA > 4 GR y < 8 GR SIN
DETERIORO DE LA FUNCIN RENAL EL TRATAMIENTO SE ENCAMINA A DISMINUIR LA PROTEINURIA CON IECAS Y
OBSERVAR POR 6M.
GLOMERULONEFRITIS FOCAL Y
SEGMENTARIA
Cual es la etiologa de la Glomerulonefritis focal y segmentaria?
R = Primaria (idioptica) o secundaria debido a otras patologas como agenesia renal, ENFERMEDAD DE
CLULAS FALCIFORMES o por VIH.

Dentro de las diferentes variedades de Glomerulonefritis focal y segmentaria cuales producen mas
proteinuria?
R = LA VARIEDAD CELULAR Y COLAPSANTE en el 80% de los casos de sndrome nefrtico

Cual es el cuadro clnico de la Glomerulonefritis focal y segmentaria?


R = PROTEINURIA asintomtica o sndrome nefrtico completo

Cuales son los datos de laboratorio y biopsia de la Glomerulonefritis focal y segmentaria?


R = La PROTEINURIA puede oscilar DESDE <1 GR HASTA 20-30 GR y en la biopsia los hallazgos de
HISTOLOGA COMO SON LA VARIEDAD CELULAR Y COLAPSANTE.

Cual es el manejo de la Glomerulonefritis focal y segmentaria?


1) LOS QUE TENGAN PROTEINURIA SUBNEFROTICA 2-3 GR/ no se les dar tratamiento inmunosupresor, siendo
el principal tratamiento el control de la TA CON META DE 130/80, UTILIZANDO IECAS, hipolipemiantes,
DIURTICOS EN CASO DE EDEMA.
2) En los pacientes que se presentan con SNDROME NEFRTICO SE USA PREDNISONA, CONTROL DE LA TA
CON META DE 125/75, ANTICOAGULANTES (>10 GR/DIA) y en caso de RESISTENCIA SE USARA
CICLOSPORINA.

Cual es el pronostico de la Glomerulonefritis focal y segmentaria?


R = MALO SI LA PROTEINURIA ES > 10 GR/DIA
LIDDLE, GITELMAN Y BARTTER
Que es la enfermedad de Liddle?
R = AD. El sndrome de Liddle (o SEUDOHIPERALDOSTERONISMO) es un trastorno AD caracterizado por
HIPERTENSIN SENSIBLE A SAL con expansin de volumen, HIPOKALEMIA y ALCALOSIS
METABOLICA, con ACTIVIDAD DE RENINA Y ALDOSTERONA EXTREMADAMENTE BAJAS y FG
NORMAL.

Que es el sndrome de Gitelman?


R = Se caracteriza por HIPOKALEMIA e HIPOMAGNESEMIA, adems de contar con HIPOCALCIURIA,
perdida de sal y alcalosis metablica con HIPERALDOSTERONISMO, HIPERRENINEMIA, asi como
HIPERTROFIA E HIPERPLASIA DEL APARATO YUXTAGLOMERULAR.

Cual es el cuadro clnico del sndrome de Gitelman?


R = Debilidad muscular y DERMATITIS inespecfica. ESPASMOS DISTALES ocurren EN PERIODOS DE
FIEBRE, vomito o diarrea. Algunos sufren condrocalcinosis.

Que es el sndrome de Bartter?


R = AR. Se caracteriza por HIPOKALEMIA GRAVE, perdida de sal y alcalosis metablica con
HIPERALDOSTERONISMO, HIPERRENINEMIA, as como HIPERTROFIA E HIPERPLASIA DEL APARATO
YUXTAGLOMERULAR. Casi TODOS TIENEN NEFROCALCINOSIS por la
hipercalciuria.

Cual es el cuadro clnico del sndrome de Bartter?


R = Tipo I y II: Las manifestaciones tempranas incluyen POLIHIDRAMNIOS, retardo en el crecimiento,
polidipsia, deshidratacin, avidez por sal y debilidad muscular. En el tipo III: Se manifiesta en la
INFANCIA similar al Sx de Gitelman. El tipo IV: TODO LO ANTERIOR MAS SORDERA SENSORINEURAL E
IRC TEMPRANA.
SNDROME DE FANCONI
Que es el sndrome de Fanconi?
R = Se debe a la DISFUNCIN GLOBAL DEL TBULO PROXIMAL.

Cuantos tipos de Sx de Fanconi hay y que los caracteriza?


1) HEREDITARIO: Cistinosis, galactosemia, tirosinemia.
2) ADQUIRIDO: INTOXICACIN POR PLOMO O CADMIO, TETRACICLINAS
CADUCADAS, tolueno (inhalar pegamento), agentes quimioteraputicos
como el cisplatino.
3) FANCONI LIKE: Pacientes adultos con disproteinemias como amiloidosis,
en enfermedad de cadenas ligeras y mieloma mltiple.

Cual es la anormalidad metablica mas frecuente en el sndrome de


Fanconi?
R = ACIDOSIS METABOLICA HIPERCLOREMICA.
ATR
Cuales son las tubulopatias asociadas a alcalosis metablica?
R = Sndrome de CHEEK-PERRY(GITLEMAN), LIDDLE Y BARTTER

Que es la acidosis tubular renal o ATR?


R = Grupo de entidades patolgicas caracterizadas por defectos de transporte, ya sea
REABSORCIN DE BICARBONATO (HCO3), EXCRECIN DE HIDROGENIONES O AMBAS.

Donde se lleva a cabo la reabsorcin de HCO3 y la excrecin de H+?


R = HCO3 EN EL TCP Y H+ EN TCD.

Que caracteriza a la ATR tipo I DISTAL O CLSICA?


R = ACIDOSIS METABLICA CON BRECHA ANIONICA NORMAL O HIPERCLOREMICA, HIPOKALEMIA
E IMPOSIBILIDAD DE ACIDIFICAR LA ORINA A < 5.5 bajo el estimulo de acidosis metablica
intensa.

Cuales son las bases fisiopatolgicas de la ATR I?


R = 1) Defecto secretor (DEFICIENCIA EN LA SECRECIN DE HIDROGENIONES), formas dominantes,
recesivas y asociadas a sndrome de Sjogren, LES, AR. 2) defecto de gradiente, en el cual hay un
reflujo de los hidrogeniones normalmente secretados a nivel distal (anfotericina B) y 3)
Incapacidad de generar o mantener una diferencia luminal negativa transepitelial distal como en la
uropatia obstructiva, enfermedad de clulas falciformes, hiperplasia adrenal perdedora de sal.

Como se diferencia la ATR tipo I de la tipo II?


R = Que estos pacientes a menudo PRESENTARAN NEFROLITIASIS Y NEFROCALCINOSIS.
Como se maneja la ATR tipo I?
R = ADMINISTRACIN DE LCALI, SE PREFIERE CITRATO DE POTASIO
Que caracteriza a la ATR TIPO II O PROXIMAL?
R = Puede ocurrir sola o acompaada de otros defectos tubulares como
sndrome de Fanconi y se caracteriza por una DISMINUCIN EN EL UMBRAL
RENAL PARA EL HCO3, el cual usualmente es de 22 mml/L en infantes y 26
mmol/L en adultos.

Que caracteriza a la ATR tipo IV hiperkalemica?


R = Usualmente ocurre con la presencia de IR MODERADA, sin embargo la
magnitud de la HIPERKALEMIA y de la acidosis es desproporcionadamente
extensa para el nivel de IR.

Cual es la fisiopatologa de la ATR tipo IV o hiperkalemica?


R = Defecto en la AMONIOGENESIS

Cual es la principal enfermedad relacionada con la ATR tipo IV o


hiperkalemia?
R = DM, adems del uso de AINES, IECAS y ciclosporina.
SINDROME UREMICO HEMOLITICO

En el SHE es comn observar la prolongacin del tiempo de?


R = Sangra
INFECTOLOGIA

ENARM
MENINGITIS
Cual es la presin de apertura a la toma de LCR en meningitis bacteriana?
R = Normal <180 o elevada > 180, siendo comn 200-500

Cuales son los datos del LCR de la meningitis bacteriana aguda?


I. Presin de abertura: Normal o > 180 siendo comn observar valores entre 2
500
II. Apariencia: Leucocitos y bacterias turban el LCR
III. Glucorraquia: Concentracin < 40 (NORMAL > 45)
IV. Proteinorraquia: Valores arriba de 50 (NORMAL 40)

Cual es el tratamiento de eleccin en los contactos de pacientes con enfermedad


meningocococica?
R = RIFAMPICINA

Que constituye el sndrome de austrian?


R = MENINGITIS, ENDOCARDITIS y NEUMONA por estreptococo pneumoniae
en PACIENTE ALCOHLICOS
MENINGITIS
Tratamiento de eleccin para meningitis por NEUMOCOCO?
R = CEFTRIAXONA

Tratamiento de eleccin para meningitis por S. AUREUS METICILINO SENSIBLE?


R = Nafcilina, DICLOXACILINA

Tratamiento de eleccin para meningitis por S. AUREUS METICILINO RESISTENTE?


R = VANCOMICINA o TEICOPLANINA

Tratamiento de eleccin para meningitis por LISTERIA?


R = AMPICILINA

Tratamiento de eleccin para meningitis por H. INFLUENZA?


R = CEFTRIAXONA o cefotaxima

Tratamiento de eleccin para meningitis por ANAEROBIOS?


R = METRONIDAZOL

Cual es el tratamiento de eleccin en la meningitis CRIPTOCOCOCICA?


R = ANFOTERICINA B + 5-fluocitosina
MENINGITIS
Caracterstica de meningoencefalitis subaguda?
Meningoencefelatis por tuberculosis

Agente etiolgicos en meningitis en recin nacidos?


E.COLI (en el trabajo de parto) y estreptococos del grupo B (AGALACTIAE)

Agente etiolgico de meningitis en adultos?


1.Streptococo pneumoniae 2.Neisseria Meningitidis 3.Streptococo del grupo B

CASO CLNICO: Recin nacido, con fontanela abombadas, petequias en la cara anterior
del trax, esta en cunero y alado entro por deshidratacin. Actualmente con fiebre,
rigidez de nuca, Kerney y Brusinski, puncin lumbar con diplococos GRAM NEGATIVOS
en LCR

Agente etiolgico?:
Meningitis por Neisseria Meningitidis (MENINGOCOCO)
Tx:
PGSC a dosis altas, al personal involucrado: cipro o rifampicina

Medida implantada por la OMS para deteccin rpida de MENINGITIDIS?


DETERMINACIN DE ANTGENOS CAPSULARES: Streptococo pneumone, Neisseria
Meningitidis, Hemophilus Influenza se detecta en 3 horas aunque no los solicites-
MENINGITIS
Meningitis POR LISTERIA (BACILO GRAM +) esta relacionado con:
LECHE O DERIVADOS LACTEOS, CARNES MAL COCIDAS, INMUNODEPRIMIDOS y px
de la tercera edad. Tx :AMPICILINA como primera opcin

Con respecto del Streptococo Pneumoniae caract de infeccin?:


Se da principalmente en pacientes de los 2 a los 20 aos, px ALCOHLICOS, con
OTITIS PREVIA.

Meningitis por Bacilos GRAM NEGATIVOS caract de infeccin?


Se presenta en diabticos, cirrticos, alcohlicos px con IVU, px con cncer y/o
inmunodeprimidos. El bacilo GRAM POSITIVO y entrico mas importante la
listeria monocitogena, otro bacilo que no es entrico es la hemophilus Influenza
(disminucin en su incidencia actualmente)

Agente etiolgico de la Meningitis en un px que previamente se le realizo


procedimiento neuroquirurgico?
STAPHILOCOCOS AUREUS

Citocinas proinflamatorias que producen dao a nivel SNC, en la barrera


hematoencefalica, plexos coroides?
IL 1 Y FNT que aumentan en las primeras 2 horas
MENINGITIS
Laboratorio de LCR?
PUNCIN LUMBAR: 1.-PRESION NORMAL DEL LCR 180mm de Agua, 2.-POLIMORFONUCLEARES:
menos de 5 polimorfos 3.-GLUCOSA: es el 50% dela glucosa central del px entre 45 y 55 4.-
PROTEINAS: 40mg/dl 5.-AGUA DE ROCA 6.-CLORUROS: 90 a 110 en LCR

En quienes se realiza el cultivo en TINTA CHINA?


CRIPTOCOCO NEOFORMANS EN PX CON VIH

Caso clinico
Px con rigidez de nuca, brusinski +, irritado Cefalea, fiebre, sin traumatismo craneoenceflico,
ANTECEDENTE DE NADAR EN UN MANANTIAL LANZNDOSE CLAVADOS, a la puncin lumbar: aspecto
turbio, presin:200mmhg glucosa:20mg/dl protenas:85 mg/dl Tincin de GRAM fue NEGATIVA, el
cultivo se realizo, que tipo de meningitis tiene este px:
a.-M aseptica b.-M purulenta c.-M Tuberculosa (Agente etiolgico: NAEGLERIA o gardenelas son
amibas de vida silvestre) muy agresivas, con pronostico mortal,
Tx?:
ANFOTERICINA B

La meningitis Viral caract del LCR:


CELULARIDAD: NORMAL PROTENAS: ligeramente AUMENTADAS ASPECTO: AGUA DE ROCA
GLUCOSA: NORMAL PRESIN: nl o ligeramente NORMAL CLORUROS: NORMALES
MENINGITIS
CASO CLNICO: Px con fiebre intensa (40), ingresa al servicio de urg, con trastorno
del edo. De conciencia, con 2 crisis convulsivas.. de ocupacin CUIDADOR DE
CABALLOS (antecedente de haber enterrado a su caballo y despus fue atacado
por aves de rapia)
LCR: Glucosa: 100 liquido transparente, presin: 185mm de agua, celularidad:
linfocitos cloruros normales
Cual es la impresin dx:
MENINGOENCEFALITIS VIRAL EQUINA VENEZOLANA

CASO CLNICO: Nia Indgena procedente de AREA RURAL, se presenta al servicio


de urg. Con 3 semanas de evolucin con fiebre persistente, vomito, trastorno del
estado de conciencia, ala exploracin fsica presenta papiledema ligero, con
afectacin del 3er, 4to, 6to par craneal, con rigidez de nuca. LCR: ASPECTO
XANTOCROMICO, Presin: 220mm de agua Glucosa: 30 Protenas: de 75mg/dl ,
Celularidad por linfocitos CLORUROS: 70 Meningitis tuberculosa

Tx de meningitis en Recin Nacidos?


AMPICILINA (e.coli)- CEFOTAXIMA (streptococo)

MENINGITIS
Tx de meningitis en nios de 1 a 3 meses?
AMPI + CEFO o CEFTRIAXONA + (valorar administracin de DEXAMETASONA) para evitar
VENTRICULITIS

Tx de meningits en px de 3 meses y menos de 50 aos?


CEFOTAXIMA + VANCOMICINA o CEFTRIAXONA + VANCOMICINA (cubrir cocos gram +, Y
bacilos gram -)

Tx de meningitis en un px mas de 50 aos con antecedente de alcoholismo o tiene DM


AMPICILINA + VANCOMICINA + CEFALOSPORINA (tx p cocos gram + bacilos gram -)

Px neutropenico, o px con meningitis con TRAUMATISMO CRANEOENCEFALICO, o que le


hicieron un procedimiento neuroquirurgico que tx se le dara?
Cubrir Gram - (ceftazidil), cubrir Gram + meticilino resistente (Vancomicina)

px que en el cultivo presenta meningococo que tx se da?


PENICILINA G si es resistente: CEFTRIAXONA O CEFOTAXIMA y si es alrgico
VANCOMICINA (2 gr x dia)

Px que le hicieron puncin lumbar y obtuvieron en el LCR un cultivo Gram que tx:
Pseudomona aeroginosa: Ceftazidine(6gr x dia) Kliebsella, e coli: CEFOTAXIMA O
CEFTRIAZONA (4gr x dia)

Tx para: Listeria: AMPICILINA(12 gr al dia) H. Influenza: CEFTRIAXONA o cefotaxima


Anaerobios: METRONIDAZOL (2 grx dia)
FOD
Cual es la definicin de FOD?
R = Elevacin de la temperatura por arriba de 38 GRADOS en varias determinaciones durante
mas de 3 SEMANAS sin llegar al diagnostico DESPUS DE 1 SEMANA de estudio hospitalario

Cual es la definicin de FOD en paciente hospitalizados?


R = Paciente hospitalizado 24 HRS CON FIEBRE y 2 CULTIVOS NEGATIVOS despus DE 2 DAS.

Define FOD en paciente inmunodeficiente?


R = FIEBRE >38.3 en paciente con NEUTROPENIA < 500 durante > 3 DAS y cultivos negativos
despus DE 2 DAS.

Define FOD asociado a VIH?


R = Fiebre por encima de 38.3 GRADOS y que permanece sin diagnostico por MAS DE 3
SEMANAS

Cuales son las principales causas de FOD?


1. Infeccin por TB y CMV
2. Neoplasia por linfoma o leucemia
3. Idioptica si despus de 6m de estudio no hay causa
SEPSIS
Como se diagnostica el sndrome de respuesta inflamatoria sistmica?
R = Cuando se presentan 2 o mas de los siguientes parmetros:
1. FIEBRE corporal > 38 o hipotermia < 36
2. TAQUIPNEA con FR > 24 x minuto
3. TAQUICARDIA > 90 x minuto
4. LEUCOCITOSIS > 12,000 o leucopenia < 4,000 o 10% de bandas

Como diagnosticas sepsis grave?


R = Sepsis asociada a disfuncin de un rgano con los siguientes datos:
1. Cardiovascular: TAS < 90 mm Hg que responde a fluidoterapia
2. Respiratoria: Relacin FiO2/PaO2 < 200
3. Renal: Diuresis < 0.5 ML/KG/HR durante 1 hr a pesar de fluidoterapia
4. Trombocitopenia: < 80,000 o su reduccin a la mitad con respecto a la obtenida
72 hrs antes
5. Acidosis metablica: Con PH < 7.30
6. Choque sptico: Hipotensin con TAS < 90 durante 1 hr y que no responde a
fluidoterapia necesitando vasopresores
SEPSIS
En caso de bacteriemia o sepsis cuales son algunas caractersticas
distintivas de los agentes patolgicos?
A. Neisseria meningitidis: Se acompaa de PURPURA O PETEQUIAS
B. Rikettsia: Cuando aparecen LESIONES PETEQUIALES por mordida
de garrapata en zona endmica
C. Pseudomona aureoginosa: ECTIMA GANGRENOSO
D. S. Aureus o S. Pyogenes: ERITRODERMIA GENERALIZADA.

Cual es la complicacin mas frecuente de la bacteriemia?


R = SIRPA caracterizado por HIPOXEMIA E INFILTRADOS
PULMONARES DIFUSOS.
NEUROCISTICERCOSIS
Cual es el agente infeccioso en neurocisticercosis?
R = Tenia SOLIUM

Cual es el cc de la neurocisticercosis?
1) Forma activa: Sin evidencia del parasito con hallazgos en la TAC de
calcificaciones o hidrocefalia
2) Parenquimatosa: Larvas en corteza y ganglios basales
3) Forma subaracnoidea: Con fibrosis leptomeningea provocando
neuropatas por atrapamiento o hidrocefalia

Cual es el mtodo diagnostico de eleccin para le


neurocisticercosis?
R = TAC de eleccin e IRM en casos dudosos.

Cual es el manejo de la neurocisticercosis?


R = ALBENDAZOL 400 mg c/12 1 SEMANA, PRAZICUANTEL X 15 DAS.
SE RECOMIENDA EL USO DE PREDNISONA 1 DA ANTES DEL FRMACO
Y CONTINUAR CON DISMINUCIN DE LA DOSIS 14 DAS DESPUS.
TETANOS
Cual es el cuadro clnico del ttanos?
R = Comienza con ESPASMOS LEVES EN LA MANDBULA (trismo), el
cuello y la cara. La RIGIDEZ se desarrolla rpidamente en el TRAX,
ESPALDA, MSCULOS ABDOMINALES y en ocasiones la laringe
interfiriendo con la respiracin. Los espasmos musculares son
contracciones sbitas, fuertes y dolorosas.

Cual es el tratamiento de eleccin para ttanos?


R = PENICILINA SDICA CRISTALINICA 20, 000 000. Se debe
administrar globulina inmunitaria antitetnica 5000 U/IM y una vez
recuperado el paciente se da esquema completo.
CARBUNO O ANTRAX
Cual es el agente etiolgico del Carbunco o ANTRAX?
R = BACILLUS ANTHRACIS, bacilo GRAM +

La forma cutnea de carbunco o ntrax que es la mas comn como se manifiesta?


R = Desarrollo de PPULA EN ZONA DE INOCULACIN, RODEADA DE VESCULAS, LA ZONA CENTRAL SE ULCERA
Y SECA DESARROLLNDOSE ZONA DEPRIMIDA DE COLOR NEGRO, NO DOLOROSO. Se resuelve
espontneamente pero 20% puede ser fatal.

Como se manifiesta la forma intestinal de carbunco o ntrax?


R = FIEBRE, DISNEA, CIANOSIS, desorientacin y signos de septicemia. Evoluciona rpidamente a choque, coma
y muerte.

Como se manifiesta el carbunco o ntrax respiratorio y quienes lo PADECEN FRECEUNTEMENTE?


R = Se manifiesta en personas que MANIPULAN PIEL Y LANAS. Se produce FIEBRE CON TOS NO PRODUCTIVA y
mal estar general, con buena evolucin a 2-3 das y a continuacin SBITAMENTE PRESENTA DIFICULTAD
RESPIRATORIA GRAVE, CIANOSIS Y SEPTICEMIA FATAL CON MUERTE EN <24 HRS.

Como realizas el diagnostico de carbunco o ntrax?


R = Identificacin del bacilo con TINCIN GRAM, PCR.

Cual es el manejo de carbunco o ntrax?


R = CIPROFLOXACINO, levofloxacino o PENICILINA G.

Como previenes el carbunco o ntrax?


R = PROTECCIN DE PIEL, MUCOSAS Y LA VACUNACIN
SALMONELOSIS
Cuantos antgenos tiene la salmonella typhi?
R = 3: H FLAGELAR, K CAPSULAR y O SOMTICO

Cuales son los sitios frecuentes de infeccin secundaria por salmonella typhi?
R = Hgado, bazo, la medula sea, PLACAS DE PEYER DEL LEON TERMINAL y la vescula biliar.

Cual es el cuadro clnico caracterstico de salmonella typhi?


R = ROSOLA TIFOIDICA que se caracteriza por MACULAS ERITEMATOSAS DE 2-4 MM QUE BLANQUEAN A
LA PRESIN, localizadas en la parte superior del abdomen y trax anterior y que por lo general dura de 2-3
das.

Cual es la complicacin mas frecuente de la salmonella typhi?


R = La mas comn es la HEMORRAGIA GI que resulta de la erosin de un vaso de la pared intestinal secundaria
a la necrosis de las placas de peyer en el leon terminal

Cual es el medio diagnostico mas sensible en la primer semana?


R = HEMOCULTIVO, despus el mielocultivo.

Semana en la que se presentan las complicaciones de fiebre tifoidea?


R = Fines de SEGUNDA Y TERCERA SEMANA

Con que inoculo de S. Tiphy se desarrolla la enfermedad?


R = 100,000
SALMONELOSIS
A que semana aparece la rosola tifoidica, la cual aparece a la digito presin?
R = 2da semana

Como dx a un portador crnico de salmonella?


R = COPROCULTIVOS + DURANTE 1

En que semana los px se perforan o tienen hemorragias con choque hipovolemico?


R = 3ERA semana

En que semanas en la fiebre tifoidea se realizan los cultivos especficos?


R = 1era HEMOCULTIVO, 2da MIELOCULTIVO y 3era COPROCULTIVO/urocultivo

Cual es el cuadro clnico de la salmonelosis?


R = Fiebre elevada 2-3 semanas, dolor abdominal, DIARREA EN SOPA DE
CHICHARO, ROSOLA, DELIRIO, ESPLENOMEGALIA.
Cual es la prueba de laboratorio confirmatoria de fiebre tifoidea?

R = 1: 640 , LEUCOPENIA y cuadro clnico caracterstico


SALMONELOSIS
Cual es el sitio donde suele albergarse la salmonella?
R = Vescula biliar

Que se presenta en la primera semana de la salmonelosis?


R = FIEBRE, HIPERSENSIBILIDAD ABDOMINAL, TIFLITIS (CUANDO HAY DOLOR EN FOSA ILIACA
DERECHA POR INVASIN DE LA PLACA DE PEYER Y CREPITA AL TACTO)

Que se presenta en la segunda semana en la fiebre tifoidea?


R = DIARREA, estreimiento, delirio, EXANTEMA EN CARA ANTERIOR DE TRAX QUE SEDE A
LA DIGITO PRESIN. El sistema retculo-endotelial se hipertrofia con
HEPATO/ESPLENOMEGALIA, hiperplasia de las PLACAS DE PEYER. Bazo e hgado desarrollan
ndulos tifoideos.

Cuales son las complicaciones habituales de la fiebre tifoidea en la 3era semana?


R = HEMORRAGIA Y PERFORACIN INTESTINAL, peritonitis con placa simple de abd con aire
libre. Orquitis, meningitis, nefritis.

Cual es el tratamiento de la salmonella multiresistente o en lugares o zonas endmicas?


R = CIPROFLOXACINO, levofloxacino, ceftriaxona
SALMONELOSIS
Cual es el mejor mtodo diagnostico en un paciente que se
automedico en fiebre tifoidea?
R = MIELOCULTIVO por que el medicamento no penetra a esa zona

Tratamiento para fiebre tifoidea?


R = CLORAMFENICOL 50/mg/kg por 2 SEMANAS

Tratamiento para anemia + fiebre tifoidea?


R = AMOXICILINA o ampicilina, NO DARLE CLORAMFENICOL por ke
causa aplasia medular.

Tx fiebre tifoidea en embarazada?


R = AMOXICILINA o ampicilina
SALMONELOSIS
Tx fiebre tifoidea en nio con resistencia?
R = QUINOLONAS NO, cefotaxima o CEFTRIAXONA

En que casos se utiliza la dexametasona en fiebre tifoidea?


R = En el ESTADO TIFOIDICO CON SHOQUE

Que se le da al portador de salmonella?


R = CIPROFLOXACINA por 3 MESES

Que tx das a un px portador crnico que no respondi a


ciprofloxacino?
R = COLECISTECTOMA, por que ah vive la salmonella.
BRUCELOSIS
Cuales son los principales vectores de la brucelosis y a quienes afecta
comnmente?
R= ORDEADORES, rastros, CARNICERO, veterinarios son vectores.
DERIVADOS DE LA LECHE como el queso o tejidos del animal afectado.

Cuales son las cepas de brucella que produce enf en humano/ FIEBRE
ONDULANTE?
R = MELITERSIS DE CABRA + frecuente a nivel mundial, SUIZ DEL CERDO +,
ABORTUS DE GANADO VOVINO +++.

Microbiologicamente que es la brucella?


R = COCOBACILO GRAM -, crece a 37 grados, es inmvil a pesar de TENER
FLAGELO.

Cual es la prueba de laboratorio para brucelosis?


R = 2- MERCAPTOETANOL, es una inmunoglubulina IgG Vs BRUCELLA
BRUCELOSIS
Cual es el medio de cultivo de eleccin para brucelosis?
R = RUIZ CASTAEDA medio doble y ROSA DE BENGALA (fines
epidemiolgicos)

Que tratamiento utilizas de primera eleccin contra brucella?


R = Combinado DOXICICLINA + GENTAMICINA de 3-6 semanas.

Que tx utilizas en brucelosis en hueso y SNC?


R = DOXICICLINA Y RIFAMPICINA

Cual es el tratamiento de brucelosis en embarazada?


R = TMP/SMZ+ RIFAMPICINA + ACIDO FLICO

Cual es el tratamiento de nios menores de 12 con brucelosis?


R = TMP/SMZ + RIFAMPICINA

Cual es el tratamiento en pacientes alrgicos a sulfas en brucelosis?


R = RIFAMPICINA Y CEFTRIAXONA.
BOTULISMO
Cual es el cuadro clnico del CLOSTRIDIUM BOTULINUM
transmitido por alimentos?
R = Los sntomas comienzan en 6 HRS DESPUS A 2
SEMANAS, se manifiesta con DIPLOPA, VISIN BORROSA,
PTOSIS, disfagia, sensacin de sequedad de mucosa oral,
DEBILIDAD MUSCULAR que afecta nicamente a los
hombros, miembros torcicos y mas tarde a los plvicos.

Cual es el manejo?
R = Administracin de TOXINA BOTULNICA con PREVENCIN
DE INSUFICIENCIA RESPIRATORIA que amerite ventilacin
mecnica
PESTE BUBONICA
Cual es el agente causal por la peste bubnica?
R = YERSINIA PESTIS

Cual es el cuadro clnico de la peste bubnica?


R = Es la forma mas frecuente, se transmite por la PICADURA DE PULGA. Inicio con FIEBRE
ALTA, nauseas, MIALGIAS y ADENOPATAS DOLOROSAS REGIONALES, el PACIENTE
EVOLUCIONA A ESTUPOR, COMA Y MUERTE.
Que formas clnicas tiene la peste?
R = Peste bubnica, septicmica y neumnica

Como diagnosticas peste?


R = BIOPSIA DE GANGLIOS o EXUDADO FARNGEO

Cual es el tratamiento de la peste?


R = ESTREPTOMICINA, CLORANFENICOL O TETRACICLINAS.

Cual es la prevencin de la peste?


R = Hay una vacuna con CEPA INACTIVADA F1
RABIA
Cual es el agente causal de la rabia?
R = Virus de la familia Rabdovirus

Cual es el cuadro clnico de la rabia?


R = Dolor en el sitio de la mordedura, fiebre, malestar general, nausea y vomito. Diez das
mas tarde hay manifestaciones del SNC pudiendo ser enceflica o paralitica y las 2 formas
evolucionan a coma - muerte.
A. ENCEFALICA: Se caracteriza por delirio, somnolencia, hidrofobia (espasmos larngeos
dolorosos al beber agua)
B. PARALITICA: Da una parlisis ascendente.

Cual es el manejo de la rabia?


I. Es necesario observar al animal los siguientes 10 DAS tomando UNA BIOPSIA DEL CUERO
CABELLUDO y analizarla con Ac fluorescente o PCR.
II. Atencin de la LESIN LAVANDO CON AGUA Y JABN ABUNDANTE A CHORRO DURANTE
10 MINUTOS, para MUCOSA ORAL O NASAL CON SOLUCIN FISIOLGICA DURANTE 5
MINUTOS. Desinfectar la herida con agua oxigenada o tintura de yodo.
III. Si se requiere SUTURAR LA HERIDA se debe APLICAR PRIMERO INMUNOGLOBULINA
ANTIRRBICA HUMANA y se aproximan los bordes o se dan puntos temporales en caso
que la herida sea profunda,
IV. En exposicin leve se administra vacunacin antirrbica en la regin deltoidea los das 0-3-
7-14 Y 28.
V. Se APLICARA INMUNOGLOBULINA INTRALESIONAL LA PRIMER Y LA OTRA MITAD VA IM.
VI. Se usara SUERO HETEROLOGO EN CASO EXTREMO DE NO CONTAR CON
INMUNOGLOBULINA.
INFLUENZA
DENGUE Y FIEBRE AMARILLA
Cual es el mosquito transmisor del dengue?
R = AEDES AEPYPTI.

Cual es el cc del dengue?


1) El DENGUE CLSICO se inicia con FIEBRE, BRADICARDIA, pulso lento, EXANTEMA
MACULAR PUNTIFORME, tos, ardor farngeo, MIALGIAS, ARTRALGIAS.
2) El DENGUE HEMORRGICO se caracteriza por aumento en la permeabilidad vascular con
clasificacin del I-IV.

Que es lo que distingue a la fiebre amarilla del dengue laboratorialmente?


R = CUERPOS DE INCLUSIN VIRAL llamados CUERPOS DE COUNCILMAN EN LA SANGRE.

Cual es el cuadro clnico de la fiebre amarilla?


R = FIEBRE, cefalea, mialgias, DOLOR LUMBOSACRO, ERITEMA OCULAR Y FACIAL,
PETEQUIAS EN ENCIAS Y MUCOSA NASAL, HEMATEMESIS, MELENA, ALBUMINURIA,
encefalitis o meningoencefalitis.

Cual es el manejo de la fiebre amarilla y del dengue?


R = SINTOMTICO
VIH
Que clulas ataca el VIH?
R = CD4

Cual es el ciclo vital del VIH?


R = A travs de la protena gp41, se realiza la fusin, posteriormente la RNA del VIH se
descubre e interna en la clula afectada, la enzima transcriptasa inversa del virion cataliza la
transcripcin inversa del RNA en DNA, este se transfiere hacia el ncleo en el cual se integra
en los cromosomas por medio de la integrasa. Los macrfagos actan como reservorio del
VIH y lo diseminan a otros sistemas

Que se debe realizar como prevencin en una persona que halla sido puncionada de
manera accidental con una aguja utilizada previamente en un paciente con VIH?
R = INHIBIDORES NUCLEOSIDOS DE LA TRANSCRIPTASA REVERSA

Como se manifiesta el sndrome retroviral agudo SRA en VIH?


R = SNTOMAS SIMILARES AL RESFRIADO O MONONUCLEOSIS, fiebre, escalofros, sudores
nocturnos y erupciones en la piel que dura de 1-3 SEMANAS.

De acuerdo a la CDC como se realiza dx de SIDA?


R = Recuento <200/MM DE CD4 + la presencia de ENFERMEDAD OPORTUNISTA
VIH
Como dx VIH?
R = ELISA y confirmatorio con WESTERN BLOOD

Cuales son parte del grupo de frmacos que inhiben la TRANSCRIPTASA INVERSA en VIH?
R = ZIDOVUDINA, didanosina, zalcitabina.

Cuales son las indicaciones actuales para uso de tratamiento retro vrico en VIH?
1. Sndrome de infeccin aguda
2. INFECCIN CRNICA: Enfermedad sintomtica o asintomtica con T CD4 <350/NL o RNA
DEL VIH CON > 50, 000 COPIAS
3. PROFILAXIS POST- EXPOSICIN: Se recomienda utilizar combinaciones como ZIDOVUDINA
+ LAMIVUDINA + NEVERIPINA

Cuales son las indicaciones para cambiar el tratamiento antirretrovirico?


1. Disminucin <1 log del RNA del VIH con plasma a las 4 semanas de haber iniciado el
tratamiento
2. Incremento DEL TRIPLE O MAS DEL VALOR PLASMTICO DEL RNA NO ATRIBUIBLE A
INFECCIN concomitantes o vacuolas.
3. DISMINUCIN DEL PORCENTAJE DE CLULAS TCD4
4. Deterioro clnico
5. Efectos adversos

VIH
Como se manifiesta la neumona por Pneumocystis Jiroveci?
R = CD4 < 200/MM mas AUMENTO DE LA DHL, el diagnostico definitivo se obtiene por la TINCIN DE ESPUTO CON
WRIGHT-GIEMSA

Cual es la causa mas frecuente de retinitis en pacientes con VIH?


R = CITOMEGALOVIRUS

Mecanismo por el que el VIH penetra a la clula?


R = Pinocitosis

Cual es la neoplasia maligna mas comn en pacientes con VIH?


R = Sarcoma de Kaposi

Cual es el manejo del Sarcoma de Kaposi?


R = Para la FORMA CUTNEA SE DA DOXORRUBICINA, para el INTESTINAL O VISCERAL SE DA
DOXORRUBICINA, BLEOMICINA Y VINBLASTINA.

Como se maneja la profilaxis en pacientes con VIH de acuerdo al conteo de las clulas CD4?
1) CD4 < 200: VS P. JIROVECI con TMP/SMZ 1Tab/24 hrs C/3 DIA
2) CD4 75-100: VS M. AVIUM con CLARITROMICINA y se SUSPENDE CUANDO AUMENTAN LOS CD4
3) CD4 < 50: VS CITOMEGALOVIRUS con GANCICLOVIR
4) PPD con induracin > 5 MM iniciar profilaxis VS M. TUBERCULOSIS con ISONIACIDA a 300 mg al da +
PIRIDOXINA durante 9-12 MESES.
VIH
Tratamiento de primera eleccin para la retinitis por CITOMEGALOVIRUS?
GANCICLOVIR

Tratamiento para el herpes tipo 2 en pacientes con VIH?


ACICLOVIR

De que manera se puede inhibir el sndrome de desgaste?


TALIDOMIDA

Agente infeccioso de la lengua vellosa?


Virus de EPSTEIN BARR ( relacionada con virus de mononucleosis y linfomas)
VIRUS TIPO 4
Sitio mas comn de lesin dermatolgica, SARCOMA DE KAPOSI manchas color de piel?
El sitio de afectacin mas frecuente es la CARA EN PARPADOS Y PUENTE NASAL
RELACIONADO CON EL VIRUS TIPO 8

Tratamiento de toxoplasmosis?
PIREMITAMINA o sulfas + CLINDAMICINA en dosis altas
VIH
Posibilidad de contagio por coito anal receptivo en el VIH?
1:100 1:30

Zona donde se alojan el virus de la inmunodeficiencia es alta y til para seguimiento del tx
de VIH?
Biopsia de mucosa rectal otra opcin es PCR

Transmisin perinatal del VIH?:


13% al 40%

Coito con insercin anal: 1: 1,000


Transfusin: 1: 100,000
Coito insercin vaginal: 1: 10,000 varia si la mujer se encuentra en periodo menstrual o si
tiene una ETS

Aguja infectada?:
1-300

Compartir agujas?:
1-150 (drogadictos)

A que se refiere la regla de los 3 con relacin a aguja infectada?


Hepatitis B : 30% Hepatitis C: 3% HIV: .3%
VIH
Diagnostico para valorar el tratamiento respuesta teraputica
RCP se realiza al mes y medio y a los 3 meses y posteriormente CADA 6 MESES (no se usa para diagnostico)

Que pasa con una persona que tiene VIH y las siguientes enfermedades Concomitantes?
Hepatitis B: Indiferente (no pasa nada)
Hepatitis C: Agresividad
HEPATITIS G: FRENA LA PROGRESIN DEL VIH

Con respecto a la estatificacin de etapas A-B-C y grados I al III?


A: asintomticos B: Sntomas Constitucionales (fiebre, perdida de peso, diarrea) C: Infecciones Oportunistas
(neumocistis jirovecci, linfoma del SNC, Criptosporidiasis, Toxoplasma del SNC, Micovacterium Avium intra
celular)
I.- 500 copias II.-MAS DE 200 Y MENOS DE 500 III.-MENOS DE 200 copias

INHIBIDORES DE TRANSCRIPTASA INVERSA -no anlogo de los Nucleosidos- NNRTI


De la ZIDOVUDINA AZT cual es el efecto indeseable? PROBLEMAS MEDULARES, anemia severa,
trombocitopenia, leucopenia

DE LA EFEVIRENZ cual es el efecto indeseable?


Insomnio y pesadillas (DEPRESIN)

INHIBIDORES DE PROTEASAS
Del Indinavir efecto indeseable mas frecuente: FORMADOR DE CLCULOS clico renoureteral
Ritonavir + lopinavir : Es el caletra efecto sinrgico
VIH
Profilaxis (exposicin en trabajadores de la salud)(exposicin SEXUAL) que maniobras o
procedimientos?
Tiene una posibilidad de infeccin de 1:300 SE REALIZA UN ELISA con fines legales, ( AL
MES) si se considera EXPOSICIN DE BAJO RIESGO dar AZT+3TC X UN MES se REALIZA
ELISA SI ES NEGATIVO SE SUSP si es exposicin de alto riesgo AZT+3TC +UN TERCER
MEDICAMENTO. NO DAR TX A LAS 72 POSTEXPOSICION

Copias de CD4?
Entre 50 y 100 copias en el momento del dx: MVAIC, CMV, Linfomas del SNC
MENOS DE 200: Criptocococis, toxoplasmosis, NEUMOCISTIS JIROVECCI
Entre 200 y 500: LENGUA VELLOSA, Sarcoma de Kaposi, TB Pulmonar, Infeccin por virus
herpes, candidiosis

Px con HIV y tiene exposicin con px con tb?


Profilaxis: PPD MAS DE 5MM ISONIACIDA + RIFAMPICINA + PIRAZINAMIDA POR 12 MESES
+ PIRIDOXINA

En que consiste el sndrome retroviral agudo?


1) Paciente con infeccin de VIH con SNTOMAS A CORTO PLAZO (de 2 A 3 SEMANAS y
sucede en el 40%de los casos) Y CONSISTE EN FIEBRE ADENOPATA, RASH CUTNEO
COLOR SALMON Y HEPATOMEGALIA CUADRO MUY PARECIDO AL DE LA
MONONUCLEOSIS INFECCIOSA,
2) DX ELISA A LAS 6 SEMANAS Y 3 MESES si sale positivo se realiza el WESTERN BLOT PARA
CONFIRMARLO
VIH
Combinaciones aprobadas:
I. Tenofovir (TDF)+ emtricitabina (FTC) y efavirenz (EFV)
II. Atazanavir (ATV) + ritonavir (rtv)+tenofovir (TDF) / emtricitabina (FTC) (si no funciona el primer
esquema)
III. Caletra +tenofovir (TDF) / emtricitabina (FTC) (El caletra se puede cambiar con AZT + 3TC)

Efectos colaterales?
I. Zidovudina.. ANEMIA por hipoplasia medular
II. Nerfinavir. DIARREA
III. Didanosina. PANCREATITIS
IV. Zalcitabina.. NEUROPATA PERIFRICA
V. Estaduvidina. NEUROPATA PERIFRICA (lamivudina +zidovudina) 3TC.. Neuropatia perifrica
VI. Abacavir.. FIEBRE RASH, HIPERSENSIBILIDAD FATAL
VII. Tenofovir. TOXICIDAD RENAL
VIII. Saquinavir. LPIDOS ELEVADOS

Px con embarazo + VIH Cual es el tx?

I. ZIDOVUDINA (2do y 3er trimestre) se continua en el trabajo de parto (para evitar la


infeccin vertical)

II. POSPARTO (NO LACTANCIA 10-20 tiene posibilidad de infectar) 2da opcin LAMIVUDINA
VIH
Cuales son parte del grupo de frmacos INHIBIDORES DE
PROTEASA en VIH?
R = SaquinAVIR, ritonAVIR, indinAVIR.

Cuales son parte del grupo de frmacos INHIBIDORES DE LA


FUSIN?
R = Enfubitida, que entre sus efectos adversos se encuentra la
hipersensibilidad y neumona bacteriana

Cuales son parte del grupo de frmacos que INHIBEN LA


TRANSCRIPTASA INVERSA en VIH?
R = ZIDOVUDINA, didanosINA, zalcitabINA.
ASPERGILOSIS
Como se adquiere la aspergilosis?
R = Por inhalacin de ESPORAS DEL HONGO, las cuales se encuentran en HOJAS SECAS,
GRANEROS DE MAIZ, ESTIRCOL Y VEGETALES EN DESCOMPOSICIN.

En los pacientes con rinitis crnica por aspergilus, que datos de laboratorio encuentras en
el moco?
R = Rico en EOSINOFILOS y cristales de CHARCOT-LEYDEN

En que consiste la aspergilosis pulmonar endobronquial saprofita o aspergiloma?


R = En el CRECIMIENTO DEL HONGO dentro de las cavidades pulmonares, SECUNDARIAS
GENERALMENTE A TB, SARCOIDOSIS, HISTOPLASMOSIS O BRONQUIECTASIAS.

Como dx aspergilus?
R = CULTIVO DE SABOURAD, deteccin de ANTICUERPOS, galactomanano en suero (chbts
en su pared celular) y biopsia

Cual es el tratamiento de aspergilus?


R = Esteroides, ANFOTERICINA B, LOBECTOMA EN CASO DE ASPERGILOMA o debridacion de
tejido infectado.

Cual es el tratamiento de eleccin en caso de toxicidad secundaria al DESOXICOLATO de


ANFOTERICINA B en caso de aspergilosis invasiva?
R = VORICONAZOL 6mg/kg/dia.
CANDIDOSIS
Como se manifiesta la CANDIDIASIS DIGESTIVA y como se trata?
R = Se ASOCIA a lesiones ORALES Y PERIANALES, produce DIARREA CON MOCO, el
diagnostico se realiza con el EXAMEN DIRECTO DE LAS HECES CON KOH y se trata
con NISTATINA ORAL.

Como se manifiesta la CANDIDIOSIS URINARIA y como se trata?


R = Se presenta como CISTITIS, pielonefritis, BOLAS FNGICAS RENALES, ABSCESOS
RENALES o necrosis de las papilas. Se trata con fluconazol o ANFOTERICINA B.

Como se diagnostica CANDIDIOSIS PULMONAR y como se trata?


R = Cuando realizas HEMOCULTIVOS POSITIVOS o ANTGENO MANAN DEL
SUERO. El tratamiento se realiza con ANFOTERICINA B, en los pacientes que no la
toleran se utiliza FLUCONAZOL.

Como se manifiesta la ENDOCARDITIS por candidiosis?


R = FIEBRE persistente, SOPLOS CARDIACOS y ESPLENOMEGALIA, en algunos casos
hay embolizacin a grandes arterias, como la iliaca o femoral. LA VLVULA MAS
AFECTADA ES LA MITRAL.
CANDIDOSIS
Como diagnosticas ENDOCARDITIS por cndida?
R = Ecocardiagrafia, hemocultivos seriados y deteccin de
ANTGENO MANAN en suero?

Cual es el tratamiento de endocarditis por cndida?


R = RESECCIN DE VLVULA afectada y ANFOTERICINA B

Como se presenta la candidiasis diseminada aguda o sepsis?


R = Se presenta en PACIENTES NEUTROPENICOS, CON FIEBRE,
LEUCOCITOSIS, TROMBOCITOPENIA, inestabilidad
hemodinmica y choque.
CRIPTOCOCOCIS (AVES, PALOMAS)
Cual es el agente causal de la criptococosis y donde se
alberga?
R = CRYPTOCOCCUS NEOFORMANS, se encuentra en HECES
DE PALOMAS (CUIDADORES DE AVES) O NIDOS

Como diagnosticas criptococosis?


ANTGENO MANAN en suero o biopsia.
En LCR con TINCIN DE TINTA CHINA

Como se trata la criptococosis?


R = ANFOTERICINA B y 5-fluorocitosina
HISTOPLASMOSIS (CAVERNAS)
Donde se alberga el Histoplasma capsulatum?
R = En forma de moho en suelo RICO EN NITRGENO, como el de
los GALLINEROS, o donde duermen las aves y CAVERNAS DE
MURCILAGOS

Como diagnosticas histoplasmosis?


R = El mtodo de eleccin es el CULTIVO, pero tarda mucho y
haces frotis de los fluidos o tejidos infectados, las muestras para
cultivo en HISTOPLASMOSIS DISEMINADA SE OBTIENEN DE
MEDULA SEA, HGADO, GANGLIOS LINFTICOS O LESIONES
MUCOCUTANEAS.

Cual es el manejo de la hispoplasmosis?


R = Anfotericina B
MUCORMICOSIS
Cual es el agente etiolgico de la mucormicosis?
R = MUCOR, ABSIDIA, RIZOPUS denominado ficomicetos.

Como se manifiesta la infeccin de los senos paranasales en mucormicosis?


R = Predominantemente a PACIENTES CON DM mal tratada, con FIEBRE,
congestin nasal, SECRECIN NASAL SANGUINOLENTA, DOLOR SINUSAL Y
DIPLOPA, a la exploracin fsica se observa DISMINUCIN DE LOS MOVIMIENTOS
OCULARES, quemosis y PROPTOSIS y SI INVADE EL SENO FRONTAL PRODUCE COMA.

Como se manifiesta la mucormicosis gastrointestinal?


R = Se disemina al peritoneo produciendo la muerte en 70 das

Como se diagnostica mucormicosis?


R = FROTIS EN FRESCO donde se ven HIFAS HIALINAS o en BIOPSIAS donde se
observa NECROSIS ISQUMICA o hemorrgica

Cual es el manejo de la mucormicosis?


R = El tratamiento consiste en altas dosis de ANFOTERICINA B, ASEO QUIRRGICO
DE LOS TEJIDOS.
COCCIDIOIDOMICOSIS
Cuales son los signos y sntomas de la coccidioidomicosis?
R = TOS, RINORREA, febricula, dolores musculares y en la RADIOGRAFA DE
TRAX CON CAVITACIN.

- A 43-year-old man developed a cough shortly after returning from a 1-


month hiking trip in California. While there, he was hiking in the central
California valleys. During his trip, he had developed a flu-like illness
consisting of fever, cough, and muscle pains, which resolved spontaneously.
A CXR shows a thin-walled cavity in the right upper lobe, and the sputum
reveals fungal elements. Which of the following is the most likely causative
organism?

(A) ringworm
(B) Cryptococcus neoformans
(C) Candida albicans
(D) mycobacteria
(E) coccidioidomycosis
BLASTOMICOSIS
Cual es el agente causal de blastomicosis?
R = BLASTOMYCES DERMATITIDIS

Cuales son las manifestaciones clnicas de blastomicosis?


R = PULMONAR Y CUTNEA. NEUMONA aguda que no responde al tratamiento y
CURA ESPONTNEAMENTE. Las LESIONES CUTNEAS predominan en las ZONAS
EXPUESTAS, caractersticamente son PAPULAS, NDULOS O PLACAS BIEN
DELIMITADAS, nicas o mltiples, NO DOLOROSAS NI PRURIGINOSAS, que
evolucionan a lesiones verrugosas, costras o ulceras.

Como diagnosticas blastomicosis?


R = De ELECCIN CULTIVO pero tarda mucho, se pueden identificar organismos con
la TINCIN DE PLATA, METENAMINA O ACIDO PERIODICO DE SCHIFF en la biopsia
de los tejidos infectados o en la citologa de esputo. Donde se observan CLULAS
LEVADURIFORMES CON YEMAS DE BASE AMPLIA.

Cual es el tratamiento de blastomicosis?


R = En la forma LEVE - ITRACONAZOL, en la forma GRAVE - ANFOTERICINA B
PALUDISMO
Cual es el agente causal de paludismo o malaria?
R = PARASITOSIS intracelular eritrocitaria por PLASMODIUM, que se transmite
por el MOSQUITO ANOPHELES

Cuantas especies de plasmodium existen?


R = 4, P. falciparum, P. vivax, P. ovale y P. malariae

Cual es el ciclo biolgico del plasmodium?


R = Los mosquitos se infectan al ingerir sangre infectada con micro y
macrigametocitos. Dentro del mosquito, se multiplica sexualmente en su intestino,
produciendo los ooquistes, y al madurar se rompen liberando ESPOROZOITOS, que
migran a las glndulas salivales del mosquito y son la forma infectante en los seres
humanos. Ya en el TORRENTE SANGUNEO viajan a hgado donde se reproducen y
forman los MEROZOITOS que tienen la capacidad de infectar los eritrocitos.

Que produce la lisis eritrocitaria al contraer paludismo?


R = La PLASMEPSINA, que es una proteasa del acido aspartico que DEGRADA LA
HEMOGLOBINA.
PALUDISMO
Cual es el tipo de genero en paludismo mas frecuente?
R = P. FALCIPARUM, es la mas frecuente y de peor pronostico con
fiebre > 40

Como se diagnostica el paludismo?


R = OBSERVACIN DE PARASITO EN FROTIS SANGUNEO O GOTA
GRUESA. La TINCIN DE GIEMSA permite observar las
granulaciones de Schuffner.

Como se trata el paludismo?


R = QUININA Y CLOROQUINA. El SULFATO DE QUININA se usa en
RESISTENCIA A CLOROQUINA
BORRELIOSIS O ENFERMEDAD DE
LYME
Cual es el agente causal de la enfermedad de Lyme?
R = Espiroqueta BORRELIA BURGDORFERI GRAM (-) de color violeta con
tincin Giemsa que se CULTIVA EN MEDIO NOGUCHI. Se adquiere por
MORDEDURA DE GARRAPATA del genero IXODES.

Cual es el cuadro clnico de la enfermedad de Lyme?


R = Se divide en 3 etapas
I. ERITEMA CRNICO O MIGRANS, ppulas eritematosas que se EXTIENDEN
DE FORMA CENTRIFUGA DEJANDO LA PARTE CENTRAL DEL CUERPO
SANA, acompaada de sntomas generales como astenia, adinamia,
fiebre, mialgias y cefalea.
II. Alteraciones neurolgicas, cardiacas y reumticas
III. POLIARTRITIS MIGRATORIA, alteraciones dermatolgicas y neurolgicas

Como se diagnostica la enfermedad de Lyme?


R = ELISA o PCR
BORRELIOSIS O ENFERMEDAD DE
LYME
Tratamiento de eleccin para enf de Lyme?
R = DOXICICLINA por mes y medio

Cual es el tratamiento para borreliosis en su etapa


precoz?
R = DOXICICLINA.

Cual es el tratamiento para borreliosis en su etapa


tarda?
R = CEFTRIAXONA
LEPTOSPIROSIS
Cual es el vector de la leptospirosis, que enzimas produce y que nutrientes necesita?
R = Leptospira interrogans en ORINA DE RATA. PRODUCE CATALASA Y HIALURODINASA y requieren
TIAMINA B1 Y VITAMINA B12 PARA SU DESARROLLO

Cuales son las manifestaciones clnicas de la Leptospirosis?


R = De inicio sbito, con FIEBRE, derrame conjuntival, epistaxis, MIALGIAS, cefalea intensa, EXANTEMA Y
HEPATOMEGALIA.

Que es el sndrome de Weil en Leptospirosis?


R = Al cursar la primera fase, viene una ASINTOMTICA para posteriormente PRESENTARSE
LEPTOSPIROSIS ANICTERICA.

Como se manifiesta el sndrome de Weil?


R = CON FIEBRE, CEFALEA, MIALGIAS, EXANTEMA, miocarditis, ICTERICIA, insuficiencia heptica,
INSUFICIENCIA RENAL, FENMENOS HEMORRGICOS, LEPTOSPIURIA y dao al SNC (MENINGITIS
ASEPTICA).

Cual es la triada clsica del sndrome de Weil?


R = Dao en HGADO, RIN y SNC.

Cual es el tratamiento de la Leptospirosis?


R = PENICILINA, estreptomicina o tetraciclinas
TULAREMIA
Cual es el agente causal de Tularemia?
R = FRANCISELLA TULLARENSIS POR PICADURA DE GARRAPATA
Cual es el cuadro clnico de Tularemia?
R = De inicio sbito con FIEBRE, ASTENIA, ADINAMIA, hiporexia, ataque al
estado general, posteriormente hay diversas formas clnicas:
ULCEROGANGLIONAR, manifestada con adenomegalias y ulceras cutneas;
TIFOIDE GLANDULAR, donde se afecta el pulmn, ganglios, hgado y bazo.
SE PRESENTA CON NEUMONA, NECROSIS VISCERAL Y SEPSIS.

Como diagnosticas Tularemia?


R = En medio de CULTIVO DE AGAR SANGRE cistena o ELISA.
Como tratas la Tularemia?
R = ESTREPTOMICINA, tetraciclina.
RICKETSIA
FIEBRE Q
LEISHMANIASIS
Que vectores transmiten leishmaniasis?
R = MOSQUITOS del genero LUTZOMYA Y PHLEBOTOMUS, LEISHMANIA pertenece a la familia
del tripanosoma

Cual es el ciclo biolgico de la leishmaniasis?


R = Al picar el mosquito a un mamfero infectado, ingiere los macrfagos parasitados, que
posteriormente se destruyen, dejando libre al parasito en su forma flagelar e infectante
(promastigote), se reproducen por fision binaria longitudinal, y migran hacia la porcin anterior
del aparato digestivo, hasta la probocide (boca). La infeccin se produce al alimentarse de un
nuevo husped, inocula los PROMASTIGOTES, que al ser fagocitados se transforman en
AMASTIGOTES, donde se multiplican, causan la lisis de las clulas que los contienen e infectan a
otras clulas.

En la leishmaniasis viceral o Kala-Azar a que clulas afecta el parasito?


R = A los MACRFAGOS del sistema retculo endotelial

Como se manifiesta la leishmaniosis cutnea?


R = En el sitio de inoculacin, PAPULA ERITEMATOSA/PRURIGINOSA NODULO ULCERA PIEL/TEJ.
CEL. SUB EXUDA LIQ. SEROHEMATICO ADENOPATA REGIONAL.

Como se manifiesta la leishmaniosis mucocutanea o espundia?


R = ULCERA CUTNEA pasan years LESIN OBSTRUCTIVA NICA/MULTIPLE
LARINGE/TABIQUE NASAL/ANO Y VULVA.
LEISHMANIASIS
Como se manifiesta la leishmaniasis viceral o Kala-Azar (fiebre negra)?
R = Hay HIPERPIGMENTACIN CUTNEA DE LAS EXTREMIDADES, mal estar general, DOLOR ABDOMINAL, HEPATO-
ESPLENOMEGALIA, LINFADENOPATIAS, DIARREA es fatal si no se trata adecuadamente.

Como diagnosticas leishmaniasis?


R = VER EL AMASTIGOTE EN BIOPSIAS DE PIEL, lesiones mucosas, hgado, ganglios linfticos, medula sea tenidas con
Giemsa. En los CULTIVOS de los tejidos se busca el PROMASTIGOTE.

Con que prueba de laboratorio diagnosticas leishmaniasis?


R = INTRADERMORREACCIN DE MONTENEGRO y ELISA.

Cual es el tratamiento de la leishmaniasis viceral?


R = Para la FORMA VISCERAL el tratamiento DE ELECCIN ES ESTIBOGLUCONATO DE SODIO O ANTIMONIATO DE
GLUTAMINA IM POR 50 DAS.

Que medicamentos empleas en caso de que la leishmaniasis visceral no responda a los antiamoniales?
R = ANFOTERICINA LIPOSOMAL POR 5 DAS Y EL ISETHIONATO DE PENTAMIDINA.

Que medicamento utilizas en leishmaniasis cutnea?


R = ISETIONATO DE PENTAMIDINA, paromomicina APLICACIN TPICA 2 VECES AL DA POR 15 DAS.
GIARDIASIS
Como diagnosticas giardiasis?
R = Observacin de QUISTE EN HECES, ELISA.

Que componente de los trofozoitos de giardiasis te causa


hipotrofia de las vellosidades intestinales
R = El DISCO SUCTOR

Cual es el tratamiento de eleccin para la giardiasis?


R = El TINIDAZOL con dosis nica de 2 gr. METRONIDAZOL. La
furazolidona causa hemolisis en pacientes con dficit de 6-
GDP.
AMEBOSIS
Cual es el cuadro clnico abdominal de la amebiasis por E. Hystolitica?
1. COLITIS AMEBIANA AGUDA manifestada por dolor abd tipo clico, tenesmo y
EVACUACIONES DIARREICAS CON MOCO Y SANGRE.
2. COLITIS FULMINANTE con mortalidad mayor al 50% se presenta con diarrea sanguinolenta
intensa, ataque al estado general, fiebre, dolor abdominal intenso y PERFORACIN
INTESTINAL con choque sptico secundarios a la extensin de la ulcera hasta la serosa.
3. AMEBOMA O GRANULOMA AMEBIANO que cursa con DIARREA SANGUINOLENTA Y
OBSTRUCCIN INTESTINAL.

Cual es el cuadro clnico de la amebosis cutnea?


1. ULCERAS FUGEDENICAS (de COLOR PURPURA, de crecimiento rpido, MUY DOLOROSAS y
ABUNDANTE MATERIAL NECRTICO),
2. LESIONES VEGETANTES (friables, de evolucin subaguda, LOCALIZADAS EN LOS PLIEGUES
CUTNEOS),
3. ENTAMOEBOSIS (se presenta por una reaccin de hipersensibilidad, CON LESIONES
SEMEJANTES A URTICARIA.

Como se manifiesta el absceso heptico amebiano?


R = De inicio agudo, CON HEPATOMEGALIA Y DOLOR EN HIPOCONDRIO DERECHO QUE SE
IRRADIA HACIA EL HOMBRO.
AMEBOSIS
Como diagnosticas amibiasis?
R = COPROPARISOTOSCOPICO EN FRESCO, hematoxilina frrica o TINCIN
DE GOMORI, en busca de TROFOZOITOS, ELISA. En la piel se busca
trofozoito.

Cual es el tratamiento de amibiasis?


1) METRONIDAZOL, TINIDAZOL.
2) Dehidrohemetina y emetina son amebicidas de la pared intestinal, pero no
sobre la luz intestinal siendo estos YODOQUINOL y paramomicina.

Que medicamento es eficaz para la amebiasis heptica?


R = CLOROQUINA
TRIPANOSOMIASIS AFRICANA O
ENFERMEDAD DEL SUENO
Cual es el agente etiolgico de la TRIPANOSOMIASIS AFRICANA O ENFERMEDAD DEL
SUEO y cual es su vector?
R = Tripanosoma BRUCEI, Tripanosoma RHODESIENSE y Tripanosoma GAMBIENSE. Su
vector es la MOSCA TSE TSE.

Cual es el agente causal de la tripanosomiasis americana o ENFERMEDAD DE CHAGAS


y cual es su vector?
R = TRIPANOZOMA CRUZI y su vector es la chinche

Cual es el ciclo biolgico de los tripanosomas?


R = Al alimentarse de un individuo con la infeccin, los vectores ingieren el amastigote,
que se multiplica dentro del tracto digestivo y las FORMAS INFECCIOSAS
(TRIPOMASTIGOTE) son eliminadas en las heces; la infeccin se produce al depositar las
heces en una herida de la piel antes de picar en las membranas mucosas o en la
conjuntiva. Posteriormente, se multiplican en el sitio de la entrada, y entran a la sangre
como tripomastigotes circulantes e invaden a las clulas, CON PREDILECCIN POR EL
MIOCARDIO, MUSCULO LISO Y SNC.
TRIPANOSOMIASIS O ENFERMEDAD
DE CHAGAS
Como se manifiesta la tripanosomiasis o ENFERMEDAD DE CHAGAS POR T. CRUZY?
R = En la piel, en el sitio de entrada se observa CHAGOMA que es PARECIDO A LA FURUNCULOSIS CON
LINFADENOPATIA REGIONAL. En conjuntiva, edema bipalpebral unilateral, CONJUNTIVITIS Y
LINFADENITIS PRE AURICULAR (SIGNO DE ROMANA) CON FIEBRE Y ESPLENOMEGALIA LEVE.
CRONICAMENTE se manifiesta por INSUFICIENCIA CARDIACA PREDOMINANTE DEL LADO DERECHO, con
arritmias ventriculares y episodios de tromboembolia sistmica o pulmonar. MEGA ESFAGO Y
MEGACOLON con sntomas de disfagia, constipacin intensa, hipertrofia parotidea.

Como se diagnostica la tripanosomiasis americana o ENFERMEDAD DE CHAGAS?


R = En la FASE AGUDA se observa con la TINCIN DE GIEMSA de sangre anti coagulada. Diagnostico
serolgico se basa en hemaglutinacin indirecta o ELISA.

Como se diagnostica la TRIPANOSOMIASIS AFRICANA?


R = Para T. GAMBIENSE por medio de PRUEBAS DE AGLUTINACIN y para T. RHODESIENSE es mediante
la observacin del parasito en la tincin de GIEMSA.

Cual es el tratamiento de eleccin para la TRIPANOSOMIASIS AMERICANA O CHAGAS?


R = NIFURTIMOX por 30-90 DAS o BENZIDIMAZOL de 30-90 DIAS.
Cual es el tratamiento de la TRIPANOSOMIASIS AFRICANA?
R = Para T. GAMBIENSE ES PENTAMIDINA POR 10 DAS. Para T. RHODESIENSE ES LA SURAMINA
POR 7 DAS.
TENIASIS
Cual es el agente causal de la teniasis?
R = T. saginata y T. solium

Cuales son las partes estructurales que componen a la tenia?


R = En su extremo anterior tienen una estructura llamada ESCLEX (CABEZA), seguida de un
cuello. En su esclex presentan cuatro ventosas orales. En sus ventosas tiene un rstelo, con
una corona doble de ganchos (escolices). El CUERPO (ESTRBILO) se conforma por varios
segmentos llamados PROGLOTIDOS, que en su interior tienen RGANOS SEXUALES.

Cuanto miden los adultos de T. SAGINATA?


R = 2-7 METROS de largo

Cuanto puede llegar a medir la T. SOLIUM?


R = 15-18 METROS

Cual es el ciclo biolgico de la teniasis?


R = La teniasis intestinal es causada por la INGESTA DE CISTICERCOS. Al legar a intestino
delgado, se unen a la pared intestinal. De los 2 a los 4 meses de la infeccin, alcanzan su
madurez, caracterizada por la presencia de proglotidos gravidos que liberan huevos y se
excretan por las heces. La cisticercosis ocurre por la ingestin de los alimentos o agua
contaminados con excremento humano que contiene huevos o proglotidos. Los huevos se
abren en el intestino, liberando a las oncoesferas, que invaden la pared intestinal pasando al
torrente sanguneo y linftico para migrar hacia musculo estriado, cerebro e hgado.
TENIASIS
Cual es el cuadro clnico de la teniasis?
R = Normalmente es asintomtica. Cuando cursa con sntomas son:
PLENITUD ABDOMINAL, FATIGA, DOLOR LEVE OCASIONAL E HIPOREXIA.

Que dficit vitamnico cursa con teniasis?


R = Dficit de VITAMINA B12

Cual es el cuadro clnico de CISTICERCOSIS en el SNC?


R = CRISIS CONVULSIVAS, cefaleas, vomito, HIPERTENSIN INTRACRANEAL.

Como diagnosticas la teniasis?


R = COPROPARASITOSCOPICO SE OBSERVAN HUEVOS, ELISA.

Cual es el tratamiento para teniasis?


R = PRAZICUANTEL a DOSIS NICA DE 10 MG/KG. Ninguna droga mata a
los huevos de tenia solium. NICLOSAMIDA para TENIA SAGINATA
NEUROLOGIA

ENARM
MANIFESTACIONES DE LESION AL SNC
Como se muestra una lesin del TRACTO PTICO?
R = Como CUADRANTOPSIA homnima contralateral.

Que caracteriza al sndrome de Horner?


R = Miosis, ptosis y anhidrosis debido a lesin de las vas del SISTEMA SIMPTICO ispolaterales.
1) Miosis: Se debe a denervacin simptica del musculo tarsal
2) Anhidrosis: Consecuencia de la denervacin simptica del musculo tarsal
3) Para localizar la lesin se realizan pruebas con cocana, hidroxifentamina y fenilefrina.

Que ocasiona las pupilas de Argyll Robertson?


1) Son PUPILAS IRREGULARES Y MS PEQUEAS de lo normal en la oscuridad.
2) Demuestran falta del reflejo fotomotor, tanto directo como consensual, sin embargo conservan la
acomodacin.
3) Se asocia a NEUROSIFILIS.

Que ocasiona las pupilas de Holmes-Adie?


1) Se trata de una PUPILA GRANDE Y TNICA de forma irregular
2) Tiene reaccin pobre ante estimulo luminoso
3) Suele indicar una lesin de las vas parasimpticas posganglionares, a nivel del ganglio ciliar o los nervios
ciliares cortos.
4) Se ve una REACCIN CONSTRICTORA EXAGERADA ante la administracin de agonistas muscarinicos como la
meticolina o la PILOCARPINA.
5) Se asocian a reflejos de estiramiento muscular disminuidos o ausentes.
MANIFESTACIONES DE LESION AL SNC
Cuales son las manifestaciones oculares en caso de obstruccin de las RADIACIONES
PTICAS?
R = CUADRANTOPSIAS HOMONIMAS

-A 75-year-old woman presents with symptoms of visual change and facial weakness. On
examination, the pupils are equal and reactive to light, the fundi appear normal, and there is
a right homonymous visual field defect. Which of the following is the most likely cause of the
right homonymous hemianopia?

(A) right optic nerve


(B) chiasm
(C) right optic radiations
(D) right occipital lobe
(E) left optic radiations

- The hemianopia is due to a lesion of the left optic radiations. The posterior cerebral artery
arises from the basilar artery but is sometimes a branch of the internal carotid. With
posterior cerebral artery lesions affecting the occipital cortex, it is possible for the
hemianopia to be an isolated finding.
CORRELACIN TOPOGRFICA DE DEFECTOS DEL CAMPO VISUAL. A: abolicin CV izquierdo (lesin nervio ptico izquierdo). B: escotoma
yuncional. C: hemianopsia bitemporal heternima (lesin quiasmtica). D: hemianopsia homnima derecha (lesin quiasmtica). E:
sectoronopsia cudruple homnima derecha (lesin ganglio geniculado izquierdo). F: Sectoranopsia horizontal derecha (lesin ganglio
geniculado izquierdo). G: Cuadrantanopsia homnima inferior derecha (lesin lbulo parietal izquierdo). H: Cuadrantanopsia homnima
superior derecha (lesin lbulo temporal izquierdo). I: Hemianopsia homnima derecha con respeto macular (lesin occipital izquierda).
MANIFESTACIONES DE LESION AL SNC
Que es la disautonomia?
R = Proceso patolgico central o perifrico que se manifiesta por anomalas de la regulacin
de la TA, diaforesis termorreguladora, funcin sexual.

Que es el mioclono?
1) Es comn en epilepsia, presentando ESPASMOS. Se presenta en casos de anoxia,
hiperuricemia.
2) El tratamiento es con levodopa, privacin alcohlica y responde a acido valproico y
benzodiacepinas.

Que motoneurona se ve afectada en la espasticidad?


R = MOTONEURONA SUPERIOR

Cual es tratamiento de espasticidad?


R = Fisioterapia, dantroleno debilita la contractura muscular al interferir con el calcio,
diazepam, inyeccin de fenol para disminuir la espasticidad selectiva o toxina botulnica.
MANIFESTACIONES DE LESION AL SNC
En que zona se encuentra la lesin en un paciente estuporoso/comatoso en ausencia
unilateral de respuesta a pesar de la aplicacin de estmulos en ambos lados del cuerpo?
R = CORTICOESPINAL

En que zona se encuentra la lesin en paciente en ausencia bilateral de respuesta a


estmulos?
R = Afeccin de TALLO ENCEFLICO, lesin bilateral de va piramidal

La postura de decorticacion a que dao se debe?


R = Lesin de CAPSULA INTERNA y del PEDNCULO CEREBRAL ANTERIOR.

La postura de decerebracion a que dao se debe?


R = DESTRUCCIN MESENCFALO y de la porcin anterior del puente de vacilio

Que causa el sndrome de encerramiento o estado desaferentado?


1) Lesiones agudas que afectan principalmente la PARTE ANTERIOR DEL PUENTE DE VACILIO Y
RESPETAN EL TEGUMENTO.
2) Tienen respuesta ocular y estn consientes pero en estado comatoso y se RECUPERAN DE
2-3.
Posicin Decorticacin Posicin Descerebracin
MOTONEURONAS
Que caracteriza a las lesiones de la motoneurona?
R = Debilidad sin perdida sensorial

Cual es la clasificacin de la enfermedad de motoneurona y que caracteriza a cada una de ellas?


1) Parlisis bulbar progresiva: degeneracin que predomina en ncleos motores de pares
craneales
2) Parlisis pseudobulbar: se debe a enfermedad cortico bulbar alterando neurona motora
superior
3) Atrofia muscular espinal progresiva: Hay degeneracin de las astas anteriores de la medula
espinal afectando a neurona motora inferior
4) Esclerosis lateral primaria: Dficit puro de neurona motora superior en extremidades inferiores
5) Esclerosis lateral aminotrofica: Dficit mixto de NMS e INFERIOR en extremidades pudiendo
alterar el estado cognitivo con DEMENCIA FRONTOTEMPORAL O PARKINSONISMO.

Cual es el cuadro clnico de enfermedad degenerativa de neuronas motoras?


1) Alteracin bulbar: dificultad para la deglucin, masticacin, tos, respiracin y disartria.
2) Parlisis bulbar progresiva: descanso del paladar disminuyendo reflejo nauseoso, tos dbil y
desgaste muscular. Lingual con fasciculaciones.
3) Parlisis pseudobulbar: Contractura y espasmo lingual, disminucin de la motilidad de la
lengua. Hay trastorno motor en extremidades con esfnteres normales
4) ESCLEROSIS LATERAL AMINOTROFICA: ES LETAL de 3-5 aos comnmente POR INFECCIN
PULMONAR, presentndose con periodos de REMISIN Y EXACERBACIN DE DIFICULTAD
PARA CAMINAR, TRAGAR , TORPEZA Y FINALMENTE RESPIRAR CON CARCTER
AUTOSOMICO DOMINANTE EN GEN 21.
MOTONEURONAS
Cuales son los signos positivos y negativos que indican lesin a la MOTONEURONA SUPERIOR o sndrome piramidal?
1) Positivos: ESPASTICIDAD, HIPERREFLEXIA en reflejos de estiramiento muscular, clonus, distona, atetosis y exaltacin de
reflejos cutneos como signo de Babinski
2) Negativos: Debilidad o paresia hemicorporal y prdida de la destreza, coordinacin y control motor.
3) La combinacin de ambos afectan la ejecucin normal del movimiento con un deterioro en la calidad de vida del paciente.

Cual es el cuadro clnico del sndrome de la MOTONEURONA INFERIOR?


R = ATROFIA muscular por denervacin, parlisis o PARESIA FLCIDA de todos los msculos de la unidad motora afectada,
HIPOTONA, hiporreflexia o ARREFLEXIA al interrumpirse la parte eferente del arco reflejo y fasciculaciones producidas por la
actividad espontanea de la unidad motora.

Cuales son las caractersticas clnicas del sndrome de la columna posterior?


1) Se caracteriza por perdida o DISMINUCIN DE LA SENSACIN DE VIBRACIN, SENSACIN DE POSICIN, DISCRIMINACIN DE
DOS PUNTOS, TACTO Y RECONOCIMIENTO DE LA FORMA
2) Estimulos que no eran dolorosos SE CONVIERTEN EN DOLOROSOS y LOS ESTMULOS DOLOROSOS SE DESENCADENAN CON
UMBRALES DE ESTIMULACIN MAS BAJOS.

Que caracteriza al sndrome de la arteria espinal anterior?


R = Tambin conocido como sndrome medular ventral que ocasiona PARAPLEJIA O CUADRIPLEJIA, perdida bilateral de la
sensacin de dolor y temperatura por debajo del nivel de la lesin.

Que estudios realizas para diagnosticar la enfermedad de la motoneurona?


R = ELECTROMIOGRAFA: muestra DENERVACIN, EN ESCLEROSIS LATERAL AMINOTROFICA debe presentarse en 3
extremidades mnimo. LINFOMA DE HODKING puede cursar con NEUROPATA MOTORA.

Cual es el tratamiento de lesiones de neuronas motoras?


R = RICUZOL: disminuye la LIBERACIN PRESINAPTICA DE GLUTAMATO haciendo lenta la ELA progresiva.
PLASMAFERESIS logra mejora. ANTICOLINERGICOS: atropina y amitriptilina
MOTONEURONAS
- A 53-year-old man complains of clumsiness with both hands, like having difficulty doing up buttons or using his keys. Physical
examination reveals fasciculations of his thigh and forearm muscles; diffuse muscle weakness, loss of muscle bulk, and
increased tone in the upper and lower limbs. There is generalized hyperreflexia, and positive Babinski signs bilaterally. Which of
the following is the most likely natural progression of this condition?

(A) a long history of remissions and exacerbations (sclerosis lateral aminotrofica)


(B) sensory loss in the distribution of peripheral nerves
(C) focal seizures
(D) a progressively downhill course
(E) cogwheel rigidity

- This man has amyotrophic lateral sclerosis (ALS). The disease causes neuronal loss in the anterior horns of the spinal cord and
motor nuclei of the lower brain stem. The disease is one of constant progression, rather than remissions and exacerbations, and
death usually occurs within 5 years. There is no sensory loss and no seizure diathesis, because only the motor system is
involved. There can be signs ofhyperreflexia and spasticity, depending on the balance of upper and lower motor neuron
damage, but not cogwheel rigidity.

-A 45-year-old man presents with weakness and fasciculations in his arms and legs. His cranial nerves are normal, but there is
weakness of his left handgrip and right leg quadriceps with loss of muscle bulk. There are obvious fasciculations over the left
forearm and right thigh. Tone is increased in the arms and legs and the reflexes are brisk. Which of the following is the most
likely diagnosis?

(A) amyotrophic lateral sclerosis (ALS)


(B) myotonic muscular dystrophy
(C) amyotonia congenita
(D) tabes dorsalis
(E) migraine

- The most common initial symptom of ALS is weakness and wasting of the extremities. The fasciculations can be a very
prominent part of the disease. This is rare in other neurologic disorders.
ESLEROSIS LATERAL AMINOTROFICA
Que es la esclerosis lateral aminotrfica?
1) Se refiere a una condicin neurologa de INICIO EN LA VIDA ADULTA
caracterizada por una DEGENERACIN PROGRESIVA DE LAS NEURONAS
MOTORAS SUPERIORES E INFERIORES.
2) La degeneracin de las neuronas de la corteza motora se manifiesta como
hiperreflexia, signos positivos de Hoffman, Babinski y clonus.
3) El termino esclerosis lateral se refiere a la DUREZA CUANDO SE PALPAN LAS
COLUMNAS LATERALES DE LA MEDULA ESPINAL en las autopsias de pacientes
con esta enfermedad.
4) El termino aminotrfica se refiere a la ATROFIA MUSCULAR, DEBILIDAD Y
FASCICULACION debidas a la degeneracin de las neuronas motoras inferiores.

Que hallazgos muestra la electromiografa al respecto de la esclerosis lateral


aminotrfica?
R = Demuestra DENERVACIN de POR LO MENOS 3 EXTREMIDADES.

Cual es el manejo de la esclerosis lateral aminotrfica?


R = RILUZOLE QUE ES UN ANTAGONISTA DEL GLUTAMATO Y LA VITAMINA E.
Deben ser tratados como si tuvieran lesin de la columna cervical hasta que se
demuestre lo contrario.
CHARCOT-MARIE-TOOTH
Cuales son las caractersticas de la mononeuropatia
hereditaria de Charcot-Marie-Tooth?
1) Carcter AD en CROMOSOMA 17.
2) Clnicamente con DEFORMIDADES DEL PIE,
TRASTORNOS DE LA MARCHA.
3) El electrodiagnstico muestra DISMINUCIN DE LA
CONDUCCIN MOTORA Y SENSITIVA (neuropata
motora y sensitiva hereditaria tipo I) NMSH tipo I.
4) NMSH tipo II disminuye solo un poco.
5) Histolgicamente hay PERDIDA AXONAL.
TRAUMATISMO
Que te indica el signo de mapache?
R = Equimosis en parpados debido a fractura de base del craneo por TCE

Que maniobra resulta til en el TCE que causa salida de LCR por nariz u odos (contiene glucosa)?
R = CONSERVADOR, con ELEVACIN DE LA CABEZA, RESTRICCIN DE LQUIDOS, administracin de
acetazolamida EN CASO DE PERSISTENCIA SE REALIZA DRENAJE LUMBAR SUBARACNOIDEO

Cual es el cuadro clnico de traumatismo en medula espinal?


1) La transeccin medular completa resulta en PARLISIS FLCIDA INMEDIATA Y PERDIDA DE LA SENSIBILIDAD
DEBAJO DE LA LESIN.
2) Se pierde retencin urinaria y fecal.
3) Al final hay paraplejia en las piernas en flexin o extensin.
4) En las LESIONES MENORES los pacientes que dan con DEBILIDAD LEVE DE LAS EXTREMIDADES, TRASTORNOS
SENSITIVOS DISTALES O AMBOS.

Que caracteriza al sndrome de BROWN-SEQUARD?


R = Es una LESIN UNILATERAL de la MEDULA ESPINAL que da lugar a un TRASTORNO MOTOR IPSOLATERAL con
deterioro concomitante de la PROPIOSEPCION y PERDIDA CONTRALATERAL de las apreciaciones del DOLOR Y
TEMPERATURA

Cual es el manejo del traumatismo medular?


R = INMOBILIDAD. GRANDES DOSIS DE ESTEROIDES metilprednisolona 300 mg en bolo seguidos por 5.4
mg/kg/hr x 24 hrs.
TRAUMATISMO
Cual es la principal etiologa de un HEMATOMA SUBDURAL y sus principales caractersticas
clnicas?
R = A subdural hematoma is almost always of VENOUS ORIGIN and secondary to a minor or
severe INJURY TO THE HEAD, but may occur in blood dyscrasias or cachexia in the absence of
trauma. ACUTE subdural hematomas commonly present with a FLUCTUATING LEVEL OF
CONSCIOUSNESS and significant cerebral damage. CHRONIC subdurals may also present with
SEIZURES OR PAPILLEDEMA

-A 94-year-old man presents with progressive headaches, light-headedness, drowsiness, and


unsteady gait over 6 weeks. On examination, his blood pressure is 160/90 mm Hg, pulse
70/min, lungs clear, and he has no focal weakness. His gait is unsteady but sensation in the
feet is normal. A CT scan reveals a hyperintense clot over the left cerebral cortex. Which of
the following is the most likely cause for this clot?

(A) is venous in origin


(B) is arterial in origin
(C) is from injury to the middle meningeal artery
(D) is from a subarachnoid hemorrhage
(E) is from injury to the middle cerebral artery
ESCALA DE GLASGOW MODIFICADA (NIOS)

PRUEBA RESPUESTA PUNTUACIN


Espontanea 4
APERTURA DE A rdenes 3
OJOS Al estimulo doloroso 2
Nula 1
Palabras apropiadas y sonrisas,
LLANTO COMO fija la mirada y sigue los objetos. 5
Tiene llanto, pero consolable. 4
RESPUESTA Persistente e irritable 3
VERBAL Agitado. 2
Sin respuesta. 1
Obedece ordenes 6
Localiza el dolor 5
RESPUESTA Retirada ante el dolor 4
MOTORA Flexin inapropiada 3
Extensin 2
Nula 1
Si EGC: 15-----------------------Consciente
Si EGC: 13-14-------------------Estupor ligero
Si EGC: 11-12-------------------Estupor moderado
Si EGC: 9-10--------------------Estupor profundo
Si EGC: 7-8---------------------Coma superficial
Si EGC: 5-6---------------------Coma moderado
Si EGC: 3-4---------------------Coma profundo
REACTIVIDAD PUPILAR.
Mioticas Dimetro < 2mm
Segn el
Medias Dimetro 2-5mm
tamao
Midriaticas Dimetro > 5mm
Isocoricas Iguales
Segn relacin
Anisocoricas Desiguales
entre ellas
Discoricas Forma irregular
Segn Reactivas Contraccin al foco
respuesta luminoso
a la luz Inmviles al foco
Arreactivas
luminoso
SNDROME DEL TNEL CARPIANO
A que se asocia el sndrome del tnel carpiano?
R = HIPOTIROIDISMO, EMBARAZO, diabetes, AR o ACROMEGALIA.

Cuales son las manifestaciones clnicas del sndrome del tnel carpiano?
1) HORMIGUEO, dolor, ENTUMECIMIENTO EN LA MANO, sobre todo al
comienzo de la maana y en ocasiones con DEBILIDAD DE LA
MUSCULATURA TENAR con atrofia del abductor corto del pulgar.
2) Puede observarse el SIGNO DEL TINEL que se provoca con un golpesito en
el tnel carpiano REPRODUCIENDO DOLOR Y HORMIGUEO.

Como dx el sx del tnel carpiano?


R = CLNICA + ELECTROMIOGRAFA.

Cual es el manejo del sx del tnel carpiano?


R = Descompresin quirrgica.
COMPRESIN DEL NERVIO CUBITAL

Como se manifiesta clnicamente la compresin del nervio cubital?


1) ADORMECIMIENTO EN EL QUINTO DEDO y en el lado MEDIAL de la mano.
2) El dolor y las parestesias pueden iniciar en el codo e irradiarse a la mano.
3) LAS PARESTESIAS SE EXACERBAN CON LA FLEXIN DEL CODO.

Como diagnosticas la compresin del nervio cubital?


R = Electrofisiologa.

Cual es el manejo de la compresin del nervio cubital?


R = Los ESTEROIDES Y LOS AINES. Se recomienda ciruga.
NERVIO CUBITAL
Cual es la sintomatologa en caso de dao al nervio cubital?
R = Injury to the ulnar nerve results in impaired adduction and abduction of the
fingers. The nerve is commonly injured in ELBOW DISLOCATIONS and fractures. The
fibers arise from the eighth cervical and the first thoracic segments. The ulnar is a
mixed nerve with sensory supply to the medial hand.

- A 40-year-old man is injured in a car accident and fractures his left elbow. He now
complains of numbness of his fourth and fifth fingers, and weakness in his hand
grip. Neurologic findings confirm weakness of handgrip with weakness of finger
abduction and adduction, and decreased sensation over the fifth finger and lateral
aspect of fourth finger. Which of the following is the most likely diagnosis?

(A) ulnar nerve injury


(B) radial nerve injury
(C) median nerve injury
(D) carpal tunnel syndrome
(E) axillary nerve injury
COMPRESIN DEL NERVIO RADIAL

Como se manifiesta la compresin del nervio


radial?
1) Parlisis de la noche del sbado por dejar
el brazo colgando despus de una
borrachera.
2) MUECA FLCIDA y debilidad a la extensin
de los dedos.
CIATICO Y TARSO
Cual es la etiologa mas comn de la parlisis del
citico?
R = INYECCIONES INTRAMUSCULARES profundas

Que sintomatologa da el sndrome del tnel del


tarso?
R = NERVIO TIBIAL, rama del citico, cuando se
comprime da origen a DOLOR, PARESTESIAS Y
ENTUMECIMIENTO SOBRE BASE DEL PIE
PREDOMINANDO DURANTE LA NOCHE sin afectar
taln.
MERALGIA PARESTESICA
Que causa y como se manifiesta clnicamente la meralgia
parestesica?
1) Se debe al atrapamiento del NERVIO CUTNEO LATERAL DEL
MUSLO POR DEBAJO DEL LIGAMENTO INGUINAL.
2) Se produce DOLOR URENTE, PARESTESIAS Y
ENTUMECIMIENTO DE LA CARA ANTEROLATERAL DEL
MUSLO.
3) Se presenta en pacientes con EXCESO DE PESO por lo que el
TRATAMIENTO ES BAJAR DE PESO.
4) Nervio FEMOROCUTANEO EXTERNO, se origina en RACES
L2 Y L3 EL CUAL SE PRESIONA EN CASO DE EMBARAZO,
DIABETES Y OBSESOS.
LESIONES DE LOS PARES CRANEALES
Que datos clnicos se encuentran en el dao al III (OCULOMOTOR) PAR CRANEAL?
1) Frecuentemente causa ANISOCORIA, en la que la pupila mas dilatada denota al LADO AFECTADO.
2) El fenmeno de MARCUS GUNN consiste en que con el ESTIMULO LUMINOSO NO OBTENEMOS
RESPUESTA EN EL LADO AFECTADO despus de presentar contraccin por efecto de reflejo consensual
al estimular contralateralmente
3) Suele deberse a CAUSAS HIPOXICAS
4) En pacientes con compresin cerebral se mostrara midriasis ipsolateral al sitio afectado como
resultado de compresin del III par craneal en cualquier parte de su recorrido debido a que las fibras
que inervan el esfnter de la pupila discurren en porcin perifrica del nervio.
5) PTOSIS PALPEBRAL CONTRALATERAL a la lesin.
6) La lesin en este par craneal afecta los MSCULOS ELEVADOR DEL PARPADO, recto medial, recto
superior, recto inferior y oblicuo inferior (TODOS MENOS EL RECTO EXTERNO Y EL OBLICUO
SUPERIOR), lo que provoca que los movimientos de ADUCCIN, ELEVACIN Y DEPRESIN DEL OJO
IPSOLATERAL ESTN AFECTADOS.
7) Lo anterior provoca que cuando el paciente este de frente el ojo tiende a DESVIARSE HACIA ABAJO Y
AFUERA.

Que datos clnicos se encuentran en el dao al IV (PATTICO O TROCLEAR) PAR CRANEAL?


1) Presentan debilidad del MUSCULO OBLICUO SUPERIOR
2) DIPLOPIA CON ADUCCION Y HACIA ABAJO

Que datos clnicos se encuentran en el dao al VI (OCULOMOTOR EXTERNO) PAR CRANEAL?


1) Afecta al MUSCULO RECTO LATERAL
2) El ojo afectado se dirige A LA LINEA MEDIA CON INCAPACIDAD DE ABDUCIR EL OJO.
LESIONES DE LOS PARES CRANEALES
Que datos clnicos se encuentran en el dao al V (TRIGMINO) PAR CRANEAL?
1) La parlisis de la primera divisin (OFTLMICA) involucra PERDIDA DE LA SENSIBILIDAD EN LA FRENTE
junto con parlisis del III par y IV.
2) La parlisis de la segunda divisin (MAXILAR) resulta en la PERDIDA DE SENSIBILIDAD DE LA MEJILLA y se
debe a lesiones del seno cavernoso.
3) La parlisis de la tercera divisin (MANDIBULAR) resulta en DFICIT EN LA MASTICACIN

Que cuadro clnico resulta del sndrome ocasionado por la afeccin del ganglio de Gasser?
R = Sndrome caracterizado por DOLOR FACIAL, CEFALEA, PERDIDA DE LA SENSIBILIDAD SENSORIAL, SORDERA Y
PARLISIS DEL VI Y VII PARES. EL DOLOR AUMENTA AL MOVER LA MANDBULA.

Cual es el cuadro clnico resultante de la neuralgia del glosofarngeo?


R = Es un trastorno unilateral que afecta principalmente a V3 presentndose en forma de ESPASMOS
DOLOROSOS que pueden durar de segundos a minutos y pueden estar asociados a estmulos cutneos y
auditivos.

Cual es la etiologa de la parlisis de Bell?


R = Se asocia a HERPES VIRUS

Que caractersticas clnicas tiene la parlisis de Bell?


1) PARLISIS DEL VII PAR CON INICIO SBITO DE DEBILIDAD FACIAL y prdida del reflejo acstico.
2) Hay PRDROMO VIRAL y adormecimiento o dolor en el odo, lengua o cara.
3) La CUERDA DEL TMPANO SE VE ERITEMATOSA y HAY REDUCCIN EN LA LAGRIMACIN IPSOLATERAL o
salivacin.
4) El paciente no puede cerrar los ojos por lo que se debe evitar el dao a la cornea
5) Se diagnostica con ELECTROMIOGRAFA denotndose una DISMINUCIN EN LA VELOCIDAD DE CONDUCCIN
LESIONES DE LOS PARES CRANEALES
Cual es el manejo de la parlisis de Bell?
R = VALACICLOVIR o famciclovir y PREDNISONA

Cual es la caracterstica clnica de la afectacin del VIII (VESTIBULOCOCLEAR) par craneal?


1) Porcin coclear: El principal sntoma asociado a la lesin o atrofia de esta porcin es la HIPOACUSIA
SENSORINEURAL. Se realiza el diagnostico por AUDIOMETRA.
2) Porcin vestibular: VRTIGO POSTURAL PAROXSTICO BENIGNO.

Que caractersticas clnicas se observan por dao al XI (ESPINAL ACCESORIO) par craneal?
R = Dolor significativo acompaado de una DISMINUCIN DE LA MOVILIDAD DEL HOMBRO, siendo la
lesin iatrogena la mas comn (p.e la BIOPSIA DE LOS GANGLIOS LINFTICOS).

Cuales son los datos clnicos que involucra una lesin en los pares craneales IX Y X (GLOSOFARNGEO Y
VAGO)?
1) DISFAGIA, parlisis de la cuerda vocal ipsolateral, as como DEBILIDAD DEL PALADAR Y LA FARINGE.
2) La ELEVACIN DEL PALADAR ES ASIMTRICA y la VULA SE DESVA AL LADO CONTRARIO DE LA
LESIN.
3) Se pierde la capacidad sensorial de UN TERCIO POSTERIOR DE LA LENGUA, con disminucin en la
sensibilidad en la faringe posterior y laringe.
4) La LESIN ms comn del VAGO es la que involucra al NERVIO LARNGEO RECURRENTE, lo que
provoca PARLISIS DE LA CUERDA VOCAL IPSOLATERAL Y SUBSECUENTE DISFONA.
5) La neuralgia del glosofarngeo tiene CARACTERSTICAS SIMILARES A LA NEURALGIA DEL TRIGMINO,
pero es MAS EVIDENTE EN LA LENGUA, LAS AMGDALAS, ODO Y ANGULO DE LA MANDIBULA
LESIONES DE LOS PARES CRANEALES
Que caractersticas clnicas tiene el dao al XII (HIPOGLOSO) par craneal?
R = DEBILIDAD en la lengua, FASCICULACIONES y DESVIACIN DE LA LENGUA hacia el lado de la lesin.

Que medicamento es de eleccin en neuralgia del trigmino y del glosofarngeo?


R = CARBAMACEPINA, pero tambin puede incluir baclofen y fenitoina.

- A 52-year-old man complains of episodes of severe unilateral, stabbing facial pain that is intermittent for
several hours, and then disappears for several days. The pain is describes as electric shock-like and only lasts
a few seconds. Physical examination of the face and mouth is entirely normal. Which of the following
treatments is most effective for this condition?

(A) morphine
(B) indomethacin
(C) cimetidine
(D) carbamazepine
(E) lidocaine (Xylocaine) gel

- This patient has trigeminal neuralgia. Carbamazepine (an anticonvulsant drug) is given in doses varying from
600 to 1200 mg/day. Phenytoin has also been used. The two drugs can also be used in combination. Operative
procedures include alcohol injection of the nerve or ganglion, partial section of the nerve in the middle or
posterior fossa, decompression of the root, and medullary tractotomy.Radiofrequency surgery can destroy pain
fibers but spare motor fibers.
VERTIGO
Cual es la diferencia entre vrtigo de origen central y de origen perifrico?
R = In CENTRAL VERTIGO, the vertigo can be MILD AND CHRONIC. In PERIPHERAL disease, THE SYMPTOMS ARE
GENERALLY MORE SEVERE, BUT FINITE (although often recurrent).

- A 63-year-old woman develops symptoms of nausea, vomiting, and dizziness, which she describes as a to-and-
fro movement of the room like as if she is on a boat. Which of the following findings suggests the vertigo is
central in origin?

(A) deafness
(B) symptoms are more protracted but less severe
(C) unidirectional nystagmus
(D) visual fixation inhibits vertigo and nystagmus
(E) spinning sensation is toward the fast phase of nystagmus

Cuales son las caractersticas del vrtigo de ORIGEN PERIFRICO?


R = TINNITUS AND DEAFNESS may be found in PERIPHERAL VERTIGO, but NOT CENTRAL. The nystagmus is
usually unidirectional and is never vertical. VISUAL FIXATION INHIBITS VERTIGO AND
NYSTAGMUS during testing in peripheral vertigo
- A 47-year-old man presents to the emergency room with symptoms of dizziness and difficulty walking. He
describes his dizziness as a spinning sensation of the room with associated nausea and vomiting. Which of the
following findings suggests the vertigo is peripheral in origin?

(A) optic neuritis


(B) tinnitus
(C) bidirectional nystagmus
(D) vertical nystagmus
(E) visual fixation does not affect vertigo or nystagmus
VERTIGO
Cual es la sintomatologa del vrtigo benigno posicional paroxstico y que maniobra utilizas
para demostrarlo?
R = Benign paroxysmal positional vertigo (BPPV), which is characterized by sudden-onset
brief episodes of vertigo LASTING LESS THAN A MINUTE. The symptoms are USUALLY
BROUGHT ON BY HEAD MOVEMENT. The cause is commonly attributed to CALCIUM DEBRIS
IN THE SEMICIRCULAR CANALS, KNOWN AS CANALITHIASIS. The debris is loose otoconia
(calcium carbonate) within the utricular sac. Although BPPV can occur after head trauma,
there is usually no obvious precipitating factor. IT GENERALLY ABATES SPONTANEOUSLY AND
CAN BE TREATED WITH VESTIBULAR REHABILITATION. A DIX-HALLPIKE MANEUVER
REPRODUCES HIS SYMPTOMS

- A 48-year-old man complains of recurrent episodes of sudden-onset dizziness. He notices


an abrupt onset of a spinning sensation when sitting up or lying down in bed. The symptoms
last for 30 seconds and then resolve completely. He has no hearing change or other
neurologic symptoms, and his physical examination is completely normal. A Dix-Hallpike
maneuver reproduces his symptoms. Which of the following is the most likely mechanism
for his vertigo symptoms?

(A) basilar migraine


(B) brain stem ischemic events
(C) benign cerebellar tumors
(D) calcium debris (calcium carbonate crystals) in the semicircular canals
(E) Menieres disease
EVC
Cual es la duracin de un ataque isqumico transitorio?
R = Dura < 24 HRS y el DFICIT CLNICO SE RESUELVE EN MENOS DE 1-2 HRS. Se
debe utilizar ASA COMO PROFILAXIS.

Cuanto debe medir un infarto lacunar y como se debe ver en la TAC?


R = Mide <5 MM de dimetro, en la TAC se observan como LESIONES PEQUEAS EN
SACABOCADO HIPODENSAS, con RESOLUCIN parcial o completa de 4-6 SEMANAS

Que sintomatologa da el EVC en arteria cerebral anterior?


R = DEBILIDAD y perdida sensitiva en la pierna contralateral y debilidad leve en
brazo.

Que cambios produce el infarto cerebral anterior?


R = CAMBIOS CONDUCTUALES y alteracin de la MEMORIA.

Que sintomatologa da la obstruccin de la arteria cerebral media?


R = AFASIA, HEMIPLEGIA CONTRALATERAL, perdida sensitiva, HEMIANOPSIA
HOMNIMA, desviacin ocular al lado de la lesin.
EVC
Que sintomatologa causa la obstruccin de la ARTERIA CEREBRAL en su PORCIN POSTERIOR?
R = Causa AFASIA DE WERNIKE y HEMIANOPSIA HOMONIMA

- A 74-year-old woman develops acute neurologic symptoms and presents to the emergency room. An urgent
magnetic resonance imaging (MRI) scan demonstrates acute occlusion in the right posterior cerebral artery. Which of
the following clinical symptoms is she most likely to have?

(A) homonymous hemianopia


(B) total blindness
(C) expressive aphasia
(D) ataxia and dysarthria
(E) a right-sided hemiplegia

-Occlusion of the right posterior cerebral artery is most likely to cause homonymous hemianopia. This artery conveys
blood to the inferior and medial portion of the posterior temporal and occipital lobes and to the optic thalamus.

- A 63-year-old man developed a transient episode of vertigo, slurred speech, diplopia, and paresthesias. He is
symptom-free now, and clinical examination is entirely normal. His pastmedical history is significant for hypertension
and dyslipidemia. Which of the following is the most likely cause for symptoms?

(A) posterior circulation transient ischemic attack (TIA)


(B) anterior communicating artery aneurysm
(C) hypertensive encephalopathy
(D) pseudobulbar palsy
(E) occlusion of the middle cerebral artery

-Posterior circulation TIA is suggested by the transient episodes. The basilar artery is formed by the two vertebral
arteries and supplies the pons, the midbrain, and the cerebellum. With vertebrobasilar TIAs, tinnitus, vertigo, diplopia,
ataxia, hemiparesis, and bilateral visual impairment are common findings.
EVC
Cual es el cuadro clnico de la obstruccin de la arteria basilar?
R = Alteracin del estado de alerta, parlisis de los nervios craneales y cadas.

Como manejas la hipertensin maligna?


R = Esta entidad se da con una TAD >130, esta se debe REDUCIR A 1/3 pero NUNCA a un VALOR MENOR DE 95
MM HG. NO se debe usar NEFEDIPINO sublingual. En el caso de HIPERTENSIN INTRACRANEAL NO SE DEBE
USAR ESTEROIDES.

Como se procede ante un EVC < 3 hrs?


R = TROMBOLISIS IV con activador del plasminogeno no tisular. En los que esta CONTRAINDICADA ESTA
TERAPIA son aquellos con:
1) TAS > 185 mm Hg o TAD > 110
2) Sintomatologa leve o que mejore de forma rpida
3) CONVULSIONES al inicio de TCE en los 3 MESES PREVIOS
4) CIRUGA MAYOR 2 semanas previas
5) Glucemia < 50 o > 400
6) PLAQUETAS < 100,000
7) MUJERES EMBARAZADAS o en lactancia
8) Ingesta actual de anticoagulantes orales con TP > 15

Cuales son los datos clnicos de una hemorragia intracraneal?


R = Adems de depender de la localizacin del hematoma suele observarse CEFALEA, CRISIS CONVULSIVAS,
NAUSEA Y VOMITO.
EVC
Cual es el cuadro clnico de la HEMORRAGIA SUBARACNOIDEA?
1) Se debe principalmente a la ROTURA DE UN ANEURISMA SECUNDARIA A UN TRAUMATISMO en pacientes con
ANTECEDENTE DE HAS
2) CEFALEA SUBITA, intensa y pulstil espontanea o ASOCIADA A
ALGN ESFUERZO FISICO.
3) El px lo refiere como EL DOLOR MAS INTENSO QUE HA TENIDO EN SU VIDA
4) SE PUEDE ACOMPAAR DE NAUSEA, VOMITO, FONOFOBIA Y FOTOFOBIA.
5) Si la evolucin es rpida el px puede evolucionar RAPIDAMENTE A ESTADO DE COMA.
6) Este tipo de hemorragia puede presentarse como MUERTE SUBITA.
7) Para su diagnostico se utiliza la TAC, si hay DUDA DX SE REALIZA PUNCIN LUMBAR PARA BILIRRUBINA Y CRENOCITOS QUE
CONFIRMAN EL DIAGNOSTICO
8) LA HIPERGLUCEMIA Y EL AUMENTO DE LA TEMPERATURA SE ASOCIAN A MAL PRONSTICO.

Cual es el manejo de la HEMORRAGIA SUBARACNOIDEA?


1) REPOSO ABSOLUTO
2) Monitorizacin
3) Vendaje de miembros inferiores
4) No estimulacin
5) ANALGSICOS IV
6) NIMODIPINA ORAL para vasoespasmo cerebral sintomtico
7) HIPERVOLEMIA PROFILCTICA para prevenir vasoespasmo cerebral
Cuales son las caractersticas de la HEMORRAGIA SUBDURAL?
1) Son hematomas localizados entre la DURAMADRE Y LA ARACNOIDES
2) TCE CON GLASGOW < 8
3) AGUDO < 24 HR, SUBAGUDO >24 HRS Y < 2 SEMANAS Y CRNICO > 2 SEMANAS
4) Los sntomas son FOCALIZACIN E HIPERTENSIN INTRACRANEANA; en las FORMAS SUBAGUDA Y CRNICA PREDOMINAN LA
CEFALEA Y DEBILIDAD.
5) Aquellos hematomas que DESPLACEN EL SEPTUM PELUCIDUM > 5 MM de la lnea media DEBEN SER EVACUADOS.
EVC
Cuales son las caractersticas de la HEMORRAGIA EPIDURAL?
1) DISMINUCIN DEL ESTADO DE ALERTA seguido de
2) INTERVALO DE LUCIDEZ y posteriormente
3) DETERIORO NEUROLGICO PROGRESIVO

Cuales son las caractersticas de la trombosis venosa cerebral?


1) 90% presenta como sntoma principal CEFALEA de inicio gradual
2) Debe sospecharse ante un paciente con TAC Y PUNCIN LUMBAR NORMAL
3) 40% convulsiones
4) En la TROMBOSIS DEL SENO SAGITAL puede haber DELIRIUM Y AMNESIA.
5) A la exploracin fsica con papiledema y en trombosis venosa focalizacin
6) El ESTNDAR DE ORO PARA EL DIAGNOSTICO ES UNA ANGIOGRAFA EN FASE VENOSA AS
COMO RM CON VENOGRAFA
7) El tratamiento de eleccin es la ANTI COAGULACIN formal por lo menos SEIS MESES
debido al riesgo de recurrencia as como de tromboembolia pulmonar.
TUMORES SNC
Cuales son las manifestaciones clnicas de una TUMORACIN EN EL LBULO FRONTAL?
R = Declinacin progresiva del intelecto y CAMBIOS EN LA PERSONALIDAD.
Cuales son las manifestaciones clnicas que te da una TUMORACIN EN EL LBULO
TEMPORAL?
1) Si afectan el TERCIO ANTERIOR se presentara EPILEPSIA Y TRASTORNOS DE LA CONDUCTA
como agresividad e impulsividad
2) SI afecta los 2/3 POSTERIORES se pueden manifestar con HEMIANOPSIA LATERAL
HOMNIMA
3) Convulsiones con alucinaciones olfativas o gustativas. Alteraciones en el campo visual.

Cuales son las manifestaciones clnicas que da una TUMORACIN EN EL LBULO PARIETAL?
R = TRASTORNOS CONTRA LATERALES DE LA SENSIBILIDAD. ESTEREOGNOSIA (no
reconocen objetos colocados en la mano), SINDROME DE GERTSMAN
combinacin de alexia, agrafia, acalculia y agnosia digital.

Que manifestaciones clnicas da una TUMORACIN EN EL LBULO OCCIPITAL?


R = HEMIANOPSIA HOMNIMA CRUZADA o defecto del campo visual,
PROSOPAGNOSIA ( no reconoce familiares)
Que manifestaciones clnicas da la lesin en el TALLO CEREBRAL Y CEREBELOSAS?
R = PARLISIS DE LOS PARES CRANEALES, ATAXIA, incoordinacin, nistagmo.
TUMORES DEL SNC
Cual es el tipo de tumor mas frecuente en la medula espinal?
R = Ependimoma

Que es la enfermedad de Von Recklinghausen?


R = Es una alteracin en el CROMOSOMA 17, llamado NEUROFIBROMATOSIS TIPO I que consiste en maculas
MLTIPLES HIPERPIGMENTADAS Y NEUROFIBROMAS

- A 10-year-old boy has multiple tan-colored patches on his skin, and freckle-like skin changes in his armpit
area. The rest of clinical examination is normal. Which of the following conditions is also found in patients
withthis disorder as they get older?

(A) bilateral eighth nerve tumors


(B) irregular small pupils
(C) multiple cutaneous and subcutaneous tumors
(D) cataracts
(E) hip involvement

-The two common forms of neurofibromatosis (NF-1 and NF-2) are genetically distinct. NF-1 is the type with
multiple caf au lait spots and is associated with axillary or inguinal freckling, iris hamartomas (Lisch nodules),
peripheral neurofibromas, and bony abnormalities (including kyphoscoliosis). NF-2 is associated with CNS
tumors, particularly bilateral eighth nerve tumors. Skin lesions are spare or absent, and early lens opacities can
occur.

TUMORES DEL SNC
Que caracteriza a la neurofibromatosis tipo II?
R = Es una alteracin en el CROMOSOMA 22. Provoca TUMORES DEL 8vo PAR CRANEAL

Que es el sndrome de STURGE-WEBER?


R = ANGIOMA CAPILAR CUTNEO congnito que puede ocasionar ANGIOMA DE COROIDES

Cuales son las caractersticas de los astrocitomas (gliomas)?


1) Tumores intracraneales MAS FRECUENTE EN NIOS y jvenes entre 20 y 40.
2) EL ASTROCITOMA PILOCITICO es la FORMA MAS BENIGNA de los astrocitomas, PREDOMINA EN NIOS y
adultos jvenes, afectando PRINCIPALMENTE AL CEREBELO.

Cuales son las caractersticas de los OLIGODENDROGLIOMAS?


1) Estos tumores suelen tener zonas de CLACIFICACIONES Y HEMORRAGIAS
2) Tumor de crecimiento lento

Cuales son las caractersticas de los ependimomas?


1) Junto al sistema ventricular REVESTIDO DEL EPNDIMO
2) En los nios aparecen principalmente en la primera dcada de la vida DENTRO DE LOS VENTRCULOS.

Cuales son las caractersticas de los germinomas?


R = Estos tumores tienen muchas caractersticas comunes a las neoplasias de clulas germinales que se
localizan en las gnadas, incluidos los marcadores tumorales

Cual es el cuadro clnico del meduloblastoma?


1) CRISIS CONVULSIVAS PARCIALES O GENERALIZADAS
2) Sndrome de hipertensin intracraneana caracterizado por la triada de: CEFALEA, VOMITO Y EDEMA DE
PAPILA BILATERAL.
3) Alteraciones cognitivas que se pueden presentar en tumores del lbulo frontal.
PARKINSON
Que tratamiento resulta efectivo en el temblor esencial benigno?
R = PROPANOLOL y si falla primidona o CLONACEPAM

Que medicamentos causan PARKINSON-LIKE?


R = Neurolpticos, METOCLOPRAMIDA o reseprina.

Cual es el neurotransmisor involucrado en Parkinson y cual es la fisiopatologia?


1) DOPAMINA, habiendo un desequilibro con acetilcolina
2) HISTOLOGICAMENTE la enfermedad de Parkinson se caracteriza por DEGENERACIN NEURONAL DE LA
SUSTANCIA NIGRA, PARS COMPACTA (PRODUCTORA DE DOPAMINA) y las fibras que la comunica con el
estriado.
3) En el microscopio se encuentran los CUERPOS DE LEWY en la sustancia nigra pars compacta (CUERPOS DE
INCLUSION INTRANEURONAL, EOSINOFILOS CONSTITUIDOS POR ALTAS CONCENTRACIONES DE LA PROTEINA
ALFA-SINUCLEINA).

Cual es el cuadro clnico del Parkinson?


1) Motoras: TEMBLOR EN REPOSO, RIGIDEZ y bradiscinecia. CON LOS MOVIMIENTOS VOLUNTARIOS SE
ATENUA EL TEMBLOR Y SE EXACERBA CON LAS EMOCIONES. La bradicinecia se manifiesta como lentitud de
movimientos voluntarios. El paciente presenta PERDIDA DE LA EXPRESIN FACIAL, menores movimientos de
los labios y lengua al hablar (hipofonia), PERDIDA DE LOS MOVIMIENTOS FINOS DE ESCRITURA (micrografa) y
manipulacin de objetos pequeos, DIFICULTAD PARA LEVANTARSE DE UNA SILLA E INICIAR LA MARCHA la
cual se describe como FESTINANTE CON PASOS CORTOS Y RPIDOS (APRESURADA).
2) Dermatolgica: SEBORREA DE CUERO CABELLUDO y cara con aumento de la sudoracin.
3) Psiquitrica: DEPRESIN.
PARKINSON
Que es la enfermedad de Wilson en el dx diferencial de Parkinson?
R = Inicio a temprana edad, ANILLOS DE KAYSER-FLEISCHER, hepatitis crnica y aumento de la CONCENTRACIN
TISULAR DE COBRE.

- A 24-year-old man presents with mild jaundice, tremor, and personality changes. Examination reveals
slowness of finger movement, rigidity, and coarse tremor of the outstretched hands. As well there is abnormal
slow movement of the tongue and pharynx resulting in a change in speech and occasional difficulty swallowing.
He is icteric, the liver span is 10 cm, and no spleen is palpable. Which of the following findings is most likely
seen in this condition?

(A) a reduction of copper excretion in the urine


(B) an increase of the serum ceruloplasmin content
(C) no renal involvement
(D) retention of normal neurologic movements
(E) a peculiar greenish-brown pigmentation of the cornea

- In Wilsons disease, there is usually a reduction of the serum ceruloplasmin content. Signs and symptoms of
injury to the basal ganglia are accompanied by cirrhosis of the liver. Renal involvement is characterized by
persistent aminoaciduria. The most common neurologic finding is tremor. The corneal pigmentation (Kayser-
Fleischer ring) is the most important diagnostic finding on physical examination. If it is absent, any neurologic
findings cannot be ascribed to Wilsons disease.

Cual es el estndar de oro en el diagnostico de Parkinson?


1) La NECROPSIA
2) RM donde se observa REDUCCIN DE LA SUSTANCIA NEGRA PARS COMPACTA en la porcin mesencefalica
superior.
3) PET donde hay MENOR CAPTACIN del REACTIVO FLUORO-1-DOPA en el cuerpo estriado.
PARKINSON
Cual es el tratamiento del Parkinson?
1) LEVODOPA en pacientes MAYORES A 70 que es AGONISTA DE LA DOPAMINA
utilizada en combinacin con CARBIDOPA prevenir la conversin perifrica de la
levodopa a dopamina por accin de la ENZIMA DOPADESCARBOXILASA.
2) ANTICOLINERGICOS: en pacientes MENORES A 70, CONTRAINDICADOS
EN HPB Y GLAUCOMA DE NGULO CERRADO.
Que mecanismo de accin tiene la carbidopa?
R = INHIBE LA ENZIMA QUE CONVIERTE LA LEVODOPA A DOPAMINA

Cuales son los agonistas de la dopamina utilizados en Parkinson?


R = BROMOCRIPTINA y pergolida.

Como actan los inhibidores de la COMT?


R = Inhibidores de la Catecolamina-0-Metiltransferasa REDUCEN EL METABOLISMO
DE LA LEVODOPA A 3- METILDOPA y por lo tanto alteran la farmacocintica de la
levodopa en plasma llevando a NIVELES MAS SOSTENIDOS DE DOPAMINA
PLASMTICA: TOLCAPONA/ ENTAGAPONA

Cuales son los frmacos ANTIPSICOTICOS en Parkinson?


R = RISPERIDONA, clozapina: pudiendo esta causar deplecin de MO
PARKINSON
Que medida quirrgica se puede utilizar en Parkinson refractario a tx medico?
R = TALAMOTOMIA o PALIDOTOMIA de 1 solo lado.

- Which of the following is the most likely finding in a 79-year-old woman with Parkinsons disease?

(A) constant fine tremor


(B) muscle atrophy
(C) akinesia
(D) pupillary constriction
(E) spontaneous remission

- The characteristic triad in Parkinsons disease (Tremor, Rigidity, Akinesia) has been expanded to include Postural instability. This
forms the mnemonic TRAP. Autonomic instability is also common. Findings on examination also include masklike facies,
dysarthria, stooped posture, and abnormal gait

Que frmacos producen Parkinsonismo?


1) FENOTIACINAS, tioxantenos, butiprofenonas, neurolpticos, valproato y fluoxetina.
2) Produciendo SNTOMAS PARECIDOS A LOS DEL PARKINSON causando distonias, rigidez, fascies en mascara, bradicinesia y
temblor con menor frecuencia.
3) Los sntomas desaparecen una vez suspendido el frmaco.

Que entidades conforman al Parkinsonismo plus?


R = Son sndromes que incluyen SIGNOS Y SNTOMAS DE LA ENFERMEDAD DE PARKINSON RELACIONADOS CON
ANORMALIDADES EN PROTENA TAU tales como:
1) Paralisis supranuclear progresiva: Es un trastorno neurodegenerativo que se presenta en la 6ta a 7ma dcada de la vida y se
caracterizan por rigidez, acinesia o bradicinesia, inestabilidad, cadas, disartria, disfagia y demencia. En la RM se observa
atrofia del coliculo superior en el mesencfalo. La respuesta al tratamiento con agonistas de dopamina y levodopa es pobre.
2) Degeneracion corticobasal: Los sntomas se desarrollan en la sexta dcada de la vida incluyendo rigidez focal o asimtrica,
bradicinesia, temblor de accin y reposo y distona marcada. En la RM se observa perdida cortical localizada en la zona
superior de los lbulos frontales y parietal contralaterales.
3) Enfermedad difusa de los cuerpos de Lewy: Es un TRASTORNO NEURODEGENERATIVO caracterizado por la presencia de
sntomas parkinsonianos y alteraciones neuropsiquiatricas comnmente acompaados por DEMENCIA.
ALZHEIMER
Cual es la manifestacin principal de la enfermedad de Alzheimer?
Dficit de ACETILCOLINESTERASA
La PERDIDA DE MEMORIA a corto plazo

Cual es la manifestacin encontrada en TAC y Biopsia en Alzheimer?


1) Alzheimers disease can be quite diffuse, but there is particular INVOLVEMENT OF THE MEDIAL
TEMPORAL LOBES AND CORTICAL ASSOCIATION AREAS.
2) THE ATROPHY OF THE HIPPOCAMPUS IS PARTICULARLY MARKED.
3) Microscopic examination reveals NEUROFIBRILLARY TANGLES AND AMYLOID PLAQUES.

- A 69-year-old woman presents to the clinic with memory difficulty. The patients daughter is
concerned because she is having difficulty doing her finances, such as paying bills. Memory
impairment testing reveals the poor ability to generate lists of words or copy diagrams
(intersecting pentagons). Her remaining physical examination is normal. Which of the following
anatomic findings is most likely with her condition?

(A) atrophy of the medial temporal lobes


(B) atrophy of the entire frontal and temporal lobes
(C) cranial nerve involvement
(D) transient episodes of hemiplegia
(E) atrophy of the caudate
HUNTINGTON
En que gen se ha identificado la enfermedad de Huntington?

R = Herencia AD CROMOSOMA 4
Cual es el cuadro clnico de la enfermedad de Huntington?
R = Aparece entre los 30-50 aos. MOVIMIENTOS ANORMALES y CAMBIOS INTELECTUALES CON IRRITABILIDAD,
ESTADO DEL ANIMO, TERMINANDO EN DEMENCIA.

- A 38-year-old man presents with involuntary facial grimacing, shrugging of the shoulders, and jerking
movements of the limb. His father was similarly affected. There is also a history of mood changes for the past 3
months. On examination, he appears restless with intermittent slow movements of his hands and face. He has
difficulty performing rapid finger movements, and tone is decreased in the upper and lower limbs. Which of
the following is most likely to represent the progression of his illness?

(A) a normal life span


(B) a 50% chance of only male children being similarly affected
(C) mental deterioration
(D) eventual development of rigidity
(E) development of hemiparesis

- This is a case of Huntingtons chorea. It is an autosomal dominant gene (found on the short arm of
chromosome 4), and male and female children are equally affected. Movement disorder, mental deterioration,
and personality change are the hallmarks of the disease, but can be very subtle initially. The disease starts
typically between ages 35 and 40 (although the variation is wide) and runs its course in about 15 years. The
akinetic rigid variety (Westphal variant) of Huntingtons typically has a childhood onset.
HUNTINGTON
Que datos de TAC se obtienen en corea de Huntington?
R = Atrofia cerebral y del ncleo caudado

Cuando hay corea sin antecedentes se duda del dx, pero que causa
la corea de Syndenham?
R = Es autolimitada por infeccin por estreptococo del Gpo A

Que padecimiento AD con datos clnicos similares a CH se debe a


mutacin del gen 12 y con ascendencia japonesa?
R = Atrofia dentorubro palidolisiana con tx igual a CH

Cual es el neurotransmisor involucrado en corea de hungtinton y


que tratamiento bloqueador de receptores de ese neurotransmisor
controlan la discinecia y cambio conductual?
R= GABA y se utiliza HALOPERIDOL
DEMENCIA POR CUERPOS DE LEWY
DEMENCIA POR CUERPOS DE LEWY
GILLES DE LA TOURETTE
Que caracteriza al sndrome de Gilles de la Tourette?
R = Los TICS MOTORES constituyen el 80% siendo comn >21
afectando PRINCIPALMENTE LA CARA.

Cual es el cuadro clnico del sndrome de Gilles de la


Tourette?
1) Tics motores: Olfateo, PARPADEO, fruncir el ceno, ENCOGER
LOS HOMBROS O ASENTIR CON LA CABEZA.
2) Tics fnicos: Murmullos, quejidos, ladridos y aullidos,
coprolalia.

Cual es el tratamiento de eleccin del sx de Gilles de la


Tourette?
R = HALOPERIDOL
ESCLEROSIS MULTIPLE
Que caracteriza a la esclerosis mltiple?
1) Quiz tenga base autoinmune con desmielinizacion con gliosis reactiva en sustancia blanca enceflica y de la medula espinal
2) Se le ha asociado con Chlamydia pneumoniae, virus herpes 6 y adenovirus
3) El gen HLA-DRB1 en el cromosoma 6p21 es, entre los muchos genes HLA asociados con esclerosis mltiple.
4) El OLIGODENDROCITO encargado de la mielinizacion ES EL BLANCO PRINCIPAL del proceso inflamatorio
principalmente mediado por LT CD 8 CITO TXICOS y macrfagos.
5) Patolgicamente se distingue por placas de desmielinizacion afectando mas frecuentemente la sustancia blanca
periventricular

Cual es la clasificacin de la esclerosis mltiple?


1) Existen 4 tipos bsicos de esclerosis mltiples segn su progresin en brote - remisin, primaria progresiva, secundaria
progresiva y progresiva-recurrente.
2) 90% de los pacientes comienza con el TIPO BROTE- REMISIN caracterizado por la presentacin de uno a dos ataques
por year con remisiones y recuperaciones en los meses posteriores.

Cual es el cuadro clnico de la esclerosis mltiple?


1) La PRESENTACIN COMN ES DEBILIDAD, ENTUMECIMIENTO, HORMIGUEO, INESTABILIDAD DE UNA
EXTREMIDAD, paraparesia espstica, NEURITIS RETROBULBAR, trastornos esfinterianos. Los sntomas desaparecen
de das a semanas. En el examen aparecen ATROFIA PTICA, nistagmo, disartria y dficit sensitivo en extremidades.
2) Patrn ataque - recada - exacerbacin, definindose ataque como el cuadro agudo de disfuncin del SNC que dure mas de
24 hrs en ausencia de fiebre, infecciones o alteraciones metablicas.
3) FENMENO DE UTHOFF, el cual consiste en la aparicin o exacerbacin de los sntomas tras el ejercicio o un bao con
agua caliente
4) TANTO LAS TAREAS COGNITIVAS COMO LA ACTIVIDAD FSICA PROVOCAN CANSANCIO PROFUNDO Y
LOS PACIENTES TARDAN MAS TIEMPO EN RECUPERARSE
5) El CANSANCIO es uno de los sntomas mas comunes y discapacitantes de la enfermedad.
6) La afectacin de la va visual es casi una regla con la presencia de NEURITIS PTICA que se manifiesta como CEGUERA
MONO O BINOCULAR.
7) EL SIGNO DE LHERMITTE (sensacin de electricidad que va de la medula hacia las extremidades al flexionar el cuello).
ESCLEROSIS MULTIPLE
Cual es el estudio de eleccin para dx de esclerosis mltiple?
1) IRM de ENCFALO o MEDULA ESPINAL que demuestra MIELOPATIA FOCAL O MLTIPLE
EN LA SUSTANCIA BLANCA. Se aplican los criterios de Barkhof y Tintore.
2) El estudio del LCR permite valorar la inflamacin intratecal. SE BUSCAN BANDAS
OLIGOCLONALES DE IgG que no estn presentes en suero. Un total de PROTENAS
MENOR A 100 Y LEUCOCITOS MENORES A 50 ES LO COMN
3) Los POTENCIALES EVOCADOS VISUALES son los mas SENSIBLES Y ESPECFICOS, los cuales
DENOTAN DESMIELINIZACION.

En que consiste la malformacin de Arnold Chari, la cual debe descartarse en esclerosis


mltiple?
R = Observndose la regin del agujero magno UNA PARTE DEL CEREBELO Y PARTE INFERIOR
DEL TALLO ENCEFLICO SE DESPLAZAN HACIA EL CONDUCTO CERVICAL produciendo dficit
piramidal y cerebeloso en extremidades.

Cual es el tratamiento de esclerosis mltiple?


R = INTERFERON B disminuye exacerbaciones al igual que inmunoglobulina IV. NO HAY
FORMA DE DETENER EL AVANCE, se utiliza PREDNISONA EN RECIDIVAS.

Cual es el pronostico de la esclerosis mltiple?


R = Mientras mayor sea el numero y volumen de las lesiones peor ser el pronostico.
ESCLEROSIS MULTIPLE
-A 22-year-old woman presents with acute vision loss and pain in the left eye, but no other symptoms. On examination she
appears well, visual acuity is not possible, and she can only perceive movement and bright light. The direct papillary reflex is
absent but the indirect (consensual) response is normal. The optic disc is edematous. Which of the following symptoms is also
most likely present in patients with this condition?

(A) limb weakness


(B) hemiplegia
(C) cervical myelopathy
(D) sphincter impairment
(E) seizures

- This patient has multiple sclerosis (MS). Weakness or numbness in one or more limbs is the initial manifestation of disease in
about half the patients. Other common initial presentations include optic neuritis (25%) and acute myelitis. Hemiplegia,
seizures, and cervical myelopathy (in older patients) occur occasionally as the initial manifestation. Sphincter impairment
usually occurs later in the disease.

-A 27-year-old woman presents with acute vision loss and pain in the left eye, but no other symptoms. On examination, she
appears well, visual acuity is not possible, and she can only perceive movement and bright light. The direct pupillary reflex is
absent but the indirect (consensual) response is normal. The optic disc is edematous. Which of the following is the most likely
diagnosis?

(A) diabetic microvascular disease


(B) arteriosclerosis
(C) trauma
(D) multiple sclerosis
(E) Creutzfeldt-Jakob disease

-Visual loss in multiple sclerosis varies from slight blurring to no light perception. Other eye symptoms include diplopia and
pain. The classic syndrome of optic or retrobulbar neuritis occurs commonly at some point in the disease, and it is the
presenting symptom in 25% of cases.
EPILEPSIA
EPILEPSIA
ATAXIA DE FRIEDREICH
Que es la ataxia de Friedreich?
1) AR, en CROMOSOMA 9 q13-9 q21.
2) Es una MARCHA ATXICA, manos torpes, respuesta extensora plantar y disminucin de ROTS.
3) The pathologic changes are found in the SPINAL CORD TRACTS. Degeneration is seen in the
POSTERIOR COLUMNS, the lateral corticospinal tract, and the spinocerebellar tracts.
4) PIE CAVO BILATERAL

-A 19-year-old man has had progressive ataxia of gait and great difficulty in running. In the past year, he has
developed hand clumsiness. Physical examination reveals pes cavus, kyphoscoliosis, and both cerebellar and
sensory changes in the legs. There is a positive family history of Friedreichs ataxia. Where are the pathologic
changes seen in this condition most likely to be found?

(A) spinal cord tracts


(B) basal ganglia
(C) cerebral cortex
(D) peripheral autonomic nerves
(E) peripheral motor nerves

- This young man has Friedreichs ataxia, associated with a gene defect on chromosome 9. The pathologic
changes are found in the spinal cord tracts. Degeneration is seen in the posterior columns, the lateral
corticospinal tract, and the spinocerebellar tracts. Ataxia, sensory loss, nystagmus, reflex changes, clubfeet, and
kyphoscoliosis are the characteristic findings. The heart is frequently involved, and cardiac disease is a common
cause of death.
DEFICIT VITAMINA E y B12
Que produce el dficit de vitamina E a nivel SNC?
R = Se presenta degeneracin medulocerebelosa que afecta
principalmente al cordn posterior de la medula espinal.

Cuales son los datos clnicos de dficit de vitamina E?


R = Ataxia de extremidades, disminucin sensitiva, disminucin de ROTS,
confusin y degeneracin pigmentaria de la retina.

Cual es el tratamiento para el dficit de vitamina E?


R = Alfa tocoferilo

Que trastorno en el SNC te ocasiona el dficit de vitamina B12?


Degeneracin combinada subaguda de medula espinal.
Se acompaa de polineuropata, cambios mentales o neuropata ptica.
ENCEFALOPATA DE WERNICKE
Que caracteriza la encefalopata de Wernicke?
R = Dficit de TIAMINA B1 y es COMN EN BORRACHOS.

Cual es el cc de la encefalopata de Wernike?


R = CONFUSIN, ATAXIA Y NISTAGMO que da lugar a oftalmoplejia (parlisis de la mirada
conjugada, debilidad del musculo recto externo)

-A 43-year-old man is referred from the emergency department with memory loss and difficulty walking. He
was brought in by his wife who has noticed personality changes, truancy from work, and lack of personal care
over the past 2 years. On examination he appears unkempt, smells of urine, and is uncooperative. He cannot
recall the date or season, and gets angry when asked questions. His answers are often fabricated when checked
with his wife. His gait is wide-based, and there is loss of sensation in his feet up to the shins. His motor strength
and reflexes are normal. His ocular movements are normal and there is no nystagmus. In the past he has had
multiple admissions for alcohol withdrawal. Which of the following is the most likely diagnosis?

(A) Wernickes encephalopathy


(B) Wernicke-Korsakoff syndrome
(C) Alzheimers dementia
(D) Charcot-Marie-Tooth disease
(E) vascular dementia

-The combination of symptoms is typical of chronic alcohol abuse. The mental symptoms are suggestive of
Wernicke-Korsakoff syndrome. Adistal limb sensory-motor neuropathy is also typical of alcoholism. Confusion,
tremulousness, and disorientation are typical for acute alcohol intoxication. Wernickes encephalopathy is a
symptom complex of ophthalmoplegia, ataxia, nystagmus, and acute confusional state
DEGENERACIN COMBINADA
SUBAGUDA
Cual es la etiologa de la degeneracin combinada
subaguda y cual es el cuadro clnico caracterstico?

1) Es causado por la deficiencia de COBALAMINA (VB12)


2) En un inicio encontramos signos y sntomas de lesin de la
COLUMNA POSTERIOR (PARESTESIAS DE MANOS Y PIES,
INESTABILIDAD DE LA BIPEDESTACIN Y MARCHA,
ALTERACIONES DE VIBRACIN Y POSICIN)
3) Despus de un tiempo a las 2 semanas posteriores se
identifica una paraparesia atxica simtrica con
hiperreflexia o hiporreflexia tendinosa y signo de Babinski.
VITAMINAS HIDROSOLUBLES
VITAMINAS LIPOSOLUBLES
LYME
Cuales son las manifestaciones neurolgicas
de la enfermedad de Lyme?
1) Meningitis,
2) Poliradiculopatia,
3) Mononeuropatia multiple y
4) Neuropata cutnea.
SARCOIDOSIS
Cuales son las manifestaciones neurolgicas
de la sarcoidosis?
R = Parlisis de los pares craneales, en especial
el 7 que responde a esteroides
SNDROME DE GUILLAIN-BARRE
Cual es la etiologa del sndrome de Guillain-Barre?
1) Suele asociarse a una infeccin de vas respiratorias o gastrointestinal.
2) Principalmente el CAMPYLOBACTER JEJUNI Y CMV.

Cuales son los signos y sntomas de la poliradiculoneuropatia progresiva


aguda/sub aguda conocido como sndrome de Guillain- Barre?
R = Debilidad ascendente de comienzo en piernas.

Cuales son los estudios para el diagnostico de sndrome de Guillain- Barre?


1) LCR con AUMENTO DE PROTENAS
2) Clulas normales
3) Electrofisiologa con alteraciones
4) Histologa que muestra DESMIELINIZACION PRIMARIA

Cual es el manejo del sndrome de Guillain-Barre?


R = Administracin de INMUNOGLOBULINA IV O PLASMAFERESIS.
MIASTENIA GRAVE
Cual es el cuadro clnico de la miastenia grave?
1) DIPLOPA, debilidad de msculos oculares, debilidad progresiva pudiendo
causar parlisis respiratoria, PTOSIS.
2) El cuadro clnico confirma DEBILIDAD Y FATIGA DE MSCULOS EL CUAL
MEJORA CON EL REPOSO.

Como se realiza el diagnostico de miastenia grave?


1) Respuesta a ANTICOLINESTERASA de accin corta con EDROFONIO con
vida media de 1 minuto en el cual los PACIENTES CON RESPUESTA TIENEN
MEJORA NOTORIA DURANTE 5 MINUTOS.
2) NEOSTIGMINA, la respuesta dura 2 hrs, con sulfato de atropina en caso de
efectos muscariniocos adversos.
3) ELECTROFISIOLOGIA que muestra DISMINUCIN DE LA
TRANSMISIN.
4) LAB con Ac VS RECEPTOR DE ACETILCOLINA

Cual es el tratamiento de miastenia grave?


R = NEOSTIGMINA. Timectomia en caso de timoma. PROSCRITOS
AMINOGLUCOSIDOS por que causan bloqueo neuromuscular
MIASTENIA GRAVE
- A 30-year-old woman complains of double vision, and easy fatigue with
exercise. The fatigue improves with resting, but it is interfering with her work.
Examination reveals ptosis and impaired eye movements with normal pupillary
response. The double vision is brought out by asking her to look at the ceiling, and
after a sustained interval, the eyes slowly drift down. Which of the following is the
most likely diagnosis?

(A) optic atrophy


(B) ophthalmic zoster
(C) paralysis agitans
(D) Horner syndrome
(E) myasthenia gravis

- In myasthenia gravis, weakness of the facial and levator palpebrae muscles


produces a characteristic expressionless face, with drooping of the eyelids.
Weakness of the ocular muscles may cause paralysis or weakness of individual
muscles, paralysis of conjugate gaze, ophthalmoplegia, or a pattern similar to
internuclear ophthalmoplegia. The presence of normal pupillary responses to light
and accommodation with weakness of extraocular muscles, levators, and
orbicularis oculi is almost completely diagnostic of myasthenia
SNDROME DE LAMBERT-EATON O
MIASTENIFORME
Que puede ocasionar el sndrome de lambert-eaton o miasteniforme?
1) En este hay liberacin DEFECTUOSA DE ACETILCOLINA.
2) Se presenta con DEBILIDAD DE MSCULOS PROXIMALES y se
diferencia de miastenia gravis por que LA FUERZA SE AUMENTA
CON CONTRACCIN SOSTENIDO.
Como diagnosticas el sndrome de lambert-eaton?
R = Electrofisiologa por AUMENTO DE LA RESPUESTA
MUSCULAR A LA ESTIMULACIN
Cual es el manejo del sndrome de lambert-eaton?
R = PLASMAFERESIS, inmunodepresores (azatropina, prednisona).

Ayuda la toma de LCR en el caso de absceso cerebral?


R = NO, de echo ESTA PROSCRITO por que puede causar herniacin. NEUROINFECCION
Cual es la complicacin mas frecuente del SIDA en el SNC?
R = This is one of the most common neurologic complications of AIDS. Its pathologic substrate is
degeneration of the spinal tracts in the posterior and lateral columns, which have a vacuolated
microscopic appearance. Although the morphologic changes and clinical manifestations are similar
to those associated with vitamin B12 deficiency, the pathogenetic mechanism is probably not
related to dietary deficiencies. Since there is no specific clinical or laboratory test available for the
diagnosis of this syndrome, VACUOLAR MYELOPATHY IN AIDS PATIENTS REMAINS A DIAGNOSIS OF
EXCLUSION. This implies that other HIV-related neurologic complications must be ruled out

-A 56-year-old man is brought to the emergency department by his wife because of memory loss
and difficulty walking. She has noticed personality changes, truancy from work, and lack of
personal care over the past 1 year. On examination he appears unkempt, smells of urine, and is
uncooperative. He cannot recall the date or season,and gets angry when asked questions. His
answers are often fabricatedwhen checked with his wife. The blood pressure is 150/90 mm Hg,
pulse 100/min, and he is diaphoretic and tremulous. His gait is wide based, and motor strength and
reflexes are normal. His ocular movements are normal but there is nystagmus on lateral gaze. In the
past he has had multiple admissions for alcohol withdrawal. Which of the following is the most
appropriate next step in management?

(A) prophylactic phenytoin administration


(B) prophylactic diazepam administration
(C) prophylactic carbamazepine administration
(D) calcium administration
(E) steroid administration

- Prophylactic administration of diazepam in a withdrawing alcoholic can prevent or reduce severe


syndromes such as delirium tremens (DTs). Prophylactic phenytoin, however, is not helpful. Acalm,
quiet environment with close observation and frequent reassurance is very important. Vitamin
administration (especially thiamine) is important, but frequently, severe magnesium depletion
slows improvement.
LCR
BOTULISMO
Cual es la fisiopatologa del botulismo?
R = La toxina EVITA LA LIBERACIN DE ACETILCOLINA en las uniones
neuromusculares y sinapsis autnomas, transmitida por COMIDAS ENVASADAS

Cual es el cuadro clnico del botulismo?


1) Desarrollo SBITO DE DEBILIDAD INTENSA DESPUS DE 72 HRS DE LA COMIDA.
2) El tpico cuadro es de DIPLOPA, VISIN BORROSA, midriasis no reactiva,
estreimiento por leo paralitico, PTOSIS, debilidad facial, DIFICULTAD
RESPIRATORIA Y DEBILIDAD EN EXTREMIDADES INFERIORES CON SENSIBILIDAD
CONSERVADA.

Como se dx el botulismo?
R = SUERO CON TOXINA. Electrofisiolgico con estimulacin repetida con aumento
de la respuesta muscular de manera progresiva

Cual es el manejo del botulismo?


1) ANTITOXINA en pacientes NO ALRGICOS AL SUERO DE CABALLO.
2) GUANIDINA facilita la liberacin de acetilcolina
MIGRANA
Cual es el cuadro clnico de la migraa?
1) Cefalea intensa, pulstil, unilateral en la regin frontal o temporal.
2) La migraa con aura consiste en un inicio con alteraciones visuales con
luces-centilleos o luces de colores, palidez, vrtigo, alteraciones
sensitivas o debilidad unilateral,
3) Afasia transitoria o alteraciones en el lenguaje que aparecen
previamente al establecimiento de la migraa.

A que se refiere el termino estatus migranoso?


R = Se refiere a pacientes ya conocidos con migraa con manifestaciones
clnicas por mas de 72 Hrs

Cual es el manejo de la migraa?


R = Triptanos y derivados de la ergotamina
MIGRANA
Cuales son las 2 variantes de migraa a considerar?
1) Los sntomas prodrmicos son el resultado de una disfuncin en el territorio de la circulacin
cerebral posterior con sintomatologa visual bilateral, ataxia, disartria, vrtigo, parestesias de las
extremidades e incluso debilidad. Puede haber perdida de la conciencia antes de comenzar la
cefalea.
2) Migraa hemipljica: Es poco frecuente e incluye hemiplejia, que puede persistir das despus de
calmarse la cefalea.

- An 18-year-old woman has periodic episodes that begin with severely decreased vision, followed
by ataxia, dysarthria, and tinnitus. The symptoms last for 30 minutes and are then followed by a
throbbing occipital headache. Which of the following is the most likely diagnosis?

(A) vertebral-basilar insufficiency


(B) chronic basilar artery dissection
(C) classic migraine
(D) ophthalmoplegic migraine
(E) basilar migraine

Basilar migraine can be very dramatic, and can resemble ischemia in the territory of the basilar
posterior cerebral arteries. The visual symptoms of basilar migraine typically affect the whole of
both visual fields, and can even cause temporary cortical blindness. There can also be an alarming
period of coma or quadriplegia.
CEFALEA TENSIONAL
Cual es el cuadro clnico de la cefalea tensional?
1) Dolor difuso, en banda, de carcter sordo, que
puede acompaarse de hiperestesia en el cuero
cabelludo y agravarse con el ruido y la luz.
2) Tpicamente la distribucin del dolor es en
forma de banda, bilateral y se extiende desde la
frente hacia los temporales y nuca.
3) Como profilaxis pueden utilizarse antidepresivos
triciclicos y la amitriptilina es la ms eficaz.
CEFALEA EN RACIMOS O ENFERMEDAD DE
HORTON
Que caracteriza a la cefalea en racimos o
enfermedad de Horton?
1) Intenso dolor unilateral periorbitario
acompaado de inyeccin conjuntival, lagrimeo,
rinorrea y sndrome de Horner
2) El comienzo del dolor suele ser en la madrugada
o al despertar muy temprano.
3) Para su tratamiento son tiles los frmacos
utilizados en migraa.
4) Nifedipino y berapamil como parte del manejo
profilctico.
Cual es la sintomatologa en caso de narcolepsia?
NARCOPLEPSIA
R = The early age of onset and otherwise good health suggest a diagnosis of narcolepsy, which is usually accompanied by other
symptomatology. Hypnagogic hallucinations are almost always visual. They occur most frequently at the onset of sleep, either
during the day or at night. They are generally very vivid. Cataplexy is a brief loss of muscle power without loss of consciousness.
The patient is fully aware of what is going on. The paralysis may be complete or partial.

-A 31-year-old woman complains of excessive sleepiness during the daytime for years despite adequate nighttime sleep. She
has episodes of intense drowsiness three to four times a day, even when at work or while eating meals. She has sought medical
attention in the past, after falling asleep while driving. She is slender and otherwise healthy and on no medications. Which of
the following treatments is most likely indicated for her condition?

(A) a device providing continuous positive airway pressure (CPAP) at night


(B) oral surgery
(C) tracheostomy
(D) amphetamines
(E) benzodiazepines at bedtime

-This woman does not have risk factors for sleep apnea (older age, snoring, obesity) and likely has narcolepsy. Adrenergic
stimulant drugs such as methylphenidate or amphetamines help the sleepiness, and tricyclic compounds can help the cataplexy.
Strategically planned naps can also be helpful.

-A 25-year-old man complains of excessive sleepiness during the daytime for years despite adequate nighttime sleep. He has
sought medical attention after falling asleep while driving. He is slender and otherwise healthy and on no medications. Which of
the following symptoms might he also complain about?

(A) excessive snoring (wifes report)


(B) automatic behavior (wifes report)
(C) restless sleep (wifes report)
(D) paresthesias
(E) morning headache

-Automatic behavior with amnesia is a common manifestation of the narcolepsycataplexy syndromes, occurring in 50% of cases.
Automatic behavior can be confused with complex partial seizures. Paresthesias are not part of narcolepsy syndrome. Snoring,
restless sleep, and morning headache suggest sleep apnea.
ENDOCRINOLOGIA

ENARM
SINDROME METABOLICO
PARA EL DIAGNOSTICO DE ESTE SNDROME ES
NECESARIO LA PRESENCIA DE 3 O MAS DE LOS
SIGUIENTES CRITERIOS:

OBESIDAD ABDOMINAL MUJERES > 88 CMS Y HOMBRE >


102 CMS.
TRIACILGLICERIDOS >O IGUAL A 150 MG/DL O CON
TRATAMIENTO.
HDL <40 MG/DL (HOMBRES) Y < 50 MG/DL (MUJERES).
TENSIN ARTERIAL SISTLICA >O IGUAL A 130 MMHG Y
DIASTLICA >O IGUAL A 85 MMHG.
GLICEMIA > 110 MG/DL O CON TRATAMIENTO
HIPOGLICEMIANTE.
HIPERPROLACTINEMIA
Cual es una hormona inhibidora de la prolactina?
R = DOPAMINA, el aumento de estrgenos la inhibe.

Cuales son los signos y sntomas de hiperprolactinemia?


1) Hombres: DISFUNCIN ERCTIL, HIPOGONADISMO HIPOGONADOTROFICO, INFERTILIDAD y
disminucin de la libido. En ocasiones GINECOMASTIA SIN GALACTORREA.
2) Mujeres: OLIGOMENORREA, amenorrea, GALACTORREA, si el hipogonadismo no es tratado da osteoporosis.

Que lab se solicitan en hiperprolactinemia y que padecimientos se encuentran relacionados?


R = HCG en caso de embarazo, HIPOTIROIDISMO, IR y cirrosis.

Que estudio realizas en hiperprolactinemia no asociada a hipotiroidismo, embarazo o inducida por frmacos?
R= IRM buscando micro <2 cm o macroadenoma hipofisiario >3 cm.
Que medicamentos no deben recibir las mujeres con prolactinomas?
R = ESTRGENOS (<2cm no afectan), TETOSTERONA o EMBARAZO.

Cual es el tratamiento para hiperprolactinemia?


R = BROMOCRIPTINA. Antagonistas de la dopamina como CENTENO O CABERGOLINA al acostarse,
Quinagolida a los intolerantes derivados del cornezuelo de centeno
SSIADH
En que regin del hipotlamo se secreta la ADH y que funciones tiene?
1) En el NCLEO SUPRA PTICO
2) La hormona antidiurtica (ADH), o arginina vasopresina (AVP), es una hormona LIBERADA
PRINCIPALMENTE EN RESPUESTA A CAMBIOS EN LA OSMOLARIDAD SRICA O EN EL
VOLUMEN SANGUNEO. Tambin conocida como argipresina. Hace que los riones conserven agua
mediante la concentracin de orina y la reduccin de su volumen, estimulando la reabsorcin de agua.
Recibe su nombre de esta importante funcin como regulador homeostsico de fluidos.

Cual es la forma mas comn de HIPONATREMIA NORMOVOLEMICA?


R = SSIADH

Cual es el cc del SSIADH?


1) Neuromusculares: IRRITABILIDAD, debilidad muscular, letargo, CONFUSIN y coma.
2) Gastrointestinales: NAUSEA, VOMITO y anorexia
3) La intensidad de los sntomas se relaciona con el grado de hiponatremia

Como diagnosticas SSIADH?


1) Por exclusin
2) Prueba de sobrecarga hdrica y hallazgos de niveles plasmticos inapropiadamente elevados de vasopresina
en relacin a la osmolaridad plasmtica

Cual es el manejo del SSIADH?


1) RESTRICCIN HDRICA 500-1000 ml/24 hrs
2) DEMECLOCICLINA (inhibe la respuesta renal a la vasopresina)
PATOLOGIA DE TIROIDES
Que caractersticas clnicas acompaan a la tirotoxicosis?
R = Nerviosismo, PALPITACIONES, TAQUICARDIA, fatiga, debilidad, prdida de peso con buen
apetito, DIARREA, INTOLERANCIA AL CALOR, PIEL CALIENTE, transpiracin excesiva, labilidad
emocional, alteraciones menstruales, TEMBLOR FINO EN LAS MANOS, alteraciones oculares
y aumento variable de la glndula. FA.

Como se manifiesta la CRISIS O TORMENTA TIROIDEA?


SE MANIFIESTA POR DELIRIO, TAQUICARDIA, VOMITO, DIARREA y FIEBRE.

Cual es el manejo de la tormenta tiroidea?


R = Dosis altas de PROPILTIUORACILO, yodo o contrastes yodados, PROPANOLOL
Y DEXAMETASONA

Cuales son los anlisis mas especficos para enfermedad de Graves?


R = TSH-R AB 2da generacin y TSH-R recombinante son > 75%
Que diferencia hay entre la captacin de yodo de las siguientes patologas?
1) Enfermedad de Graves y bocio multinodular toxico: ALTO
2) Tiroiditis sub aguda: BAJO
PATOLOGIA DE TIROIDES
La enfermedad de GRAVES es la causa mas comn de TIROTOXICOSIS, que la caracteriza?
1) Enfermedad de Basedow en Europa.
2) ETIOLOGA AUTOINMUNE, aumento de hormonas tiroideas, AUMENTO DE GLNDULA, AC VS TSH.
3) Mujeres 8:1. SE RELACIONA con enfermedades autoinmunes como ANEMIA PERNICIOSA, MIASTENIA GRAVE.
4) Asociacin con HLA-B8 y HLA DR3, AC VS PEROXIDASA, AC VS TIROGLOBULINA y ANA estn presentes

Que es la enfermedad de Plummer en tirotoxicosis?


R = Son adenomas txicos tiroideos simples

Que es la enfermedad de Jodbasedow en tirotoxicosis?


R = Hipertiroidismo inducido por yodo

Que patologas extraglandulares originan tirotoxicosis?


R = Aumento en la HCG, EMBARAZO MOLAR, CORIOCARCINOMA y neoplasias testiculares.
Administracin de AMIODARONA en arritmias.

Cuales son los signos y sntomas de tirotoxicosis?


1) Mirada fija, CADA DE PARPADO, taquicardia o FA, TEMBLORES FINOS, PIEL HUMEDA/CALIENTE.
2) GRAVES: Oftalmopatia con quemosis, conjuntivitis y PROPTOSIS. DERMOPATIA MIXEDEMA PRETIBIAL
con ACUMULACIN DE GLUCOSAMINOGLUCANOS que ocasiona edema con textura rugosa.
PATOLOGIA DE TIROIDES
Cual es el manejo de la enfermedad de Graves?
1. PROPANOLOL: taquicardia, temblores y ansiedad
2. Derivados de la tiourea (METIMAZOL O PROPILTIURACILO): Puede causar
AGRANULOSITOSIS
3. Metimazol: < riesgo de necrosis heptica, fulminante
4. Propiltiuracilo: Es de ELECCIN en EMBARAZO O LACTANCIA
5. Agente de contraste yodados: ayudan en tirotoxicosis de cualquier etiologa los cuales
inhibe la trimonoyodinacion de T4 por lo tanto aumenta T3 y disminuye tirotoxicosis
6. YODO RADIACTIVO: destruye tejido tiroideo sobre activo

Que medidas se usan para complicaciones de enfermedad de Graves:


1. Oftalmopatia de Graves: EXOFTALMUS; ESTEROIDES, progresivo-radioterapia
2. Cardiacas: taquicardia sinusal; PROPANOLOL
3. Insuficiencia cardiaca: DIURTICOS Y DIGOXINA

Que causa dermopatia de Graves?


R = Engrosamiento de piel por glucosaminoglucanos, se le conoce como MIXEDEMA
PRETIBIAL Y SU TRATAMIENTO SON ESTEROIDES TPICOS.
PATOLOGIA TIROIDES
Que caracteriza en la tirotoxicosis a la PARLISIS PERIDICA HIPOKALEMICA TIROTOXICA?
R = PARLISIS FLCIDA SIMTRICA REPENTINA, HIPOKALEMIA E HIPOFOSFATEMIA. El
tratamiento es con PROPANOLOL que revierte la parlisis en 3 hrs. Dextrosa IV o CHBTS lo
agravan.

Cual es el manejo de los ndulos tiroideos solitarios txicos?


R = PROPANOLOL, YODO RADIACTIVO

Cual es el manejo del bocio multinodular toxico?


R = PROPANOLOL, YODO RADIACTIVO

Cual es el cc de la TIROIDITIS DE HASHIMOTO O TIROIDITIS LINFOCITICA CRNICA y a que


enfermedad progresa?
1) Tiroides con AUMENTO DE TAMAO SIN DOLOR, de consistencia dura,
ASIMTRICO y fijo
2) PROGRESIN A HIPOTIROIDISMO.
3) Depresin, fatiga crnica, XEROSTOMA, XEROFTALMIA.

Que anticuerpos son los principales implicados en la tiroiditis de Hashimoto?


R = AC ANTIMIELOPEROXIDASA y LINFOCITOS TCD 4
PATOLOGIA DE TIROIDES
Cual es el manejo de tiroiditis de Hashimoto al causar hipotiroidismo, bocio y Hashitoxicosis?
1) Hipotiroidismo: LEVOTIROXINA
2) Bocio: T4 para disminuir tamao de glndula
3) Hashitoxicosis: Aumenta liberacin de T4 por acumulo, PROPANOLOL o yodato sdico

Cuales son los medicamentos principalmente vinculados con la tiroiditis inducida por frmacos?
R = AMIODARONA, Interferon alfa, LITIO e interleucina 2.

Cual es el cuadro clnico de la tiroiditis por frmacos?


R = Tiroiditis INDOLORA

Que caracteriza a la TIROIDITIS DE RIEDEL /tiroiditis fibrosa invasiva, tiroiditis leosa, tiroiditis
lingeosa o tiroiditis invasiva?
1) Provoca HIPOTIROIDISMO e HIPOPARATIROIDISMO.
2) Se presenta en EDAD AVANZADA.
3) GLNDULA DURA CON ADHERENCIAS al cuello ocasionando DISNEA, DISFAGIA, DOLOR Y
RONQUERA.
4) Los trastornos incluyen FIBROSIS RETROPERITONEAL.

Cuales son las caractersticas laboratoriales en la produccin de T3 o T4 de distincin entre los


padecimientos causantes de hipertiroidismo?
1) TIROIDITIS SUB AGUDA (QUERVAIN)/ HASHIMOTO: >T4 + que T3
2) GRAVES/ BOCIO NODULAR TOXICO: >T3
PATOLOGIA DE TIROIDES
Cual es la causa de la tiroiditis aguda o supurativa?
R = Infecciosa

Cual es el cuadro clnico de la tiroiditis aguda o supurativa?


R = El paciente cursa con FIEBRE, disfagia, disfona, DOLOR EN LA
REGIN ANTERIOR DEL CUELLO, eritema y GLANDULA TIROIDEA SENSIBLE

Que muestra la biopsia en la tiroiditis aguda o supurativa?


R = INFILTRADO LEUCOCITARIO

Cual es el manejo de tiroiditis supurativa?


R = Antibiticos + drenaje quirrgico

Cual es la etiologa de la tiroiditis subaguda posparto?


R = Es causada por la INFLAMACIN LINFOCITICA de la tiroides en los
primeros 3 A 6M POSTERIORES AL PARTO.
PATOLOGIA DE TIROIDES
Cual es el cc de la tiroiditis subaguda posparto?
R = EL BOCIO es INDOLORO

Como se encuentra la VSG en la tiroiditis subaguda posparto?


R = VSG NORMAL que la DIFERENCIA DE LA DE QUERVAIN.

Cual es el manejo de la tiroiditis subaguda posparto?


1) Rara vez requiere tratamiento
2) En la FASE DE TIROTOXICOSIS es til un BETABLOQUEADOR
3) En la fase de HIPOTIROIDISMO puede usarse LEVOTIROXINA
4) Los frmacos anti tiroideos y los esteroides NO ESTN INDICADOS

Cual es la etiologa de la tiroiditis subaguda o de Quervain?


1) Suele ser precedida por un PROCESO INFECCIOSO DE LAS VAS
2) Se ha relacionado con ECHOVIRUS, ADENOVIRUS, COXACKIE VIRUS Y VIRUS DE LA
PAROTIDITIS
3) La edad de presentacin es entre 20-40
PATOLOGIA DE TIROIDES
Cual es el cuadro clnico de la TIROIDITIS DE QUERVAIN?
EL cuadro clnico comienza con prdromos de :
1) MIALGIAS GENERALIZADAS
2) Faringitis
3) Fatigas
4) FIEBRE
5) DOLOR Y EDEMA DEL CUELLO
Como se encuentra la VSG en la tiroiditis de Quervain?
1) ELEVACIN DE LA VSG
2) Elevacin de PCR

Cual es el manejo de tiroiditis subaguda o de Quervain?


R = ASA es de eleccin, PROPANOLOL para SNTOMAS TIROTOXICOS

Cual es el manejo del ESTROMA DE YEDEL?


R = TAMOXIFENO de eleccin por aos
PATOLOGIA DE TIROIDES
Un teratoma ovrico que patologa puede ocasionar, relacionado a la produccin de
hormonas?
R = TIROTOXICOSIS por produccin de TSH

Como afecta el aumento de hormonas tiroideas a los huesos?


R = Aumenta la estimulacin de la RESORCIN SEA.

Cuales son las manifestaciones clnicas del CRETINISMO?


R = Alteracin del desarrollo esqueltico y del SNC, CARA TOSCA, PROTRUSIN DE LA
LENGUA Y HERNIA UMBILICAL.

A que se le llama sndrome de plumer?


R = Cuando uno de los ndulos del BOCIO MULTINODULAR se hace HIPERFUNCIONANTE
(bocio multinodular toxico), puesto que la mayora de los pacientes son eutiroideos

Porque en un paciente que utiliza corticosteroides de manera crnica se adelgaza y se ve


fina la piel?
R = Debido al CATABOLISMO PROTEICO DE LAS PROTENAS consistentes en perdidas de
colgeno y resorcin sea.

Que alimentos se consideran bociogenos?


R = Mandioca, col de brucelas, repollo y coliflor
PATOLOGIA DE TIROIDES
Cual es la estirpe histolgica mas comn del carcinoma de tiroides y que lo caracteriza?
1) Carcinoma PAPILAR en 70-90%,
2) Asociado al GEN RET
3) El antecedente de RADIACIN EN CUELLO en la infancia se relaciona
4) Manifestndose como NDULO NICO,
5) Donde la citologa muestra CLULAS NUCLEARES SOBREPUESTAS CON APARIENCIA
DE VIDRIO ESMERILADO y surcos longitudinales.
6) Mejor pronostico,

Con que enfermedades se relaciona el carcinoma papilar de tiroides?


1) Sndrome de GARDNER (poliposis adenomatosa familiar de colon)
2) Enfermedad de COWDEN (bocio familiar y hamartromas cutneos)
3) Sndrome de TURCOT (tumores cerebrales mas poliposis en intestino grueso)

A que tipo de cncer tiroideo son mas propensos los pacientes con enfermedad de
Cowden?
1) FOLICULAR, AD,
2) Lo causa perdida de gen supresor de tumores.
3) Se caracteriza comnmente por macrocefalia, HAMARTROMAS MLTIPLES, cncer
mamario o PLIPOS INTESTINALES.
PATOLOGIA DE TIROIDES
Que distingue al carcinoma FOLICULAR tiroideo del papilar?
1) Que ESTA ENCAPSULADO,
2) INVADE VASOS SANGUNEOS.
3) Su VARIEDAD extensamente INVASIVA son las CLULAS DE HURTLE
4) Tiene MAL PRONSTICO.
5) Al extirparlo se hace SEGUMIENTO CON TIROGLOBULINA para descartar metstasis

Que origen tiene el CARCINOMA MEDULAR DE TIROIDES y que sustancia produce?


1) Se origina de las CLULAS C PARAFOLICULARES,
2) Secretor de CALCITONINA.
3) Secretor de ACTH
4) Asociado a mutacin en oncogen RET, debido a eso SE DEBEN ESTUDIAR TAMBIN A
FAMILIARES

Cual es el carcinoma de tiroides con peor pronostico?


1) ANAPLASICO con mortalidad del 95% en los primeros 6 meses.
2) Crecimiento rpido
3) Se han descrito supresin del gen supresor de tumores p53
PATOLOGIA TIROIDES
Cual es el tratamiento de eleccin en carcinoma
tiroideo?
1) Extirpacin quirrgica
2) Tratamiento medico con la INHIBICIN DE LA
TSH DANDO LEVOTIROXINA A DOSIS ALTAS
3) Yodo radiactivo indicado en pacientes
POSTOPERADOS DE CA TIROIDEO
4) Radioterapia externa: Solo en pacientes con
EXTIRPACIN QUIRRGICA INCOMPLETA
5) Vigilancia clnica y por us
SX DE CUSHING
Cual es la principal causa del Sx Cushing y su cuadro clnico?
1) Exgena por esteroides y 2da causa es por micro adenoma hipofisario
2) OBESIDAD CENTRAL
3) HAS por aumento del ANGIOTENSINOGENO HEPATICO
4) Intolerancia a la glucosa por incremento de la GLUCONEOGENESIS
inducida por los glucocorticoides
5) FACIES DE LUNA LLENA
6) Trastornos menstruales
7) HIRSUTISMO

Como diagnosticas el sx de Cushing?


1) Inicialmente se debe demostrar HIPERCOLESTEROLISMO
2) Determinacin de CORTISOL LIBRE URINARIO con sensibilidad de 90%
3) SUPRESIN CON DOSIS BAJAS DE DEXAMETASONA (prueba de
NUGENT)
4) Cuando un paciente tiene un cortisol urinario y serico a las 8:00 am > 1.8
despus de 1 mg/dl de dexametasona a las 23 hrs de la noche previa, el
paciente tiene sndrome de Cushing.
SX DE CUSHING
En cuanto al origen del sndrome de Cushing cuales son los estudios a realizar?
1) Se determina la hormona ADRENOCORTICOTROPICA plasmtica, LOS VALORES POR
DEBAJO DEL LIMITE DE DETECCIN CATALOGAN AL SNDROME COMO DE ORIGEN
SUPRARRENAL.
2) La RM es la tcnica de imagen de eleccin en el sndrome de Cushing de ORIGEN
HIPOFISARIO.
3) La TAC es de eleccin para visualizar las GLNDULAS SUPRARRENALES.

En que consiste la PRUEBA DE NUGENT en el diagnostico de sx de Cushing?


R = SUPRESIN DE ACTH con administrar 1mg de dexametasona a las 23 Hrs y con nueva
medicin a las 8 AM.

En que consiste el tratamiento medico del sx de Cushing?


R = KETOCONAZOL, pues INHIBE LA SNTESIS DE CORTISOL pero se emplea por periodos
cortos pues es hepatotoxico.

En que consiste el tratamiento quirrgico del sx de Cushing?


1) CIRUGA HIPOFISARIA
2) RADIOTERAPIA HIPOFISARIA que ha mostrado ser efectiva en los casos que no se pueda
realizar la CIRUGA TRANSESFENOIDAL
ENFERMEDAD DE ADDISON
Cual es la principal causa de la enfermedad de Addison/insuficiencia suprarrenal?
1) 90% adrenalitis AUTOINMUNE,
2) En pases subdesarrollados TB.

Cual es el cuadro clnico y DHE de la enfermedad de Addison?


1) HIPERPIGMENTACION de la piel (debido a que la deficiencia de cortisol,
provoca un amento compensatorio de la PROPIOMELANOCORTINA que es
precursora de la ACTH y de la hormona estimulante de melanocitos)
2) Vomito
3) Fatiga
4) Hipoglucemia
5) HIPOTENSIN
6) HIPONATREMIA
7) HIPERKALEMIA.
Cual es el desequilibrio hidroelectrolitico principal en la enfermedad de Addison?
R = ACIDOSIS METABOLICA con HIPERCALCEMIA.
ENFERMEDAD DE ADDISON
Cuales son las manifestaciones clnicas de una CRISIS ADISONIANA o suprarrenal?
1) FIEBRE,
2) Deshidratacin,
3) DOLOR ABDOMINAL
4) HIPOTENSIN O CHOQUE HIPOVOLEMICO,
5) Urea elevada
6) HIPONATREMIA,
7) HIPERKALEMIA,
8) HIPERCALCEMIA y
9) ACIDOSIS METABLICA.
Cual es el tratamiento de la crisis Adisoniana?
R = Administracin inmediata de HIDROCORTISONA IV C-8

Como realizas el diagnostico de enfermedad de Addison?


1) ESTANDAR DE ORO es la prueba de estrs con INSULINA IV la cual hace el diagnostico de adrenalitis
secundaria, DONDE EL CORTISOL DEBE AUMENTAR, SINO ENTONCES SE HACE EL DIAGNOSTICO.
2) Medicin de CORTISOL DURANTE LA MAANA y ADRENOCORTICOTROPINA con NIVELES BAJOS.
3) ACTH IM y MEDIR CORTISOL A LOS 60 MIN, EL CUAL DEBE AUMENTAR.
4) En los pacientes con adrenalitis autoinmune se encuentran AUTOANTICUERPOS ANTI-21HIDROXILASA.

Cual es el manejo de la enfermedad de Addison?


R = Hidrocortisona o PREDNISONA.
HIPERALDOSTERONISMO
Cual es la etiologa del hiperaldosteronismo?
1) Primario: Hipersecrecin de aldosterona la cual NO SE SUPRIME EN RESPUESTA A LA EXPANSIN DE
VOLUMEN.
2) Secundario: Altos niveles de renina la cual se reprime en respuesta a la expansin de volumen.

Cuales son los datos clnicos del hiperaldosteronismo?


1) Puede cursar asintomtico o
2) Cefalea
3) HAS
4) DEBILIDAD MUSCULAR
5) PARESTESIAS
6) Tetania
7) Parlisis

Cuales son los datos de laboratorio encontrados en Hiperaldosteronismo primario?


R = HIPERNATREMIA, HIPOKALEMIA y ALCALOSIS METABLICA

-A 30-year-old man is having symtoms of muscle weakness, fatigue, and headaches. On examination, his blood
pressure is 180/100 mm Hg, pulse 80/min, JVP 4 cm, heart sounds normal, and lungs clear. His serum
potassium level is 2.5 mEq/L and bicarbonate 30 mEq/L. The urine potassium is 40 mEq/L. Plasma renin is low
and aldosterone is high.

-Primary hyperaldosteronism is characterized by hypertension with high plasma aldosterone and low plasma
rennin
HIPERALDOSTERONISMO
Como diagnosticas hiperaldosteronismo?
R = ELEVACIN DE ALDOSTERONA PLASMTICA POSTERIOR A LA INGESTIN DE
CLORURO DE SODIO por cada comida durante dos a tres das o un NIVEL DE
ALDOSTERONA EN PLASMA ALTO DESPUS DE LA INFUSIN INTRAVENOSA DE
SOLUCIN SALINA (1 lt x hr durante 4 hrs).

Cual es el manejo del hiperaldosteronismo?


1) CIRUGA A PACIENTES CON ALDOSTERONOMA con adrenalectoma laparoscpica
2) En el caso de HIPERPLASIA SUPRARRENAL BILATERAL IDIOPTICA se debe recibir
TRATAMIENTO MEDICO CON ESPIRONOLACTONA
3) En el caso de hiperaldosteronismo tratable con esteroides se utiliza la
DEXAMETASONA para mantener una supresin del eje hipotlamo hipfisis.

Cuales son los datos que caracterizan al sndrome de Conn en


hiperaldosteronismo?
R = Hipotensin, hipokalemia, aumento de aldosterona y supresin secundaria de
renina.
HIPOALDOSTERONISMO
Cual es la etiologa del hipoaldosteronismo?
1) Puede producirse por un estimulo insuficiente por parte de la renina
(HIPORRENINEMICO) NEFROPATA DIABTICA
2) Por un fallo primario en la secrecin de aldosterona (HIPERRENINEMICO)
ENFERMEDAD DE ADDISON
3) Por resistencia a la accin de la aldosterona
(SEUDOHIPOALDOSTERONISMO).
4) Algunos medicamentos pueden ocasionarlo como los AINES, heparina (suprime la
secrecin de renina), IECAS.

Cual es el cuadro clnico del hipoaldosteronismo?


1) CALAMBRES
2) Debilidad muscular

Cuales son los datos de laboratorio del hipoaldosteronismo?


1) HIPERKALEMIA
2) HIPONATREMIA
3) ACIDOSIS METABOLICA HIPERCLOREMICA
4) Deshidratacin
5) Se debe hacer diagnostico diferencial de enfermedad de Addison
HIPOALDOSTERONISMO
Como se diagnostica el hipoaldosteronismo?
1) Analticamente se deben obtener los niveles plasmticos de aldosterona,
cortisol y actividad de renina
2) Si se sospecha de enfermedad de ADDISON SE DEBE REALIZAR LA PRUEBA
DE ESTIMULACIN CON HORMONA ACTH.
3) Si existe aldosterona baja con actividad de renina (HIPORRENINEMICO),
debemos pensar en NEFROPATIA DIABETICA,
4) Si obtenemos aldosterona baja y renina alta (HIPERRENINEMICO),
debemos sospechar ADISON o una forma grave de hiperplasia suprarrenal
congnita.
5) Si lo que nos reportan es ALDOSTERONA ELEVADA Y ACTIVIDAD DE
RENINA ALTA, debemos pensar en SEUDOHIPOALDOSTERONISMO.

Cual es el manejo del hipoaldosteronismo


1) INGESTIN LIBRE DE SAL como TRATAMIENTO MEDICO no farmacolgico
2) El tratamiento medico CON FRMACOS es con FLUDROCORTISONA.
ALTERACIONES POR DHE, Na y K
ENFERMEDAD DE ADDISON:
Acidosis Metabolica con hipercalcemia
DHE: Hiponatremia Hiperkalemia
Cual es el manejo de la enfermedad de Addison?
R = Hidrocortisona o PREDNISONA.

HIPOALDOSTERONISMO:
Acidosis Metabolica Hipercloremica
DHE: Hiponatremia Hiperkalemia
Cual es el manejo del hipoaldosteronismo
1) INGESTIN LIBRE DE SAL como TRATAMIENTO MEDICO no farmacolgico
2) El tratamiento medico CON FRMACOS es con FLUDROCORTISONA.

HIPERALDOSTERONISMO:
Alcalosis Metabolica
DHE: Hipernatremia - Hipokalemia
Cual es el manejo del hiperaldosteronismo?
1) CIRUGA A PACIENTES CON ALDOSTERONOMA con adrenalectoma laparoscpica
2) En el caso de HIPERPLASIA SUPRARRENAL BILATERAL IDIOPTICA se debe recibir
TRATAMIENTO MEDICO CON ESPIRONOLACTONA
3) En el caso de hiperaldosteronismo tratable con esteroides se utiliza la DEXAMETASONA
para mantener una supresin del eje hipotlamo hipfisis.
HIPERCALCEMIA
Cuales son los eventos metablicos implicados en la regulacin del calcio?
1) En presencia de hipercalcemia se estimula la secrecin de CALCITONINA, la cual INHIBE LA OSTEOLISIS y
ESTIMULA LA CALCIURIA.
2) Cuando la hipercalcemia permanece, la CALCITONINA INHIBE LA REABSORCIN OSTEOCLASTICA, LA
SECRECIN DE PARATHORMONA Y LA SNTESIS DE VITAMINA D.

Cual es la etiologa de la hipercalcemia?


1) HIPERPARATIROIDISMO PRIMARIO: Puede deberse a ADENOMA NICO o HIPERTROFIA GLANDULAR. Se
caracteriza por ELEVACIN DE LA PTH, HIPERCALCEMIA E HIPOFOSFATEMIA.
2) Hiperparatiroidismo secundario: Puede deberse a IR, sx de mal absorcin y raquitismo.
3) El adenoma y las neoplasias malignas causan 90% de las hipercalcemias.
4) Es frecuente en el CNCER DE MAMA.

Cual es el cuadro clnico de la hipercalcemia?


1) Ca entre 10.5-12 g/dL es asintomtico
2) Niveles mayores forman TRIADA CLSICA DE ESTREIMIENTO, NAUSEA Y VOMITO.
3) Fatiga, letargo, cefalea, sed, nausea, vomito, NEFROLITIASIS,
4) ACORTAMIENTO DEL INTERVALO QT EN EL EKG.

Cual es el manejo de la hipercalcemia?


1) HIDRATACION
2) FUROSEMIDE es eficaz para evitar la reabsorcin tubular de calcio

Cual es el frmaco que se indica en hipercalcemia crnica?


R = TIAZIDAS pues previenen clculos renales.
HIPOCALCEMIA
Cual es la etiologa de la hipocalcemia?
1) Ca srico menor 8.5. Previamente se debe confirmar que la albumina se encuentre en niveles sricos
normales, ya que el descenso de 1 g/dL de albumina se acompaa de un descenso de 0.8 mg/dL de
calcio.
2) La causa mas comn de HIPOPARATIROIDISMO ES LA CIRUGA TIROIDEA Y LA SEGUNDA ES LA
RADIACIN.
3) La hipomagnesemia < 1 mg/dL puede producir hipocalcemia
4) La hipermagnesmia reduce la secrecin de PTH.
5) La hiperfosfatemia causa hipercalcemia por formacin de complejos con el calcio.
6) En pacientes transfundidos se produce hipocalcemia ya que el citrato del paquete globular acta como
paquete globular.
7) EL SINDROME DE DIGEORGE se caracteriza por AUSENCIA DE GLNDULAS PARATORIDEAS, con DFICIT
DE PTH, ASOCIADO A DISPLASIA TIMICA, y malformaciones cardiacas por alteracin en el desarrollo
de LAS TERCERAS Y CUARTAS BOLSAS FARNGEAS.
8) Las formas adultas de hipoparatiroidismo se asocian a enfermedades autoinmunes como insuficiencia
suprarrenal, anemia perniciosa e hipotiroidismo.
9) En la PANCREATITIS, el calcio se deposita en reas con necrosis grasa.
10) EL SINDROME DE HUESO HAMBRIENTO se presenta en paratiroidectomia.
11) En enfermedades del APARATO DIGESTIVO se favorece el dficit de vitamina D por disminucin en la
absorcin.
12) La DIFENILHIDANTOINA inhibe la absorcin de calcio.

Cual es el cuadro clnico de la hipocalcemia?


1) HIPEREXITABILIDAD neuromuscular.
2) Parestesias, HIPERREFLEXIA, ESPASMO CARPOPEDAL, irritavilidad, SIGNO DE CHEVOSTEK Y TROSSEAU.
3) El EKG MUESTRA PROLONGACIN DEL ST.
OSTEOPOROSIS
Cual es el manejo de la osteoporosis?
1) ALENDRONATO, RISENDRONATO.
2) PTH a dosis bajas, al igual que los bifosfonatos han demostrado aumento en la
masa sea,
3) Terapia de reemplazo hormonal con estrgenos previene fracturas vertebrales.
4) RALOXIFENO Y TAMOXIFENO ya que AUMENTAN LA DENSIDAD SEA,
DISMINUYEN RIESGO DE FRACTURAS VERTEBRALES Y EL CNCER DE MAMA
5) La TIBOLONA es una sustancia sinttica con accin tejido especfica que tiene
efecto ESTROGNICO, PROGESTACIONAL Y ANDROGNICO segn el lugar de
accin. Esta molcula mejora los sntomas vasomotores, el estado anmico, la
libido, la atrofia urogenital y previene la osteoporosis postmenopusica; no
produce mastalgia ni modifica la densidad mamogrfica. Se observan adems
efectos beneficiosos sobre algunos parmetros cardiovasculares y hemostticos.
EN EL ENDOMETRIO, LA TIBOLONA SE CONVIERTE EN SU METABOLITO
PROGESTAGNICO/ANDROGNICO Y EN CONSECUENCIA NO PRODUCE
ESTIMULACIN ENDOMETRIAL NI SE NECESITA AGREGAR UN PROGESTGENO.
Esta ventaja mejora el cumplimiento de la terapia hormonal de reemplazo (THR)
debido que no provoca sangrados por disrupcin
DENSITOMETRIA OSEA
OSTEOMALACIA
Cual es la etiologa de la osteomalacia?
R = Dficit de VITAMINA D EN ADULTOS

Cual es el cuadro clnico de la osteomalacia?


1. DEBILIDAD MUSCULAR PROXIMAL, asociada con desgaste muscular, hipotona y dificultad para la
movilizacin.
2. EL OSTEOCOPO (dolor seo), es mas notorio en LA REGION LUMBAR, PELVIS Y EXTREMIDADES
PELVICAS DONDE SE PRESENTAN FRACTURAS.
3. Las fracturas suceden con un mnimo de traumatismo.
4. Los sntomas deben incluir: DOLOR OSEO, DEFORMIDADES DEL ESQUELETO Y/O CRANEO, piernas
arqueadas o rodillas malformadas, PECHO EN PALOMA, Deformaciones dentales.

Como se diagnostica la osteomalacia?


1. MARCLAJE CON TETRACICLINAS. Este medicamento se deposita en forma de bandas en el sitio de
mineralizacin y al ser fluorescentes pueden ser visualizados al microscopio. La tasa de crecimiento
del esqueleto puede estimarse en las biopsias de las crestas iliacas al medir la distancia entre las
bandas de tetraciclina. Si disminuye la distancia entre las bandas se hace el diagnostico.
2. EL HALLAZGO RADIOGRFICO CARACTERSTICO ES LA REDUCCIN EN LA
DENSIDAD SEA CON ADELGAZAMIENTO DE LA CORTEZA.
3. Laboratorio: Los pacientes con dficit de vitamina D se presentan con HIPOFOSFATEMIA, CALCIO
SRICO BAJO E HIPOPARATIROIDISMO SECUNDARIO.

Cual es el manejo de la osteomalacia?


R = Calcitriol o CALCIO + VITAMINA D en combinacin.
DIABETES INSIPIDA CENTRAL
A que se debe la diabetes inspida central primaria y secundaria?
1) Primaria : Ac vs ARGININA-VASOPRESINA.
2) LESIN DEL HIPOTLAMO, del tallo hipofisario o por cualquier CAUSA
INFECCIOSA, TRAUMTICA etc..

Cuales son las causas de DI central y cuales son los hallazgos en la TAC?
R = DI IS USUALLY CAUSED BY DESTRUCTION, OR AGENESIS, OF THE POSTERIOR
PITUITARY, its normal signaling is lost. PITUITARY DI CAN ALSO RESULT FROM
TRAUMA, TUMORS (BOTH PRIMARY AND SECONDARY), granulomas, infections,
inflammatory diseases, chemical toxins, congenital malformations, and genetic
disorders. Depending on the cause, the MRI may demonstrate other associated
findings.

Que causa diabetes inspida en el ultimo trimestre de embarazo-puerperio?


R = Es inducido por VASOPRESINA en el ultimo trimestre de embarazo, relacionada
frecuentemente con oligohidramnios, preeclampsia o disfuncin heptica.
RESPONDE A DESMOPRESINA
DIABETES INSIPIDA NEFROGENICA
Como diagnosticas DM inspida?
1) JUICIO CLNICO, no hay prueba de laboratorio contundente.
2) RECOLECCIN DE ORINA DE 24 HRS MIDIENDO VOLUMEN <2LTS/24 HRS SIN
HIPERNATREMIA.
3) Desmopresina: MIDIENDO VOLUMEN URINARIO 12 HRS ANTES Y 12 HRS DESPUS, EN EL
CUAL LOS PACIENTES CON DIABETES INSPIDA CENTRAL HABR DISMINUCIN DE SED Y
POLIURIA,
4) Realizndose IRM analizando hipfisis e hipotlamo, se observa saco hipofisario
engrosado: Clulas de langerhans, sarcoidosis, hipofisitis linfocitica, Histiocitosis
(proliferacin de macrfagos en diferentes aparatos y sistemas).
5) Desmopresina en DM nefrogenica valorando su concentracin con restriccin de lquidos
aumentando sus valores.

Cual es el manejo de la DM inspida nefrogenica o central?


1) CENTRAL: DESMOPRESINA tmb se utiliza en DM INSPIDA RELACIONADA CON
EMBARAZO o puerperio, evitar factores de agravamiento como administracin de
glucocorticoides.
2) NEFROGENICA: HIDROCLOROTIAZIDA con suplemento de potasio, INDOMETACINA en
fase aguda.

ACROMEGALIA
Cual es la causa principal de acromegalia?
R = ADENOMA HIPOFISIARIO, tambin por secrecin ectpica de GnRH o GH por tumores carcinoides hipotalmicos,
bronquiales o pancreticos.

Que caractersticas forman parte del SNDROME DE CARNEY?


R = MIXOMA AURICULAR, NEUROMA DEL ACSTICO Y PIGMENTACIN PUNTEADA DE LA PIEL

Cuales son los signos y sntomas de la acromegalia o gigantismo?


1) ESTATURA ELEVADA, gigantismo antes del cierre hiposifiario.
2) Acromegalia significa crecimiento de extremidades.
3) SNDROME DEL TNEL CARPIANO.
4) RASGOS FACIALES TOSCOS, PROGNATISMO CON MALA OCLUSIN.
5) HAS 50%. DM 30%
Cuales son los lab para dx acromegalia o gigantismo?
1) AYUNO. > IGF-1 5 VECES DE LO NORMAL.
2) Prolactina aumentada algunas veces.
3) Glucosa: la DM ES COMN.
4) Calcio: Para descartar hiperparatiroidismo.
5) Fosforo: Se encuentra aumentada.
6) T4 y TSH: HIPOTIROIDISMO COMN.
7) Se recomienda ADMINISTRAR UNA SOLUCIN DE DEXTROSA MIDIENDO LA GH 1 HR DESPUS, SI LOS VALORES ESTN < O
NORMALES AL IGUAL QUE LA IGF-1, YA QUE ESTA HORMONA RESPONDE CON DISMINUCIN A LA
INGESTA DE GLUCOSA.
Que medicamentos se utilizan en caso de no remitir > GH o IGF-1 con adenectoma transesfenoidal hipofisiaria?
1) ANLOGOS DE LA SOMATOSTATINA COMO OCTREOTIDE para conservar disminucin del tamao del adenoma. GH srica 1
mg y 2.5 mg, pero no responde bien si previos al tratamiento la GH > 20 ng/ml.
2) PEGVISOMANT es un antagonista del receptor de la GH bloqueando sus efectos, va SC, pero no reduce el tamao de los
adenomas.
3) RADIOCIRUGA: en los pacientes no responden al tratamiento quirrgico o medico, no se realiza si hay extensin suprecelar
por que puede daar el quiasma ptico.
NEM I
Cuales son las caractersticas de la NEM I O SNDROME DE WERMER?
1) Se caracteriza por neoplasias en PARATIROIDES, PNCREAS y ADENOHIPOFISIS.
2) Se hereda de forma AD en el CROMOSOMA 11

Cual es la clnica del NEM I o Sindrome de Wermer?


1) HIPERPARATIROIDISMO PRIMARIO: Que puede manifestarse con MIALGIAS, DEBILIDAD, HIPERCALCEMIA,
ALTERACIONES MENTALES, NEFROLITIASIS Y ANORMALIDADES SEAS. La hipercalcemia puede provocar
aumento de la secrecin de gastrina y un Sx Zollinger-Ellison secundario. EL TRATAMIENTO CONSISTE EN LA
EXTIRPACIN QUIRRGICA DE LAS PARATIROIDES.
2) TUMORES DE LAS CLULAS DE LOS ISLOTES PANCRETICOS: Los GASTRINOMAS SON LA CAUSA MAS
IMPORTANTE de morbimortalidad y la mayora son multiples. Los insulinomas son generalmente benignos y
nicos.
3) NEOPLASIA DE LA ADENOHIPOFISIS: La neoplasia MAS FRECUENTE ES EL PROLACTINOMA seguida del tumor
secretor de hormona del crecimiento que causa acromegalia, pueden presentarse como enfermedad de
Cushing secundaria a adenoma hipofisario.

Como realizas el diagnostico de NEM I?


1) Con 2 de las 3 neoplasias caractersticas del sndrome
2) El MEN I familiar se define como la presencia de un paciente afectado con esta enfermedad y un familiar de
primer grado que presente al menos una de las tres neoplasias caractersticas

Cual es el manejo del NEM I?


1) CIRUGA
2) PARA CADA MANIFESTACIN de sobre produccin hormonal se da el medicamento individual
MEN II
Cual es la etiologa cromosmica del MEN II?
R = Se hereda de forma AD, en el CROMOSOMA 10.
Cual es la clasificacin del MEN II?
MEN II A o Sndrome de Sipple: Con la presencia de cncer medular de
tiroides, FEOCROMOCITOMA e hiperparatiroidismo primario.
MEN II B: Con la presencia de cncer medular de tiroides, feocromocitoma,
NEUROMAS MUCOSOS, ganglioneuromatosis intestinal y HABITO
MARFANOIDE.

Como diagnosticas el MEN II?


1) 2 a 3 tumores de los antes mencionados
2) La mutacin del protoncogen RET esta indicada en todos los familiares de
primer grado.
3) EL CARCINOMA MEDULAR DE TIROIDES SUELE SER LA PRINCIPAL CAUSA
DE MUERTE.
GASTRINOMA
Cuales son los tumores de las clulas de los islotes pancreticos?
R = Gastrinoma, insulinoma, glucagonoma, vipoma y somatostinoma. GIGVS
Cual es el cuadro clnico del gastrinoma?
1) Enfermedad ulcerosa con dolor abdominal
2) Diarrea
3) ERGE.
4) Se debe sospechar en ULCERAS PPTICAS MLTIPLES.

Como diagnosticas el gastrinoma?


1) Los niveles sricos de GASTRINA EN AYUNO MAYORES DE 1000 Y PH < 2.5 EN LA SECRECIN GSTRICA
HACEN EL DIAGNOSTICO DE GASTRINOMA.
2) Se debe SUSPENDER LA ADMINISTRACIN DE ANTAGONISTAS DE H2 UNA SEMANA ANTES Y DE
OMEPRAZOL.
3) En paciente con niveles sricos menores a 1000 y PH < 2.5 se debe pensar en H. Pilory
4) La aclorhidria por gastritis atrfica o anemia perniciosa es una de las causas mas frecuentes de
hipergastrinemia.

Cual es el manejo del gastrinoma?


1) Para la hipersecrecin acida OMEPRAZOL
2) Gastrinoma RESECCIN QUIRRGICA PARA EVITAR METSTASIS
INSULINOMA
Cual es el cuadro clnico del insulinoma?
1) SECUNDARIOS AL EFECTO HIPOGLUCEMICO como cefalea, mareo, debilidad, confusin, desorientacin y coma.
2) Sntomas secundarios al aumento de catecolaminas circulantes provocado por la hipoglucemia como sudacin, temblor y
palpitacin.
3) La mayora de las crisis se relaciona con el ayuno.

Como realizas el diagnostico de insulinoma?


1) Se basa en la TRIADA DE WIPPLE la que consiste en sntomas de neuroglucopenia, niveles de glucosa en sangre por debajo
de 45 y mejora inmediata de los sntomas despus de la administracin de glucosa
2) Si se sospecha de insulinoma se debe realizar una PRUEBA DE AYUNO DURANTE 72 HRS, considerado como el mtodo
diagnostico de eleccin, donde se miden los niveles de glucosa, pptido C e insulina cada 6 hrs y siempre que exista clnica de
hipoglucemia la prueba se suspende si aparece una glucemia inferior a 45 o si se presentan sntomas neuroglucopenicos. En
sujetos normales la insulina debe disminuir en presencia de hipoglucemia. Los criterios diagnsticos de insulinoma incluyen
insulina plasmtica mayor de 6 cuando la glucemia es menor de 40, proinsulina > .2, pptido C > 200 y cociente de
insulina/glucosa > .3.
3) Existen AINTICUERPOS ANTIINSULINA Y NIVELES ELEVEADOS DE PROINSULINA.

Cual es el manejo del insulinoma?


1) Para las crisis de hipoglucemia se realizan comidas frecuentes y de escasa cantidad
2) En el caso de hipoglucemia se administra 50 ml de dextrosa al 50%, si no hay respuesta dentro de los primeros 15 minutos, la
dosis puede repetirse, en caso de que no se pueda canalizar al paciente se administra 1 mg de guagn por va IV, IM o
subcutnea.
3) El DIAZOXIDO inhibe la secrecin de insulina, pero provoca retencin de lquidos e hirsutismo con frecuencia.
4) La reseccin completa del tumor cura a la mayora de los pacientes. Cuando no se logra localizar el tumor se realiza una
pancreatectomia distal gradual hasta localizarlo dejando por lo menos 20-30% del pncreas hasta localizarlo.
5) EN EL CASO DE TUMOR MALIGNO CON METSTASIS O TUMOR IRRESECABLE SE RECOMIENDA EL
TRATAMIENTO A LARGO PLAZO CON DIAZOXIDO, SI NO RESPONDE SE INICIA QUIMIOEMBOLIZACION.
Que son los vipomas?
VIPOMAS
R = Son tumores derivados de las CLULAS DE LOS ISLOTES DEL PNCREAS que producen grandes
cantidades de PEPTIDO INTESTINAL VASOACTIVO, el cual AUMENTA LA CONTRACCIN
DEL MUSCULO LISO, INHIBE LA SECRESION GSTRICA Y TIENE EFECTOS VASODILATADORES.

Cual es el cuadro clnico asociado a vipomas?


1) El sntoma predominante es una DIARREA ACUOSA, que produce deshidratacin,
hipokalemia y debilidad.
2) La DIARREA ES SECRETORA, persiste durante el ayuno y NO RESPONDE A
ANTIDIARREICOS.
3) Acidosis metablica

Como realizas el diagnostico del vipoma?


1) Presencia de DIARREA SECRETORA
2) Niveles elevado de PPTIDO INTESTINAL VASOACTIVO
3) TUMORACION pancretica

Como se maneja el vipoma?


1) Corregir la deshidratacin y desequilibrio hidroelectrolitico, en especial de la HIPOKALEMIA.
2) Reseccin del tumor y en presencia de metstasis se reduce primero el tumor.
3) Si no se encuentra el tumor se busca en las glndulas suprarrenales y las cadenas simpticas, en caso
de no identificarse en ninguno de estos sitios se realiza reseccin del 80% del tumor.
4) En el caso de tumor maligno inoperable o presencia de metstasis se recomienda el uso del anlogo
de la somatostatina de accin prolongada, tambin puede utilizarse la embolizacion de la arteria
heptica.
GLUCAGONOMAS
Que son los glucagonomas?
1) Son tumores derivados de las CELULAS ALFA DE LOS ISLOTES DEL PANCREAS los cuales producen una gran
cantidad de la HORMONA GLUCAGON y puede secretar otros pptidos (polipeptido pancretico, gastrina y somatostatina).
2) Es mas frecuente en mujeres PERI O POSTMENOPAUSICAS

Cual es el cuadro clnico de los glucagonomas?


1) DERMATITIS NECROLITICA MIGRATORIA es caracterstica de los glucagonomas. Inicia como una zona eritematosa y
escamosa en la ingle y glteo, se extiende al perineo y extremidades inferiores, en ocasiones hay lesiones peri orales,
posteriormente se hace ampolloso y deja erosiones cuando las ampollas se rompen.
2) Intolerancia a la glucosa o DIABETES
3) Anemia
4) QUEILITIS Y ESTOMATITIS
5) Diarrea
6) Hipercolesterolemia

Como diagnosticas los glucagonomas?


1) Con el cuadro clnico y se confirma al encontrar niveles plasmticos de GLUCAGON EN AYUNO > 1000 (NORMAL 150) Y
UNA TUMORACIN PANCRETICA.
2) NO SE SUPRIME LA LIBERACIN DEL GLUCAGON CON LA ADMINISTRACIN DE GLUCOSA Y TAMPOCO SE
ESTIMULA CON LA ADMINISTRACIN DE ARGININA.

Cual es el manejo del glucagonoma?


1) Debido a la presencia de METASTASIS EN LA MAYORIA DE LOS PACIENTES, no es posible una reseccin quirrgica
curativa, pero en estos casos se extirpa el tumor primario y se reduce el volumen de las metstasis.
2) Los SUPLEMENTOS DE ZINC mejoran la dermatitis
3) Los ANALOGOS DE LA SOMATOSTATINA normalizan los valores de glucagon y aminocidos sricos, mejoran la dermatitis
y la diarrea, promueven el aumento de peso, pero NO corrigen la intolerancia a la glucosa.
4) La quimioterapia NO es til pero cuando se decide utilizarla, la combinacin que ha demostrado mas eficacia es la de
ESTREPTOZOCINA +DACARBACINA.
5) EL 50% DE LAS MUERTES ES CAUSADO POR EVENTOS TROMBOEMBOLICOS.
SOMATOSTINOMAS
Que son los somatostatinomas?
1) Son tumores derivados de las CELULAS DELTA DEL PANCREAS productores de
somatostatina, al igual que los VIPomas y glucagonomas
2) 70% SON MALIGNOS
3) La somatostatina es un inhibidor potente de la secrecin gstrica y pancretica

Cual es el cuadro clnico de los somatostatinomas?


1) Es causado por la inhibicin de la secrecin tanto exocrina como endocrina del pncreas
2) LA TRIADA CLASICA CONSISTE EN DIABETES, DIARREA-ESTEATORREA Y
COLELITIASIS.

Como diagnosticas los somatostatinomas?


R = La mayora se localiza incidentalmente al realizar cirugas abdominales o endoscopias. EL
diagnostico se realiza al demostrar la somatostatina elevada en plasma.

Cual es el manejo de los somatostatinamas?


1) ANLOGOS DE LA SOMATOSTATINA con el fin de inhibir la activacin de los receptores por
parte de la somatostatina secretada por el tumor.
2) EXTIRPACIN QUIRRGICA si se localiza el tumor
3) Para enfermedad metastasica se usa QUIMIOTERAPIA
FEOCROMOCITOMA
Cual es la etiologa del feocromocitoma?
1) Tumores de origen NEUROECTODERMICO desarrollado a partir de las CLULAS CROMAFINES DEL SNS (ganglios
simpticos y medula suprarrenal) productores de CATECOLAMINAS y la mayora de ellos libera epinefrina y
norepinefrina.
2) Los feocromocitomas ASOCIADOS A NEM SECRETAN SOLO EPINEFRINA.

Cual es la clasificacin y localizacin mas comn de los feocromocitomas?


1) Feocromocitomas si provienen de las clulas cromafines y paragangliomas si son de origen extra renal.
2) La localizacin mas frecuente es la adrenal 85%, siempre es unilateral, en caso de ser bilateral debe
sospecharse de NEM

Cual es el cuadro clnico del feocromocitoma?


1) HAS 90%
2) Crisis adrenal caracterizada por triada clsica de: CEFALEA, PALPITACIONES Y SUDORACIN.

Cuales son los mtodos diagnsticos de feocromocitoma?


1) Demostracin de niveles elevados de catecolaminas o sus metabolitos en sangre o en orina, lo que establece el
diagnostico.
2) Se determina el nivel de catecolamina libre o no metabolizadas (epinefrina y norepinefrina) o sus metabolitos (
metanefrina, normetanefrina y ACIDO VANILMANDELICO) en orina de 24 hrs.

Cuales son los mtodos de imagen para dx de feocromocitoma?


1) TAC
2) Gamagrafia para localizar paragangliomas

Cual es el manejo del feocromocitoma?


1) Ciruga, pero antes de realizarla es necesario realizar un bloqueo alfadrenergico, generalmente se utiliza
FENOXIBENZAMINA durante 10-14d previos a la ciruga.
2) Durante la anestesia o manipulacin del tumor SUELE APARECER HIPERTENSIN Y ARRITMIAS, LA PRIMERA SE
TRATA CON FENTOLAMINA Y LA SEGUNDA CON PROPANOLOL.
TUMOR CARCINOIDE
Cual es el cuadro clnico producido por el tumor carcinoide?
R = Producen sntomas inespecficos, vagos, relacionados con el rgano afectado tales como:
1) Carcinoide apendicular: El la LOCALIZACIN MAS FRECUENTE. Si el tumor mide mas de 2 cm se debe realizar una
hemicolectomia derecha, si mide entre 1-2 cm e invade la serosa se debe realizar una hemicolectomia derecha y
linfadenectomia regional. LA APENDICETOMA SE CONSIDERA CURATIVA CUANDO MIDEN MENOS DE 1 CM.
2) Carcinoide rectal: Estos secretan mas glucagon mas que serotonina. Puede manifestarse como SANGRADO RECTAL,
DOLOR O CONSTIPACION. Los tumores mayores de > 2 cm se tratan con una reseccin anterior baja o una
abdominoperineal.
3) Carcinoide del intestino delgado: La mayora se encuentra el ILEON. Pueden provocar desde molestias abdominales
leves hasta dolor abdominal intermitente causado por la obstruccin intestinal, la cual puede ser secundaria a la
fibrosis que producen estos tumores o con menor frecuencia se manifiestan con diarrea, hemorragia por ulceracin
de la mucosa o intususcepcin. SON LA CAUSA MAS FRECEUNTE DE SX CARCINOIDE.
4) Carcinoides pulmonares y bronquiales: Neumonas recurrentes, tos hemoptisis, dolor pleurtico. La reseccin en
cuna o segmentaria se recomienda para los tumores localizados en caso de los carcinoides atpicos se utilizan
resecciones mas amplias.
5) Carcinoides gstricos: Se originan de las clulas enterocromafines de la mucosa gstrica, se manifiestan como
anemia o dolor abdominal. EL tipo 1 y mas frecuente en mujeres se asocia a hipergastrinemia y gastritis atrfica.
6) Carcinoides del colon: La mayora en COLON DERECHO, siendo el CIEGO el sitio mas afectado. Pueden producir dolor,
anorexia y perdida de peso.

Como diagnosticas los tumores carcinoides?


1) Cuadro clnico
2) Concentracin de serotonina o de sus metabolitos en orina
3) Imagen
4) Se confirma por biopsia
5) Medicin del ACIDO 5-HIDROXINDOLACETICO en una muestra de ORINA de 24 hrs, el cual si es
mayor de 10 mg indica EXCESO DE SEROTONINA.
6) En SANGRE se miden los niveles de CROMOGRANINA A, glucoproteina producida por las clulas derivadas de la
cresta neural y secretada en los tumores neuroendocrinos, relacionada directamente con el volumen tumoral
HIPERPLASIA SUPRARRENAL CONGNITA
Cual es la etiologa del hiperandrogenismo suprarrenal?
R = Dficit de 21-hidroxilasa, el gen responsable se denomina CYP21B localizado en el brazo
corto del cromosoma 6

Cual es el cuadro clnico del hiperandrogenismo renal por dficit de 21-hidroxilasa?


A. FORMA CLSICA (PERDEDORA DE SAL O VIRILIZANTE SIMPLE): Implica la existencia de
hiperandrogenismo in tero que condiciona la aparicin de macrogenitosomia en el varn
y un grado variable de virilizacin en la mujer. En la forma perdedora de sal existe un
DFICIT TANTO DE CORTISOL COMO DE ALDOSTERONA y se manifiesta en ambos sexos por
perdida salina en la etapa neonatal. En la virilizante simple no esta afectada la sntesis de
aldosterona por lo que se mantiene la homeostasis del sodio.
B. FORMA NO CLSICA: Existe HIPERANDROGENISMO DE APARICIN POSNATAL. Los
sntomas mas frecuentes durante la infancia son pubarquia prematura, piel grasa con acn,
aceleracin del crecimiento y maduracin sea y en las nias aparece hipertrofia de
cltoris, en la adolescencia y la edad adulta las mujeres pueden presentar trastornos
menstruales, hirsutismo, acn, calvicie, ovarios poliqusticos e infertilidad, los varones
afectados pueden cursar con oligoespermia, acn e infertilidad.

Como diagnosticas el dficit de la enzima 21-hidroxilasa?


R = Demostracin de VALORES ELEVADOS DE 17-HIDROXIPROGESTERONA

Cual es el manejo del dficit de enzima 21-hidroxilasa?


R = GLUCOCORTICOIDES y en nios recin nacidos es FLUOROHIDROCORTISONA y en
adultos es a base de PREDNISONA.
ENFERMEDAD DE PAGET
A 63-year-old asymptomatic woman is investigated for a HIGH ALKALINE
PHOSPHATASE (ALP) level. X-rays of the pelvis show multiple porotic and
sclerotic lesions with characteristic whorls of trabeculation. HER EXCRETION
OF URINARY HYDROXYPROLINE IS ALSO ELEVATED. Which of the following is
the most likely diagnosis?

R = Enfermedad de Paget del Hueso

The elevated ALP AND HYDROXYPROLINE are diagnostic for Pagets


disease. The bony lesions are blastic and the sacrum and pelvis are most
frequently involved, followed closely by the tibia and femur. Hypercalcemia
can complcate immobilization. The etiology is unknown, but a viral agent
has been postulated. Symptoms may be absent or severe (pain, deformity).
In metastatic cancers of most types the lesion are lytic, and the other
metabolic abnormalities do not have an elevation in hydroxyproline.
Insulina e Hipoglucemiantes orales.
Hipoglucemiantes orales.

Sulfonilureas:
1a generacin: tolbutamida (TBT), clopropramida, tolazamida,
acetohexamida.
2a generacin: gliburida, glipizida, glibenclamida, glicazida. Son ms
potentes.
Insulina e Hipoglucemiantes orales.
Hipoglucemiantes orales.

Sulfonilureas.
Mecanismos:
Estimulan liberacin de insulina, disminuyen eliminacin heptica,
aumentan somatostatina (disminuye glucagon).
Regulacin a la baja de receptores: al susp pueden tener nuevam
respuesta.
Bloq canal K sensible a ATP: despolarizacin: aumenta Ca.
Insulina e Hipoglucemiantes orales.
Hipoglucemiantes orales.

Sulfonilureas.
Farmacodinamia.
VO mejor 30preprandial.
Unin PP 90-99% (alb).
T 1/2 tolazamida 4-7hrs, clorpropramida 24-48hrs. 2da generacin 1.5-
5hrs (efecto disminuye glucosa x 12-24hrs).
Metab hgado. Excrecin renal. Cuidado en insuf heptica y renal.
Insulina e Hipoglucemiantes orales.
Hipoglucemiantes orales.
Sulfonilureas.
Efectos colaterales.
1a generacin:40%. Menos frec 2da generacin.
Hipoglucemia.
Medicamentos: desplazan de PP: sulfonamidas, clofibrato, dicumarol,
salicilatos, fenilbutazona. El alcohol:aumenta su accin..
Nusea, vmito, ictericia colestsica, agranulocitosis, A aplsica, A
hemoltica, alergia.
Clorpropramida: dosulfiram 10-15%, hiponatremia 5% x aumentar ADH en
tbulo colector.
TBT: aumenta 2v riesgo de muerte cardiovascular.
Insulina e Hipoglucemiantes orales.
Hipoglucemiantes orales.

Sulfonilureas.
Usos.
DMIND.
Contraindicada en DMID, embarazo, lactancia, insuf renal y heptica
significativa.
Insulina e Hipoglucemiantes orales.
Hipoglucemiantes orales.
Biguanidas.
Metformin y fenformin.
Fenformin asociado a c lctica.
Metformin T1/2 1.3-4.5 hrs. Mx 3g/d. No produce hipoglucemia, no
libera insulina.
Aumenta accin perif insulina, inhibe gluconeognesis, disminuye
absorcin gluc intestinal.
VO.
No en insuf renal o heptica: c lctica, hipoxemia cr, insuf cardiaca.
Efectos colat: 20%. Diarrea, nusea, malestar abd, sabor metlico,
anorexia, disminuye absorcin vit B12 y folatos.
Insulina e Hipoglucemiantes orales.
Hipoglucemiantes orales.

Otros.
Ciglitazona, pioglitazona, troglitazona: thiazolidinedionas. No dan
hipoglucemia. Disminuyen resistencia a insulina x aumentar
transportadores glucosa.
Inhib alfa glucosidasa: acarbosa. La inhiben en el borde en cepillo:
inhiben absocin CHOS.
Efectos colaterales: malaabsorcin, flatulencia, distensin abd.
NEUMOLOGIA

ENARM
REACCIONES DE HIPERSENSIBILIDAD
ALTERACIONES NOMBRADAS
TIPO NOMBRE ALTERNATIVO MEDIADORES
FRECUENTEMENTE
ATOPIA
1 ALERGIA (IMMEDIATA) ANAFILAXIA IGE
ASMA

ANEMIA HEMOLTICA
AUTOINMUNE
IGM O IGG
2 ANTICUERPO DEPENDIENTE TROMBOCITOPENIA
(COMPLEMENTO)
ERITROBLASTOSIS FETALICOS
SNDROME DE GOODPASTURE

ENFERMEDAD DEL SUERO


ENFERMEDAD DE COMPLEJO REACCIN DE ARTHUS IGG
3
INMUNE LUPUS ERITEMATOSO (COMPLEMENTO)
SISTMICO

DERMATITIS DE CONTACTO
TEST DE MANTOUX
CITOTXICA HIPERSENSIBILIDAD
4 RECHAZO CRNICO DE RGANO CLULA T
RETARDADA
TRASPLANTADO
ESCLEROSIS MLTIPLE4
SARCOIDOSIS PULMONAR
En que consiste la sarcoidiosis?
R = Enfermedad sistmica de ETIOLOGA DESCONOCIDA, caracterizada en 90% por INFLAMACIN
GRANULOMATOSA PULMONAR

Que datos clnicos o de laboratorio son mas frecuentes a encontrar en sarcoidiosis?


1) Clnicos: Tos crnica, ERITEMA NODOSO, UVEITIS, parotiditis, hepato-esplenomegalia +
LINFADENOPATIA.
2) Laboratorio: AUMENTO DE ECA 40-80%, hipercalciuria.
3) ANERGIA a pruebas cutneas 70%.
4) LAVADO BRONCOALVEOLAR con alto contenido de CD4 Y CD8.

Cuales son los datos radiolgicos que te da sarcoidosis?


1) AUMENTO DE TAMAO DE GANGLIOS LINFTICOS HILIARES BILATERALES en pacientes asintomticos
con exploracin fsica normal y exmenes de rutina normales se presenta en el 95% de pacientes con
sarcoidosis pulmonar
2) Adenopatia hiliar bilateral/ afeccin parenquimatosa bajo la forma de infiltrado reticular difuso.

Cuales son los signos espirometricos mas frecuentes en sarcoidosis?


R = CAMBIOS RESTRICTIVOS con disminucin de los volmenes pulmonares y capacidad de difusin.

Que te indica la biopsia de un paciente con sarcoidosis?


R = GRANULOMAS NO CASEIFICANTES PATOGNOMONICOS

Cual es el tratamiento de sarcoidosis?


R = ESTEROIDES E INMUNOSUPRESORES
NEUMONIA NOSOCOMIAL
Como se define la neumona nosocomial (NNC)?
R = Se define la neumona adquirida en hospital como aquella que se desarrolla despus de una estancia
intrahospitalaria de 48 hrs.

Cual es la etiologa de la NNC?


R = Los patgenos mas comunes incluyen BACILOS GRAMNEGATIVOS, como PSEUDOMONAS
AUROGINOSA, E. coli, Klebsiella y Acinetobacter.

Cual es la sospecha diagnostica de la NNC?


1) Px presenta infiltrados nuevos o progresivos
2) Hallazgos clnicos como fiebre, leucocitosis o leucopenia, esputo purulento.
3) Los hallazgos radiolgicos pueden variar DESDE INFILTRADOS AREOS EN FORMA DE PARCHE HASTA
CONSOLIDACIONES LOBARES CON BRONCOGRAMA AREO O INFILTRADOS DIFUSOS ALVEOLARES O
INTERSTICIALES.

Cual es el manejo de la NNC?


1) DE INICIO EMPRICO
2) Esquema antibitico de espectro limitado: px con < 5d de hospitalizacin con Ceftriaxona, levofloxacina,
Ampicilina y Ertrapenem
3) Esquema antibitico de espectro amplio: px con >5d de hospitalizacin Cefalosporinas antipseudomonica
como CEFEPIME, Fluoroquinolona antipseudomonica como CIPROFLOXACINO, Aminoglucosido como
AMIKACINA.
4) La terapia es normalmente de 14 a 21d.
DERRAME PLEURAL
Cual es la diferencia entre derrame pleural de tipo exudativo y de tipo
trasudativo?

TRASUDADO: se define como aquel derrame pleural ORIGINADO POR


FACTORES SISTMICOS que producen alteraciones en la formacin o
reabsorcin del lquido pleural. La principal causa es ICC y cirrosis.

EXUDADO: Es originado por varios factores locales que perturban la


formacin y absorcin de lquido pleural. SON DEBIDOS PRINCIPALMENTE
A NEUMONA BACTERIANA, NEOPLASIAS MALIGNAS, INFECCIONES
VIRALES Y EMBOLIA PULMONAR.
DERRAME PLEURAL
Cuales son las caractersticas de laboratorio del EXUDADO pleural?
1) Relacin entre protenas del liquido pleural y protena serica >.5,
2) Relacin entre DHL de liquido pleural y DHL srica >.6,
3) DHL >2/3 partes de la DHL srica.

Cuales son los datos de laboratorio de un TRASUDADO pleural?


R = Glucosa igual a glucosa srica, PH 7.40-7.55, < 1000 leucocitos

Que caractersticas tiene el derrame pleural reumatoide?


R = Liquido turbio VERDE - AMARILLENTO, FR aumentado, CRISTALES DE
COLESTEROL.

Que frmacos se utilizan en la pleurodiesis?


R = DOXICICLINA y talco
PATOLOGIA MEDIASTINAL
En lo que respecta a la patologa mediastinal, cual es la divisin anatmica?
1) Mediastino ANTERIOR: Compuesto por trquea, ganglios linfticos, arco artico y sus grandes vasos,
venas inominadas y TIMO
2) Mediastino MEDIO: CORAZON, pericardio, ganglios linfticos, hilios pulmonares, vena cava superior,
nervio frnico y vago.
3) Mediastino POSTERIOR: ESOFAGO, aorta descendente, vena cigos, conducto torcico, nervios vagos
y cadena simptica

Cuales son las lesiones que ocupan el MEDIASTINO ANTERIOR y sus principales caractersticas?
1) TIMOMA: Mas comn en ADULTOS que en nios. Los sntomas mas comunes son dolor torcico, tos,
disnea. La IMAGEN RADIOGRAFICA TPICA ES UNA MASA LOBULADA, bien definida y la TAC muestra
una masa de tejido blando, encapsulada, bien definida y en ocasiones con necrosis o hemorragia. Se
desarrollan sndromes PARATIMICOS como la miastenia grave, hipogammaglobulinemia y aplasia pura
de glbulos rojos. El sndrome de Good es una inmunodeficiencia por dficit de anticuerpos asociada a
timoma, generalmente benigno.
2) TUMORES DE CELULAS GERMINALES: Como teratomas, seminomas y tumores no seminomatosos.
3) TERATOMAS: Mezcla de tejidos provenientes de las 3 capas germinativas con potencial de
transformacin maligna.
4) SEMINOMAS: Los sntomas mas frecuentes son disnea, dolor retroesternal, tos, fiebre,
GINECOMASTIA, perdida de peso y sndrome de vena cava superior por compresin tumoral.
5) NO SEMINOMATOSO: Se asocian a KLINEFELTER. Tiene elevacin de hormonas como la
ALFAFETOPROTEINA y gonadotropina corionica humana.
6) BOCIO MEDIASTINAL: Suelen ser EUTIROIDEOS. Con disnea, estridor y disfagia.
7) LINFOMA PRIMARIO: Linfoma Hodking siendo el no Hodking el menos comn. Con sintomatologa de
FIEBRE, SUDORACION NOCTURNA, perdida de peso, asociada a sntomas por compresin de
estructuras vecinas.
8) LINFOMA NO HODKING: Sus subtipos mas comunes son el linfoma linfoblastico y linfoma clulas B de
clulas grandes.
PATOLOGIA MEDIASTINAL
Cuales son las masas del MEDIASTINO MEDIO mas frecuentes y
sus principales caractersticas:
1) QUISTES MEDIASTINALES: Los principales sntomas son tos,
disnea, fiebre, expectoracin purulenta y dolor torcico.
2) LINFANGIOMAS: Anormalidades de los vasos linfticos.

Cuales son las masas del MEDIASTINO POSTERIOR mas frecuentes


y sus principales caractersticas:
1) TUMORES NEUROGENICOS: Se encuentra los tumores benignos y
malignos de la vaina nerviosa
2) TUMORES DE LA VAINA NERVIOSA: Son tumores de crecimiento
lento y en su mayora sintomticos siendo los Schwannomas
3) TUMORES MALIGNOS DE LA VAINA NERVIOSA: Se asocia a
neurofibromatosis
MEDIASTINITIS
Cual es la etiologa de la mediastinitis?
R = Infecciosa

Cual es el cuadro clnico de la mediastinitis?


1) Escalofros, FIEBRE, mal estado general y dolor
torcico.
2) Al examen fsico puede haber dolor en el esternn,
crepitacin y el caracterstico pero inconstante SIGNO
DE HAMMAN (sonido crujiente que se escucha en la
pared anterior del trax durante la sstole cardiaca).
ASMA
En que casos se eleva la CAPACIDAD TOTAL DE DIFUSIN
PULMONAR?
R = ICC, ASMA y hemorragia pulmonar debido a un
incremento del volumen sanguneo en capilares pulmonares.

En que casos se emplea la broncoscopia rgida?


R = Hemorragia masiva, extraccin de grandes objetos y
biopsia de tumores traqueales.

Que parmetros evalan la obstruccin respiratoria de aire?


R = La reduccin del VEF1/CVF < 75%.

Con que medicamentos se realizan las pruebas de


estimulacin bronquial?
R = Histamina, METACOLINA.
EPOC
Como se define la bronquitis crnica?
R = Secrecin excesiva de moco bronquial manifestada por tos productiva durante 2-3 meses
en 2 aos consecutivos.

Que enzima liberada es la que produce el enfisema pulmonar?


R = PROTEASA de PMN

Cual es la clasificacin HISTOLOGICA del enfisema pulmonar y con que se relacionan cada
una mas frecuentemente?
1) CENTROLOBULILLAR: Se caracteriza porque el rea afectada est en el lobulillo proximal,
en especial por destruccin de los bronquiolos respiratorios y dilatacin de los lbulos
superiores sin afectar a los alvolos distales. Representa el 95% de los casos de enfisema y
la principal manifestacin en los FUMADORES asocindose comnmente con una
BRONQUITIS CRNICA.
2) PANACINAR: Es la forma ms comnmente asociada a una DEFICIENCIA DE ALFA-1
ANTITRIPSINA y se caracteriza por involucrar al extremo ciego de los alvolos de manera
homognea, mas que a los bronquiolos respiratorios y acompaado de los caractersticos
cambios destructivos. Es ms frecuente en la base de los pulmones.
3) PARASEPTAL: La enfisema paraseptal interesa prevalentemente a la parte perifrica del
lobulillo, vecina a la pleura creando grandes espacios areos en la regin interlobulillar. Es
ms frecuente en el pice pulmonar que en las bases y ocasionalmente se ASOCIA CON
NEUMOTRAX ESPONTNEO.
4) IRREGULAR: Es una enfisema cicatrizante, comprometido de manera irregular al acino
asociado a gneros asintomticos. Se le llama tambin enfisema paracicatrizal o
paraseptal.
EPOC
Como se encuentra la capacidad vital forzada pulmonar en
EPOC grave?
R = Disminuida

Como se encuentra la CTP en EPOC sin complicaciones?


R = Aumentada

Como se encuentra el volumen residual en EPOC?


R = Aumentado (VR)

Como se encuentra la relacin VR/CPT en EPOC?


R = Aumentada
EPOC
Cual es la principal caracterstica del soplador ROSADO en EPOC (enfisema
predominante)?
R = DISNEA, edad de presentacin a los 50, pacientes con tos y esputo mucoide,
pacientes delgados con bajo peso y no hay edema perifrico.

Cual es el principal sntoma del AZUL abotagado EPOC (bronquitis


predominante)?
R = TOS CRNICA con ESPUTO PURULENTO, edad de presentacin 30-40, disnea
leve y edema perifrico.

Que muestra la Rx de trax en el soplador rosado (enfisema)?


R = Hiperaereacion con hemidiafragmas aplanados.

Que muestra la Rx de trax en el azul abotagado (bronquitis predominante?


R = Sombras intersticiales.

Que iniciales controlan el tratamiento de EPOC?


R = GOLD
FIBROSIS QUISTICA
Que es la fibrosis qustica?
R = Es una enfermedad multisistemica que se transmite de forma AR y es
letal.

Cual es el tipo de mutacin que se encuentra en 60% de los casos en FQ?


R = Brazo corto de cromosoma 7 posicin AF508

Que dato te sugiere fibrosis qustica en el RN?


R =Impactacin fecal

Cual es la fisiopatologa de la fibrosis qustica?


R = Anomalas en la protena de canal de cloruro de la membrana celular

Cual es el resultado de la prueba del sudor en fibrosis qustica?


1) Concentracin de CLORURO mediante iontoforesis > 60 mEq en 2
ocasiones en la prueba de Gibson.
2) TRIPSINA INMUNORREACTIVA en neonatos, la cual se basa en el hecho
de que en etapas tempranas, esta protena se encuentra anormalmente
elevada por la obstruccin de conductos pancreticos.
FIBROSIS QUISTICA
Que tienen en comn los pacientes que tienen FQ?
R = Casi todos los varones tienen AUSENCIA BILATERAL CONGNITA DEL
CONDUCTO DEFERENTE CON AZOOSPERMIA EN 95%.

Que signos y sntomas es comn encontrar en FQ?


1) Antecedentes de enfermedad pulmonar crnica, pancreatitis recurrentes e
infertilidad.
2) Esteatorrea, dolor abdominal y diarrea son comunes.
3) La malabsorcin se manifiesta por deposiciones abundantes, ftidas y de
caractersticas grasas.

Cuales son los agentes etiolgicos de las bronquiectasias infectadas en FQ?


R = Haemophilus influenzae, Staphylococcus Aureus o Pseudomonas aeruginosa

Cual es el manejo teraputico de la FQ?


R = Comprende a la nutricin, prevencin de infecciones, inhaloterapia y manejo
de exacerbaciones inflamatorias.

Que medicamentos en FQ disminuyen las exacerbaciones respiratorios, consumo


de antibitico y aumento de la VEF1?
R = Desoxirribonucleasa humana recombinante inhalada.
BRONQUIECTASIAS
Que enfermedad es la responsable de mas del 50% de
bronquiectasias?
R = FIBROSIS QUSTICA y el resto se debe secundario a infecciones

Cual es la sintomatologa de las bronquioectasias?


R = Tos crnica, aumento del esputo purulento diario de cantidad
significativa, hemoptisis y neumonas recurrentes.

Que datos se observan en una radiografa de trax y una TAC con


bronquiectasias?
1) Radiografa de trax: Incremento inespecfico de la trama vascular,
atelectasias u opacidades irregulares
2) TAC: imagen en RIELES DE TRANVIA que refleja el engrosamiento
de las paredes y la fibrosis peribronquial.

Cual es el manejo de las bronquiectasias?


1) Agentes mucoliticos como acetilcisteina, salina hipertnica
2) Antimicrobianos en prevencin y para tratamiento
TUBERCULOSIS
Efecto adverso ms comn de la ISONIACIDA?
R = NEUROPATA PERIFRICA

Efecto adverso ms comn de la RIFAMPICINA?


R = Hemorrgicos, HEPATITIS.

Efecto adverso del ETAMBUTOL?


R = NEURITIS PTICA

Efecto adverso de ESTREPTOMICINA?


R = DAO AL VIII PAR CRANEAL, nefrotoxicidad.

Medicamentos para Tb en EMBARAZADA?


R = Isoniacida, Rifampicina y Etambutol por 4-8 semanas o 9 meses.

En una paciente con VIH + en el que usas isoniacida se debe usar que
medicamento para evitar sus reacciones adversas mas comunes?
R = VITAMINA B6
TUBERCULOSIS
Por que esta contraindicada la estreptomicina en el embarazo?
R = Puede causar sordera congnita.

Estn contraindicados los antituberculosos durante la lactancia?


R = No

Antes de usar el ETAMBUTOL que pruebas debes realizar?


R = AGUDEZA VISUAL, valorando colores rojo y verde.

Que es necesario medir en suero antes del uso de PIRAZINAMIDA?


R = ACIDO RICO

Que prueba es necesesaria realizar antes del uso de ESTREPTOMICINA?


R = AUDIOMETRICA.

Que agente etiolgico es el mas comn en enfermedad pulmonar por MNT?


R = M. Avium y M. Kanassi

Que frmacos utilizas en MNT mas antituberculosos?


R = Claritromicina o azitromicina.
CANCER PULMONAR
Cual es la neoplasia pulmonar benigna mas frecuente?
R = Hamartroma

Cuales son las neoplasias pulmonares malignas en cncer pulmonar?


1) Cncer broncognico de clulas pequeas (CPCP): representa el 20% de los canceres
pulmonares
2) Cncer broncognico de clulas no pequeas (CPCnP): representa el 80% de los
canceres pulmonares y abarca los subtipos de ADENOCARCINOMA 60%, escamoso o
epidermoide 30% y el carcinoma indiferenciado de clulas grandes < 10%

Que es el tumor de Pancoast en el cncer pulmonar?


R = Los carcinomas que se localizan en el surco pulmonar superior. Tiene una evolucin
lenta, tendencia a infiltrar los linfticos subpleurales y por contigidad las races
nerviosas del 8vo nervio cervical y del 2do y 3er dorsales.

Cuales son las manifestaciones clnicas del tumor de Pancoast?


1) DOLOR LOCALIZADO EN EL HOMBRO DE CARACTERSTICAS RADICULARES,
IRRADIADO A LO LARGO DEL BRAZO Y PARESTESIAS
2) SNDROME DE HORNER con debilidad y atrofia de los msculos extrnsecos

Cual es la incidencia de ca de clulas escamosas en cncer pulmonar primario?


R = 25-35%, se presenta con hemoptisis y se puede diagnosticar mediante citologa de
esputo.
CANCER PULMONAR
Cual es la estirpe etiolgica mas comn de ca pulmonar?
R = Adenocarcinoma 60 %

Cual es la presentacin tpica del adenocarcinoma pulmonar?


R = Masas o ndulos perifricos

Cual es el pronstico del CPCP?


R = Se propaga por VA HEMATOGENA muy rpido con SOBREVIDA MUY CORTA, de
aproximadamente 2m despus del diagnostico sin tratamiento.

Cual es el pronostico de CPCnP?


R = Se propaga mas lentamente con BUENA RESPUESTA AL TRATAMIENTO

Que porcentaje de pacientes con CPCP desarrolla secrecin inadecuada de ADH?


R = 15% ADH

Que porcentaje de pacientes con carcinoma de CPCnP de CLULAS ESCAMOSAS desarrolla


hipercalcemia?
R = 10% HIPERCALCEMIA

Cuales son los frmacos antineoplsicos mas novedosos?


R = Pacitaxel, gemcitabina, vinorelbina.
CARCINOIDE PULMONAR
Cuales son los sntomas mas comunes de los
tumores carcinoides bronquiales?
R = Hemoptisis, tos, sibilancias y neumona
RECURRENTE

Que caracteriza al sndrome carcinoide?


R = Rubores, diarrea, sibilancia e hipotensin.

FIBROSIS PULMONAR
Que enfermedad pulmonar te da el patrn radiogrfico llamado, en panal de abejas?
R = Fibrosis pulmonar y Enfermedad intersticial pulmonar o enfermedad parenquimatosa pulmonar
difusa

Cual es el nombre actual que recibe la fibrosis pulmonar idioptica?


R = Neumona intersticial fibrosante idioptica

Cual es el cuadro clnico y datos de espirometria de la fibrosis pulmonar idioptica?


1) Tos seca
2) Disnea que se incrementa con ejercicio
3) Capacidad vital forzada (CVF) disminuida
4) Relacin CVF/ VEF 1 dentro de los LIMITES NORMALES O AUMENTADA

Cuales son los criterios para fibrosis pulmonar idioptica?


1) Mayores: Exclusin de otras causas conocidas de enfermedad pulmonar intersticial, como toxicidad
por frmacos, exposicin ambiental y enfermedades del tejido conectivo. Pruebas de funcin
respiratoria que sugieran restriccin. Opacidades reticulares bibasales como mnimo VIDRIO
DESPULIDO en la TAC. Biopsia transbronquial o lavado broncoalveolar no compatible con otro
diagnostico.
2) Menores: Edad > 50. Disnea de esfuerzo. Mas de 3 meses con la enfermedad. Estertores tipo velcro
en regin subescapular bilateral.

Cual es el manejo de la fibrosis pulmonar idioptica?


R = Prednisona y azatriopina o ciclofosfamida.

Cual es el pronstico de la fibrosis pulmonar idioptica?


R = Malo con promedio de sobrevida de 1.
NEUMOCONIOSIS
Que es la neumoconiosis?
R = Enfermedad pulmonar provocada por la acumulacin de POLVOS MINERALES secundaria
a la inhalacin crnica de los mismo

Cuales son las patologas agrupadas en la neumoconiosis?


1) ASBESTOSIS por inhalacin de ASBESTO
2) SILICOSIS por la inhalacin de SLICE en su mayora en forma de cuarzo
3) SIDEROSIS por la acumulacin pulmonar de HIERRO
4) ANTRACOSIS por la exposicin secundaria a CARBON

Que caracteriza al mesotelioma, el cual es una complicacin de la asbestosis?


R = Son tumores primarios originados a partir del recubrimiento superficial de la pleura
(80%), del peritoneo (20%) y es mas comn en hombres 3:1.

Que caractersticas tiene el liquido pleural en el mesotelioma pulmonar?


R = Exudativo y hemorrgico

Que datos radiogrficos denota la silicosis?


R = Pequeas opacidades redondeadas en todo el pulmn, calcificacin de la periferia de los
ganglios linfticos hiliares CALCIFICACIN EN CASCARA DE HUEVO.

Que caracteriza al sndrome de Caplan?


R = Se caracteriza por NDULOS NECROBIOTICOS en periferia de pulmn en los
TRABAJADORES DE CARBN con ARTRITIS REUMATOIDE
TROMBOEMBOLIA PULMONAR
Cual es la causa hereditaria mas comn de hipercoagubilidad?
R = Resistencia a protena C activada o FACTOR V DE LEIDEN

Cual es la triada de Virchow identificada como factor de riesgo para desarrollo de


trombosis venosa?
1) Disminucin del flujo sanguneo o estasis venosa
2) Lesin o traumatismo vascular
3) Estado hipercoagulable o trombofilico

Datos que sugieren TEP?


1) DISNEA SBITA SOSTENIDA
2) TAQUICARDIA SOSTENIDA
3) Hipotensiona rterial sistlica < 100 mm Hg
4) SINCOPE
5) DOLOR PRECORDIAL ANTERIOR QUE SIMULA IAM
6) Colapso cardiorespiratorio

98. Cuales son las anormalidades EKG en pacientes con TEP?


1) Taquicardia sinusal
2) Patron S1, Q3 T3
TROMBOEMBOLIA PULMONAR
Que datos de laboratorio te sugieren TEP?
1) Valores plasmticos de DIMERO D, el cual es un producto de la degradacin de
fibrina, mismo que se aumenta en caso de trombos 300-500 mg/ul con ELISA.
2) Hipoxemia
3) ALCALOSIS RESPIRATORIA

Cual es el signo de Westemark en TEP?


R = ARTERIA PULMONAR CENTRAL PROMINENTE CON OLIGOHEMIA LOCAL en la
radiografa de trax. rea de vascularizacin y perfusin disminuida como
consecuencia de una embolia pulmonar, acompaada de un ensanchamiento de la
arteria pulmonar central del lado afectado. Se observa en el 2% de los casos.

Que mtodo sustituyo el diagnostico por imagen de TEP con gamagrama


pulmonar?
1) ANGIOGRAFA PULMONAR selectiva que se considera actualmente como
ESTANDAR DE ORO
2) TAC helicoidal

Cual es el estndar de oro en el tratamiento de la TEP?


1) HEPARINA
2) El manejo con acenocumarinicos (acenocumarina y warfarina) constituye el
tratamiento a largo plazo por va oral
TROMBOEMBOLIA PULMONAR
Como acta la heparina en TEP?
R = Se fija y acelera la capacidad de la antitrombina 3 para inactivar a la trombina, factores 9 y 10.

Como actan los acenocumarinicos en TEP?


R = EL mecanismo de accin depende de la inhibicin de la carboxilacion de los factores de coagulacin
2,7, 9 y 10 (vitamina K dependientes)

Cual es la meta del INR en TEP?


R = Que se encuentre entre 2-3

Que te indica un INR <2?


R = Aumenta el RIESGO DE TROMBOSIS

Que te indica un INR >4?


R = Aumenta el riesgo de HEMORRAGIA.

Como acta la estreptoquinasa, urocinasa y el activador tisular de plasminogeno?


R = Aumentan los valores de plasmina, lisando directamente los trombos intravasculares

Que medida adicional puede utilizarse en pacientes que esta contraindicado el uso de
anticoagulantes?
R = Bloqueo de la vena cava inferior

En pacientes quirrgicos de riesgo tromboembolico que medida de prevencin se utiliza?


R = Heparina de bajo peso molecular
HIPERTENSION PULMONAR
Cual es la definicin hemodinmica de la hipertensin pulmonar (HP)?
R = PRESIN ARTERIAL pulmonar media MAYOR A 25 MM HG EN REPOSO O DE 30 MM HG DURANTE EL
EJERCICIO.

Cual es la etiologa de la HP?


1) in situ Vasoconstriccin sostenida
2) Proliferacin vascular pulmonar
3) Trombosis
4) Disnea en reposo o de esfuerzo.
5) Disnea en reposo con dolor retroesternal semejante a angina de pecho.
6) Policitemia

Cuales son los sntomas que puede incluir la hipertensin pulmonar, adems de los datos de
laboratorio mas frecuentes?
Que refleja hipoxemia crnica.

Cuales son los datos EKG de HP?


R = Desviacin del eje a la derecha, hipertrofia ventricular derecha y crecimiento de aurcula derecha.

Que tratamiento esta indicado tanto en la HP primaria como secundaria?


R = Inhibidor de la fosfodiesterasa 5 (sindenafil), Anticuagulacin permanente, venoclisis prolongada
continua de prostaciclina +++

Como se maneja el Cor pulmonale como complicacin de HP?


R = Tratamiento de enfermedad de fondo, diurticos, disminucin de Na y O2
GRANULOMATOSIS DE WEGENER
En que consiste la granulomatosis de Wegener (vasculitis pulmonar)?
R = Es una enfermedad pulmonar ideopatica que se manifiesta por una combinacin de GLOMERULONEFRITIS,
VASCULITIS GRANULOMATOSA NECROSANTE de vas respiratorias superiores e inferiores y vasculitis de
pequeos vasos.

Cuales son los sntomas mas frecuentes en granulomatosis de Wegener?


R = SINUSITIS CRNICA, artralgias, exantema y disminucin de peso

Cual es el signo pulmonar mas comn en granulomatosis de Wegener?


R = Infiltrado pulmonar nodular con cavitaciones

Que prueba de lab te confirma granulomatosis de Wegener?


R = ANCA 90% con patrn de tincin citoplasmtica fluorecente cANCA

Como diagnosticas granulomatosis de Wegener sin laboratorio?


R = Con biopsia pulmonar, tejido sinusal o renal demostrando VASCULITIS GRANULOMATOSA NECROSANTE.

Cual es el tratamiento de la granulomatosis de Wegener o vasculitis pulmonar?


R = Corticosteroides y ciclofosfamida

Cual es el corticosteroide de eleccin en granulomatosis de Wegener?


R = PREDNISONA ORAL.

Cual es antibitico de eleccin en granulomatosis de Wegener?


R = TMP/SMZ
CHURG- STRAUSS
Que es el sindrome de Churg- Strauss?
El sndrome de Churg Strauss, tambin llamado angetis y granulomatosis alrgica, es un
sndrome multisistmico CARACTERIZADO POR RINITIS ALRGICA, ASMA Y EOSINOFILIA
importante en el extendido perifrico. El rgano ms comnmente comprometido es el
pulmn, seguido de la piel.
Sin embargo, el sndrome de Churg Strauss puede afectar cualquier rgano sistema,
incluyendo el sistema cardiovascular, el gastrointestinal, el renal y el sistema nervioso
central.

Cuales son manifestaciones clnicas del sndrome de Churg-Strauss?


Se manifiesta en tres fases:
1) Asma y rinitis alrgica.
2) Eosinofilia perifrica e infiltracin a rganos.
3) Vasculitis con afectacin sistmica:
a. Pulmonar: asma grave, tos, disnea, sinusitis.
b. Neurolgica: mononeuritis, polineuropata.
c. Cutnea: prpura, urticaria.
d. Cardiovascular: pericarditis, IAM.
e. Gastrointestinal: gastroenteritis eosinoflica, dolor abdominal, diarrea, hemorragia.

Que laboratorio se mantiene constantemente presente en el sx de Churg-Strauss?


R = Eosinofilia
CHURG- STRAUSS
Para su clasificacin el American College of Rheumatology (1990) public
los siguientes criterios diagnsticos contando con al menos cuatro de los
siguientes seis:

1) Asma: historia de dificultad respiratoria o sibilancias espiratorias difusas.


2) Eosinofilia: eosinofilia >10% en conteo diferencial de glbulos blancos.
3) Mono o polineuropata: desarrollo de mononeuropata, mononeuropata
mltiple, o polineuropata (distribucin en guante o en bota) atribuible a
vasculitis sistmica.
4) Infiltrados pulmonares migratorios: infiltrados pulmonares migratorios o
transitorios (no incluyendo infiltrados fijos). Atribuibles a vasculitis.
5) Anormalidades de los senos paranasales: historia de dolor agudo o
crnico de los senos paranasales u opacificacin radiogrfica de los senos
paranasales.
6) Eosinfilos extravasculares: biopsia incluyendo arterias, arteriolas o
vnulas mostrando acmulos de eosinfilos en reas extravasculares.
CHURG- STRAUSS
Cual es el manejo del sindrome de Churg Strauss?
A. La combinacin entre altas dosis de
corticoesteroides y ciclofosfamida son significativas
en el tratamiento de esta patologa con una baja
tasa de resistencia;
B. Los corticoesteroides pueden ser usados solos en
casos del sndrome de Churg Strauss, en cambio la
ciclofosfamida se utiliza solo en casos de pacientes
con ANCA positivo.
SNDROME DE GOOD PASTEUR
Menciona cuales son las causas de hemorragia alveolar difusa?
1) Enfermedad de ANTICUERPO ANTIMEMBRANA BASAL (sx de good pasteur),
2) Vasculitis y enfermedad de la colagena vascular (lupus eritematoso sistmico,
granulomatosis de Wegener).

Cuales son las causas de hemorragia alveolar difusa de causa no inmunitaria?


R = Coagulopatia, estenosis mitral, frmacos como la penicilamina y hemosiderosis
pulmonar idioptica.

Que es el sndrome de Good pasteur?


R = HEMORRAGIA ALVEOLAR recurrente idioptica y GLOMERULONEFRITIS rpidamente
progresiva que se manifiesta en varones de 30-40.

Como diagnosticas el sndrome de Good pasteur?


R = En la biopsia se observan depsitos lineales de IgG tanto en glomrulos como en
alveolos por medio de inmunofluorecencia y PRESENCIA DE AC CONTRA MEMBRANA BASAL
glomerular en suero.

Que tratamiento a dado resultado en el sndrome de Goodpasteur?


R = PREDNISONA, CLICLOFOSFAMIDA Y PLASMAFERESIS
SNDROME DE INSUFICIENCIA RESPIRATORIA
AGUDA
Como se define el sndrome de insuficiencia respiratorio (SIRA antes SIRPA)?
R = Pacientes con disnea, hipoxemia refractaria a tratamiento, disminucin de
la complianza pulmonar e infiltrados pulmonares difusos en la radiografa de
trax.

Cual es la etiologa del SIRA?


1) Causas de lesin pulmonar de manera indirecta como sobredosis de
frmacos
2) Causas de lesin pulmonar de manera directa como la broncoaspiracion o
politraumatismo

Cual es el cuadro clnico del SIRA?


1) Deterioro progresivo de la funcin respiratoria
2) Disnea en reposo
3) Hipoxemia grave Pa O2 < 50 mm Hg refractaria a la oxigenoterapia
4) ALCALOSIS RESPIRATORIA
5) Taquipnea
6) CIANOSIS
SNDROME DE INSUFICIENCIA
RESPIRATORIA AGUDA
Cuales son los criterios diagnsticos de SIRA?
R = Se diagnostica SIRA si cumple con 2 de los siguientes criterios
1) Inicio agudo o sbito de la sintomatologa
2) Opacidades pulmonares alveolares o en parches difusas bilaterales
3) Hipoxemia PaO2 < 60 mm Hg y relacin PaO2 / FiO2 < 200
4) Presin capilar pulmonar < 18 mm Hg o ausencia de signos de hipertensin
auricular izquierda en caso de no poder medir la presin en cuna pulmonar.

Cual es el manejo del SIRA?


1) Ventilacin mecnica invasiva de eleccin
2) Los frmacos no han demostrado beneficio en SIRA

Cual es el pronostico del SIRA?


R = Mortalidad entre 40-60%

Que maniobras han demostrado disminuir la mortalidad den SIRA?


1) El volumen corriente bajo (<6 ml/kg de peso ideal)
2) Mantenimiento de una presin meseta o plateau < 30 cm H2O
MICOSIS BRONCOPULMONAR
ALERGICA
Que es la micosis broncopulmonar alrgica (MBA)?
R = Trastorno de hipersensibilidad por alergia a los antgenos micticos.
Aspergilosis broncopulmonar alrgica.

Agente causal de MBA?


R = Especies de aspergilus.

Como dx MBA?
R = Tiene que reunir 6-7 criterios. 1) antecedente de asma 2) eosinofilia
perifrica 3)reactividad cutnea a aspergilus 4)Ac vs aspergilus 5) aumento
de IgE 6)bronquiectasias 7)infiltrados pulmonares.

Cual es el tratamiento de eleccin en MBA?


R = Prednisona
BIBLIOGRAFIA
EXARMED
PAPADAKIS
CTO
HARRISON

También podría gustarte